You are on page 1of 139

Tailieumontoan.

com


Sưu tầm và tổng hợp

BỘ ĐỀ THI TOÁN VÀO 10

KHOA HỌC TỰ NHIÊN HÀ NỘI

Tài liệu tổng hợp


1
Website:tailieumontoan.com

TUYỂN TẬP ĐỀ THI TOÁN


VÀO LỚP 10 CHUYÊN KHOA HỌC TỰ NHIÊN HÀ NỘI

LỜI NÓI ĐẦU

Nhằm đáp ứng nhu cầu về của giáo viên toán THCS và học sinh luyện thi vào lớp 10 môn
toán, website tailieumontoan.com giới thiệu đến thầy cô và các em bộ đề thi vào lớp 10 chuyên Đại
học khoa học tự nhiên Hà Nội. Đây là bộ đề thi mang tính chất thực tiễn cao, giúp các thầy cô và
các em học sinh luyện thi vào lớp 10 có một tài liệu bám sát đề thi để đạt được thành tích cao, mang
lại vinh dự cho bản thân, gia đình và nhà trường. Bộ đề gồm nhiều Câu toán hay được các thầy cô
trên cả nước sưu tầm và sáng tác, ôn luyện qua sẽ giúp các em phát triển tư duy môn toán từ đó
thêm yêu thích và học giỏi môn học này, tạo được nền tảng để có những kiến thức nền tốt đáp ứng
cho việc tiếp nhận kiến thức ở các lớp, cấp học trên được nhẹ nhàng và hiệu quả hơn.
Các vị phụ huynh và các thầy cô dạy toán có thể dùng có thể dùng tuyển tập đề toán này để
giúp con em mình học tập. Hy vọng Tuyển tập đề thi toán vào lớp 10 chuyên Đại học khoa học tự
nhiên này sẽ có thể giúp ích nhiều cho học sinh phát huy nội lực giải toán nói riêng và học toán nói
chung.
Mặc dù đã có sự đầu tư lớn về thời gian, trí tuệ song không thể tránh khỏi những hạn chế,
sai sót. Mong được sự góp ý của các thầy, cô giáo và các em học!
Chúc các thầy, cô giáo và các em học sinh thu được kết quả cao nhất từ bộ đề này!

Tài liệu sưu tầm và tổng hợp bản word đầy đủ liên hệ 0393732038 TÀI LIỆU TOÁN HỌC
2
Website:tailieumontoan.com

MỤC LỤC
PHẦN 1: ĐỀ THI

Đề số Đề thi Trang
1. Đề thi vào lớp 10 chuyên Đại học KHTN Hà Nội năm 2019 (vòng 1)
2. Đề thi vào lớp 10 chuyên Đại học KHTN Hà Nội năm 2019 (vòng 2)
3. Đề thi vào lớp 10 chuyên Đại học KHTN Hà Nội năm 2018 (vòng 1)
4. Đề thi vào lớp 10 chuyên Đại học KHTN Hà Nội năm 2018 (vòng 2)
5. Đề thi vào lớp 10 chuyên Đại học KHTN Hà Nội năm 2017 (vòng 1)
6. Đề thi vào lớp 10 chuyên Đại học KHTN Hà Nội năm 2017 (vòng 2)
7. Đề thi vào lớp 10 chuyên Đại học KHTN Hà Nội năm 2016 (vòng 1)
8. Đề thi vào lớp 10 chuyên Đại học KHTN Hà Nội năm 2016 (vòng 2)
9. Đề thi vào lớp 10 chuyên Đại học KHTN Hà Nội năm 2015 (vòng 1)
10. Đề thi vào lớp 10 chuyên Đại học KHTN Hà Nội năm 2015 (vòng 2)
11. Đề thi vào lớp 10 chuyên Đại học KHTN Hà Nội năm 2014 (vòng 1)
12. Đề thi vào lớp 10 chuyên Đại học KHTN Hà Nội năm 2014 (vòng 2)
13. Đề thi vào lớp 10 chuyên Đại học KHTN Hà Nội năm 2013 (vòng 1)
14. Đề thi vào lớp 10 chuyên Đại học KHTN Hà Nội năm 2013 (vòng 2)
15. Đề thi vào lớp 10 chuyên Đại học KHTN Hà Nội năm 2012 (vòng 1)
16. Đề thi vào lớp 10 chuyên Đại học KHTN Hà Nội năm 2012 (vòng 2)
17. Đề thi vào lớp 10 chuyên Đại học KHTN Hà Nội năm 2011 (vòng 1)
18. Đề thi vào lớp 10 chuyên Đại học KHTN Hà Nội năm 2011 (vòng 2)
19. Đề thi vào lớp 10 chuyên Đại học KHTN Hà Nội năm 2010 (vòng 1)
20. Đề thi vào lớp 10 chuyên Đại học KHTN Hà Nội năm 2010 (vòng 2)
21. Đề thi vào lớp 10 chuyên Đại học KHTN Hà Nội năm 2009 (vòng 1)
22. Đề thi vào lớp 10 chuyên Đại học KHTN Hà Nội năm 2009 (vòng 2)

PHẦN 2: HƯỚNG DẪN GIẢI

Tài liệu sưu tầm và tổng hợp bản word đầy đủ liên hệ 0393732038 TÀI LIỆU TOÁN HỌC
3
Website:tailieumontoan.com
SỞ GIÁO DỤC VÀ ĐÀO TẠO KỲ THI VÀO LỚP 10 CHUYÊN KHOA HỌC TỰ NHIÊN
HÀ NỘI NĂM HỌC 2019 – 2020
Môn: TOÁN (VÒNG 1)
ĐỀ CHÍNH THỨC Thời gian làm bài: 150 phút
(Không kể thời gian giao đề)
Đề thi gồm 01 trang

Đề số 1

Bài 1.

26 x + 5
a, Giải phương trình: + 2 26 x +=
5 3 x 2 + 30
x + 30
2

 x + y =
2 2
2
b, Giải hệ phương trình: 
( x + 2 y )(2 + 3 y + 4 xy ) =
2
27

Bài 2.

a, Tìm tất cả các cặp số nguyên thỏa mãn: ( x 2 − x + 1)( y 2 + xy ) = 3 x − 1

b, Với x,y là các số thực thay đổi thỏa mãn 1≤ y ≤ 2 và xy + 2 ≥ 2y, tìm giá trị nhỏ nhất của
x2 + 4
biểu thức M =
y2 +1

Bài 3. Cho hình vuông ABCD, đường tròn (O) nội tiếp hình vuông tiếp xúc với các cạnh
AB, AD tại hai điểm E,F. Gọi G là giao điểm các đường thẳng CE và BF.

a, Chứng minh rằng năm điểm A,F,O,G,E cùng nằm trên một đường tròn

b, Gọi giao điểm của đường thẳng FB và đường tròn là M(M ≠ F). CMR M là trung điểm
của đoạn thẳng BG.

c, CMR trực tâm của tam giác GAF nằm trên đường tròn (O)

Bài 4. Cho x, y, z là các số thực dương thỏa mãn xy + yz + xz = 1. Chứng minh rằng:

3
1 1 1 2 x y z 
+ + ≥  + + 
1+ x 1+ y
2 2
1+ z 2
3  1 + x2 1 + y2 1 + z2 
 

Tài liệu sưu tầm và tổng hợp bản word đầy đủ liên hệ 0393732038 TÀI LIỆU TOÁN HỌC
4
Website:tailieumontoan.com
SỞ GIÁO DỤC VÀ ĐÀO TẠO KỲ THI VÀO LỚP 10 CHUYÊN KHOA HỌC TỰ NHIÊN
HÀ NỘI NĂM HỌC 2019 – 2020
Môn: TOÁN (VÒNG 2)
ĐỀ CHÍNH THỨC Thời gian làm bài: 150 phút
(Không kể thời gian giao đề)
Đề thi gồm 01 trang

Đề số 2

Bài 1.
 3 x 2 + y 2 + 4 xy = 8
a. Giải hệ phương trình:  .
( x + y ) ( x + xy + 2 ) =
2
8

27 + x 2 + x 27 + 2 x
b. Giải phương trình: =
2 + 5 − ( x2 + x ) 2 + 5 − 2x

Bài 2.
a. Chứng minh rằng với mọi số nguyên dương n , ta luôn có
7 7 7
( 27 n + 5 )7 + 10  + (10n + 27 )7 + 5 + ( 5n + 10 )7 + 27 
     
chia hết cho 42 .
b. Với x, y là các số thực dương thay đổi thỏa mãn điều kiện
4 x 2 + 4 y 2 + 17 xy + 5 x + 5 y ≥ 1 .
Tìm giá trị nhỏ nhất của biểu thức: P = 17 x 2 + 17 y 2 + 16 xy .
Bài 3. Cho tam giác ABC cân tại A , có đường tròn nội tiếp ( I ) . Các điểm E , F theo thứ
tự thuộc các cạnh CA, AB ( E khác C và A ; F khác B và A ) sao cho EF tiếp xúc với
đường tròn ( I ) tại điểm P . Gọi K , L lần lượt là hình chiếu vuông góc của E , F trên BC .
Giả sử FK cắt EL tại điểm J . Gọi H là hình chiếu vuông góc của J trên BC .
a) Chứng minh rằng HJ là phân giác của góc EHF .
b) Kí hiệu S1 , S 2 lần lượt là diện tích của các tứ giác BFJL và CEJK . Chứng minh rằng
S1 BF 2
= .
S 2 CE 2
c) Gọi D là trung điểm của cạnh BC . Chứng minh rằng ba điểm P, , 
J D thẳng hàng.
Bài 4. Cho M là tập tất cả 4039 số nguyên liên tiếp từ −2019 đến 2019 . Chứng minh rằng
trong 2021 số đôi một phân biệt được chọn bất kì từ M luôn tồn tại ba số phân biệt có tổng
bằng 0 .

Tài liệu sưu tầm và tổng hợp bản word đầy đủ liên hệ 0393732038 TÀI LIỆU TOÁN HỌC
5
Website:tailieumontoan.com
SỞ GIÁO DỤC VÀ ĐÀO TẠO KỲ THI VÀO LỚP 10 CHUYÊN KHOA HỌC TỰ NHIÊN
HÀ NỘI NĂM HỌC 2018 – 2019
Môn: TOÁN (VÒNG 1)
ĐỀ CHÍNH THỨC Thời gian làm bài: 150 phút
(Không kể thời gian giao đề)
Đề thi gồm 01 trang

Đề số 3

Bài 1.
a) Giải phương trình x 2 − x + 2 x 3 + 1= 2 x + 1 .
 2
xy + y =1 + y
b) Giải hệ phương trình  2 2
.

 x + 2y + 2xy 4
=+ x
Bài 2.
a) Tìm tất cả các cặp số nguyên ( x; y ) thỏa mãn ( x + y )( 3x + 2y ) = 2x + y − 1 .
2

b
b) Với a, b là các số thực dương thay đổi thỏa mãn a + 2b =2 + . Tìm giá trị nhỏ
3
a b
nhất của biểu thức
= M + .
a + 2b b + 2a
Bài 3. Cho tam giác ABC có đường tròn nội tiếp ( I ) tiếp xúc với các cạnh BC, CA, AB lần
lượt tại các điểm D, E, F. Gọi K là hình chiếu vuông góc của B trên đường thẳng DE và M
là trung điểm của đoạn thẳng DF.
a) Chứng minh rằng hai tam giác BKM và DEF đồng dạng với nhau.
b) Gọi L là hình chiếu của vuông góc của C trên đường thẳng DF và N là trung điểm
của đoạn thẳng DE. Chứng minh rằng hai đường thẳng MK và NL song song với nhau.
c) Gọi J, X lần lượt là trung điểm của các đoạn thẳng KL và ID. Chứng minh rằng
đường thẳng JX vuông góc với đường thẳng EF.
Bài 4. Trên mặt phẳng cho hai điểm P và Q phân biệt. Xét 10 đường thẳng nằm trong
mặt phẳng trên thỏa mãn các tính chất sau:
i) Không có hai đường thẳng nào song song hoặc trùng nhau.
ii) Mỗi đường thẳng đi qua P hoặc Q, không có đường thẳng nào đi qua cả P và Q.
Hỏi 10 đường thẳng trên có thể chia mặt phẳng thành tối đa bao nhiêu miền? Hãy giải
thích.

Tài liệu sưu tầm và tổng hợp bản word đầy đủ liên hệ 0393732038 TÀI LIỆU TOÁN HỌC
6
Website:tailieumontoan.com
SỞ GIÁO DỤC VÀ ĐÀO TẠO KỲ THI VÀO LỚP 10 CHUYÊN KHOA HỌC TỰ NHIÊN
HÀ NỘI NĂM HỌC 2018 – 2019
Môn: TOÁN (VÒNG 2)
ĐỀ CHÍNH THỨC Thời gian làm bài: 150 phút
(Không kể thời gian giao đề)
Đề thi gồm 01 trang

Đề số 4

Câu 1.

 xy ( x + y ) =2
a) Giải hệ phương trình :  3
 x + y + x y + 7 ( x + 1) ( y + 1) =
3 3 3
31

b) Giải phương trình: 9 + 3 x ( 3 − 2 x ) = 7 x + 5 3 − 2 x

Câu 2.
a) Cho x, y là các số nguyên sao cho x 2 − 2 xy − y 2 ; xy − 2 y 2 − x đều chia hết cho 5.
Chứng minh 2 x 2 + y 2 + 2 x + y cũng chia hết cho 5
b) Cho a1 , a2 ,......, a50 là các số nguyên thỏa mãn: 1 ≤ a1 ≤ a2 ...... ≤ a50 ≤ 50 ,
100 . Chứng minh rằng từ các số đã cho có thể chọn được một vài
a1 + a2 + ..... + a50 =
số có tổng là 50
Câu 3. Cho ngũ giác lồi ABCDE nội tiếp (O) có CD / / BE . Hai đường chéo CE và BD cắt
 = PAE
nhau tại P. Điểm M thuộc BE sao cho MAB  . Điểm K thuộc AC sao cho MK song

song AD, điểm L thuộc đường thẳng AD sao cho ML // AC. Đường tròn ngoại tiếp tam
giác KBC cắt BD, CE tại Q và S (Q khác B, S khác C)
a) Chứng minh 3 điểm K, M, Q thẳng hàng
b) Đường tròn ngoại tiếp tam giác LDE cắt BD, CE tai T và R (T khác D, R khác E).
Chứng minh M, S, Q, R,T cùng thuộc một đường tròn
c) Chứng minh đường tròn ngoại tiếp tam giác PQR tiếp xúc (O)
Câu 4. Cho a, b, c là các số thực dương. Chứng minh rằng
 ab bc   1 1 
 +   + ≤2
 a + b b + c   a + b b + c 

Tài liệu sưu tầm và tổng hợp bản word đầy đủ liên hệ 0393732038 TÀI LIỆU TOÁN HỌC
7
Website:tailieumontoan.com
SỞ GIÁO DỤC VÀ ĐÀO TẠO KỲ THI VÀO LỚP 10 CHUYÊN KHOA HỌC TỰ NHIÊN
HÀ NỘI NĂM HỌC 2017 – 2018
Môn: TOÁN (VÒNG 1)
ĐỀ CHÍNH THỨC Thời gian làm bài: 150 phút
(Không kể thời gian giao đề)
Đề thi gồm 01 trang

Đề số 5

Câu 1. (3.5 điểm)


x 2  y 2  xy  1

a) Giải hệ phương trình 
x  x 2y  2y 3


b) Giải phương trình 2 x  1 x  1   


x  1  1 x 2  1 x2 
Câu 2 (2.5 điểm)

a) Chứng minh rằng không tồn tại các số nguyên x, y thỏa mãn đẳng thức

12x 2  26xy  15y 2  4617

b) Với a, b là các số thực dương. Tìm giá trị lớn nhát của biểu thức
 1 1  1
M  a  b  2  2  
a  b b  a  ab

Câu 3 (3.0 điểm)


  900 . Đường tròn tâm I nội tiếp tam giác ABD tiếp xúc
Cho hình thoi ABCD có BAD

với BD và BA lần lượt tại J và L. Trên đường thẳng IJ lấy điểm K sao cho BK song song ID.
  ABI
a) Chứng minh rằng CBK .

b) Chứng minh rằng KC  KB .

c) Chứng minh rằng bốn điểm C, K, I ,L cùng nằm trên một đường tròn.

Câu 4. (1.0 điểm)

Tìm tập hợp số nguyên dương n sao cho tồn tại một cách sắp xếp các số 1;2; 3;...;n

thành a1; a2 ; a 3 ;...; an mà khi chia các số a1; a1a2 ; a1a2a 3 ;...; a1a2 ...a n cho n ta được các số dư đôi

một khác nhau.

Tài liệu sưu tầm và tổng hợp bản word đầy đủ liên hệ 0393732038 TÀI LIỆU TOÁN HỌC
8
Website:tailieumontoan.com
SỞ GIÁO DỤC VÀ ĐÀO TẠO KỲ THI VÀO LỚP 10 CHUYÊN KHOA HỌC TỰ NHIÊN
HÀ NỘI NĂM HỌC 2017 – 2018
Môn: TOÁN (VÒNG 2)
ĐỀ CHÍNH THỨC Thời gian làm bài: 150 phút
(Không kể thời gian giao đề)
Đề thi gồm 01 trang

Đề số 6

Câu 1 (3.5 điểm).




x  y  x  3y
1. Giải hệ phương trình   2

x  y 2  xy  3


2. Với a, b là các số thực dương thỏa mãn ab  a  b  1 . Chứng minh rằng:
a b 1  ab
 
  
2
1a 1  b2 2 1  a 2 1  b2

Câu 2 (2.5 điểm).


1. Giả sử p và q là các số nguyên tố thỏa mãn đẳng thức p p  1  q q 2  1 .

a) Chứng minh rằng tồn tại số nguyên dương k sao cho p  1  kq, q 2  1  kp .
b) Tìm tất cả các số nguyên tố p, q thỏa mãn đẳng thức p p  1  q q 2  1 .
2. Với a, b, c là các số thực dương thỏa mãn ab  bc  ca  abc  2 . Tìm giá trị lớn
a 1 b 1 c 1
nhất của biểu thức M   2
2
 2
a  2a  2 b  2b  2 c  2c  2
Câu 3 (3.0 điểm). Cho tam giác ABC nhọn với AB  AC . Gọi E, F lần lượt là trung điểm
 và nằm
của CA, AB. Đường trung trực của EF cắt BC tại D. Giả sử P nằm trong góc EAF
 D
ngoài tam giác AEF sao cho PEC    DFE
EF và PEB  . Đường thẳng PA cắt đường
tròn ngoại tiếp tam giác PEF tại Q khác P.
a) Chứng minh rằng EQF  BAC   EDF .

b) Tiếp tuyến tại P của đường tròn ngoại tiếp tam giác PEF cắt CA, AB lần lượt tại M,
N. Chứng minh rằng bốn điểm C, M, B, N cùng nằ trên một đường tròn. Gọi đường tròn
này là đường tròn K  .
c) Chứng minh rằng đường tròn K  tiếp xúc với đường tròn ngại tiếp tam giác AEF.
Câu 4 (1.0 điểm). Cho n là số nguyên dương với n  5 . Xét đa giác lồi n cạnh. Người ta
muốn kẻ một số đường chéo của đa giác mà các đường chéo này chia đa giác thành đúng
k miền, mõi miền là mọt ngũ giác lồi (hai miền bất kì không có điểm chung trong).
a) Chứng minh rằng ta có thể thực hiện được với n  2018, k  672
b) Với n  2017, k  672 ta có thể thực hiện được không? Hãy giải thích.

Tài liệu sưu tầm và tổng hợp bản word đầy đủ liên hệ 0393732038 TÀI LIỆU TOÁN HỌC
9
Website:tailieumontoan.com
SỞ GIÁO DỤC VÀ ĐÀO TẠO KỲ THI VÀO LỚP 10 CHUYÊN KHOA HỌC TỰ NHIÊN
HÀ NỘI NĂM HỌC 2016 – 2017
Môn: TOÁN (VÒNG 1)
ĐỀ CHÍNH THỨC Thời gian làm bài: 150 phút
(Không kể thời gian giao đề)
Đề thi gồm 01 trang

Đề số 7

Câu 1 (3.5 điểm).

x + y + xy x + y = 4

 3 3

a) Giải hệ phương trình 





 
xy + 1 x2 + y 2 = 4 
8x  3
b) Giải phương trình 7x  2  5  x 
5
Câu 2 (2.5 điểm)
a)Tìm tất cả các giá tri của m sao cho tồn tại cặp số nguyên x ; y  thỏa mãn hệ phương
2  mxy 2  3m

trình : 
 
2  m x 2  y 2  6m

b) Với x, y là những số thực thỏa mãn các điều kiện 0  x  y  2;2x  y  2xy .
Tìm giá trị lớn nhất của biểu thức P  x 2 x 2  1  y2 y2  1

Câu 3 (3.0 điểm). Cho tam giác ABC nhọn nội tiếp đường tròn O  với AB  AC . Phân
 cắt BC tại D và cắt đường tròn O tại E khác A. M là trung điểm của
giác của góc BAC  
đoạn thẳng AD. Đường thẳng BM cắt đường tròn O  tại P khác B. Giả sử các đường

thẳng EP và AC cắt nhau tại N.


a) chứng minh rằng tứ giác APNM nội tiếp và N là trung điểm của đoạn thẳng AC.
b) Giả sử đường tròn K  ngoại tiếp tam giác EMN cắt đường thẳng AC tại Q khác N.

Chứng minh rằng B và Q đối xứng nhau qua AE.


c) Giả sử đường tròn K  cắt đường thẳng BM tại M. Chứng minh rằng RA vuông góc RC.
Câu 4 (1.0 điểm).
Số nguyên a được gọi là số “đẹp” nếu với mọi cách sắp xếp theo thứ tự tùy ý của 100
số 1, 2, 3,…, 100 luôn tồn tại 10 số hạng liên tiếp có tổng lớn hơn hoặc bằng a. Tìm số
“đẹp” lớn nhất

Tài liệu sưu tầm và tổng hợp bản word đầy đủ liên hệ 0393732038 TÀI LIỆU TOÁN HỌC
10
Website:tailieumontoan.com
SỞ GIÁO DỤC VÀ ĐÀO TẠO KỲ THI VÀO LỚP 10 CHUYÊN KHOA HỌC TỰ NHIÊN
HÀ NỘI NĂM HỌC 2016 – 2017
Môn: TOÁN (VÒNG 2)
ĐỀ CHÍNH THỨC Thời gian làm bài: 150 phút
(Không kể thời gian giao đề)
Đề thi gồm 01 trang

Đề số 8

Câu 1(3.5 điểm).


x 2  4y 2  5

a) Giải hệ phương trình  2
4x  8xy 2  5x  10y  1

64x 3  4x
b) giải phương trình 2
5x  6x  5  2
5x  6x  6
Câu 2 (2.5 điểm).
x 2  1 y2  1
a) Với x, y là những số nguyên thỏa mãn đẳng thức  . Chứng minh
2 3
x 2  y 2  40 .

b) Tìm tất cả các cặp số nguyên x ; y  thỏa mãn đẳng thức sau x 4  2x 2  y 3 .

Câu 3 (3.0 điểm)


Cho hình vuông ABCD nội tiếp đường tròn O  . P là điểm thuộc cung nhỏ AD của

đường tròn O  và P khác A, D. Các đường thẳng PB, PC lần lượt cắt AD tại AD tại M, N.
Đường trung trực của AM cắt đường thẳng AC, PB lần lượt tại E, K. Đường trung trực DN
cắt các đường thẳng BD, PC lần lượt tại F, L.
a) Chứng minh rằng ba điểm K, O, L thẳng hàng.
b) Chứng minh đường thẳng PO đi qua trung điểm của EF
c) Giả sử đường thảng EK cắt đường thẳng FL và AC cắt nhau tại T. Đường thẳng
ST cắt các đường thẳng PB, PC lần lượt tại U và V. Chứng minh rằng bốn điểm K, L, V, U
cùng thuộc một đương tròn.
Câu 4(1.0 điểm)
Chứng minh rằng với mọi số tự nhiên n  3 luôn tồn tại cách xếp bộ n số 1,2, 3,...,n
xi  xk
thành bộ số x 1, x 2 , x 3 ,..., x n sao cho x j  với mọi bộ chỉ số i; j ; k  mà
2
1i  j k n .

Tài liệu sưu tầm và tổng hợp bản word đầy đủ liên hệ 0393732038 TÀI LIỆU TOÁN HỌC
11
Website:tailieumontoan.com
SỞ GIÁO DỤC VÀ ĐÀO TẠO KỲ THI VÀO LỚP 10 CHUYÊN KHOA HỌC TỰ NHIÊN
HÀ NỘI NĂM HỌC 2015 – 2016
Môn: TOÁN (VÒNG 1)
ĐỀ CHÍNH THỨC Thời gian làm bài: 150 phút
(Không kể thời gian giao đề)
Đề thi gồm 01 trang

Đề số 9

Câu 1. (3,0 điểm).


1). Giả sử a; b là hai số thực phân biệt thỏa mãn a 2  3a  b2  3b  2 .
a). Chứng minh rằng a  b  3 .
b). Chứng minh rằng a 3  b3  45 .

2 x  3 y  5 xy
2). Giải hệ phương trình  2 .
 2 2
4 x  y  5 xy

Câu 2. (3,0 điểm).
1). Tìm các số nguyên  x; y không nhỏ hơn 2 sao cho xy  1 chia hết cho  x  1 y  1 .
2). Với x; y là những số thực thỏa mãn đẳng thức x 2 y 2  2 y  1  0. Tìm giá trị lớn nhất và
xy
nhỏ nhất của biểu thức P  .
3y  1
Câu 3. (3,0 điểm). Cho tam giác nhọn ABC không cân có tâm đường tròn nội tiếp là điểm
I . Đường thẳng AI cắt BC tại D . Gọi E; F lần lượt là các điểm đối xứng của D qua
IC ; IB .
1). Chứng minh rằng EF song song với BC .
2). Gọi M ; N ; J lần lượt là trung điểm các đoạn thẳng DE; DF ; EF . Đường tròn ngoại tiếp
tam giác AEM cắt đường tròn ngoại tiếp tam giác AFN tại P khác A . Chứng minh rằng
bốn điểm M ; N ; P; J cùng nằm trên một đường tròn.
3). Chứng minh rằng ba điểm A; J ; P thẳng hàng.
Câu 4. (1,0 điểm).
1). Cho bảng ô vuông 2015 x2015. Kí hiệu ô (i ; j ) là ô ở hang
thứ i , cột thứ j . Ta viết các số nguyên dương từ 1 đến 2015
vào các ô của bảng theo quy tắc sau:
i). Số 1 được viết vào ô (1,1)
ii). Nếu số k được viết vào ô (i ; j ) ( i  1 ) thì số k  1 được
viết vào ô (i  1; j  1) .
iii). Nếu số k được viết vào ô (1; j ) thì số k  1 được viết vào ô ( j  1; 1) (xem hình 1).
Khi đó số 2015 được viết vào ô m; n .
Hãy xác định m và n.
2). Giả sử a; b; c là các số thực dương thỏa mãn ab  bc  ac  abc  4.
Chứng minh rằng a 2  b2  c 2  a  b  c  2 ab  bc  ac .

Tài liệu sưu tầm và tổng hợp bản word đầy đủ liên hệ 0393732038 TÀI LIỆU TOÁN HỌC
12
Website:tailieumontoan.com
SỞ GIÁO DỤC VÀ ĐÀO TẠO KỲ THI VÀO LỚP 10 CHUYÊN KHOA HỌC TỰ NHIÊN
HÀ NỘI NĂM HỌC 2015 – 2016
Môn: TOÁN (VÒNG 2)
ĐỀ CHÍNH THỨC Thời gian làm bài: 150 phút
(Không kể thời gian giao đề)
Đề thi gồm 01 trang

Đề số 10

Câu 1. (3,0 điểm).


1) Với a; b; c là các số thỏa mãn
3 3 3 3
3a  3b  3c  24  3a  b  c  3b  c  a  3c  a  b .

Chứng minh rằng a  2bb  2cc  2 a  1 .



2 x  2 y  xy  5
2) Giải hệ phương trình  .
27  x  y  y  7  26 x 3  27 x 2  9 x
3

Câu 2. (3,0 điểm).
1) Tìm số tự nhiên n để n  5 và n  30 là số chính phương (số chính phương là bình
phương của một số nguyên)
2) Tìm  x; y nguyên thỏa mãn đẳng thức 1  x  y  3  x  y .
3) Giả sử x; y; z là các số thực lớn hơn 2. Tìm giá trị nhỏ nhất của biểu thức:
x y z
P   .
y  z4 z x4 x y4

Câu 3. (3,0 điểm). Cho tam giác ABC nhọn không cân với AB  AC . Gọi M là trung
điểm của đoạn thẳng BC . Gọi H là hình chiếu vuông góc của B trên đoạn AM . Trên tia
đối của tia AM lấy điểm N sao cho AN  2 MH .
1) Chứng minh rằng BN  AC .
2) Gọi Q là điểm đối xứng với A qua N . Đường thẳng AC cắt BQ tại D .Chunwgs minh
rằng bốn điểm B; D; N ; C cùng thuộc một đường tròn,gọi đường tròn này là (O) .
3) Đường tròn ngoại tiếp tam giác AQD cắt (O) tại G và D . Chứng minh rằng NG song
song với BC .
Câu 4. (1,0 điểm). Ký hiệu S là tập hợp gồm 2015 diểm phân biệt trên một mặt phẳng. Giả
sử tất cả các điểm của S không cùng nằm trên một đường thẳng. Chứng minh rằng có ít
nhất 2015 đường thẳng phân biệt mà mỗi đường thẳng đi qua ít nhất hai điểm của S .

Tài liệu sưu tầm và tổng hợp bản word đầy đủ liên hệ 0393732038 TÀI LIỆU TOÁN HỌC
13
Website:tailieumontoan.com
SỞ GIÁO DỤC VÀ ĐÀO TẠO KỲ THI VÀO LỚP 10 CHUYÊN KHOA HỌC TỰ NHIÊN
HÀ NỘI NĂM HỌC 2014 – 2015
Môn: TOÁN (VÒNG 2)
ĐỀ CHÍNH THỨC Thời gian làm bài: 150 phút
(Không kể thời gian giao đề)
Đề thi gồm 01 trang

Đề số 11

Câu 1. 1) Giải phương trình ( 1+ x + 1− x )(2 + 2 )


1 − x2 =
8.

 x − xy + y =
2 2
1
2) Giải hệ phương trình  2
 x + xy + 2 y =
2
4

Câu 2. 1) Giả sử x, y, z là ba số dương thỏa mãn điều kiện x + y + z = xyz. Chứng minh
x 2y 3z xyz ( 5 x + 4 y + 3 z )
rằng: + + = .
1+ x 1+ y
2 2
1+ z 2
( x + y )( y + z )( z + x )
2) Tìm nghiệm nguyên của phương trình: x 2 y 2 ( x + y ) + x + y + 3 + xy.

Câu 3. Cho tam giác ABC nhọn với AB < BC. Gọi D là điểm thuộc cạnh BC sao cho AD là
phân giác của ∠BAC . Đường thẳng qua C song song với AD cắt trung trực của AB tại F.
1) Chứng minh tam giác ABF đồng dạng với tam giác ACE.
2) Chứng minh rằng các đường thẳng BE, CF, AD đồng quy tại một điểm, gọi điểm đó là G.
3) Đường thẳng qua G song song với AE cắt đường thẳng BF tại Q. Đường thẳng QE cắt
đường tròn ngoại tiếp tam giác GEC tại P khác E. Chứng minh rằng các điểm A, P, G, Q, F
cùng thuộc một đường tròn.
Câu 4. Giả sử a, b, c là các số thực dương thỏa mãn đẳng thức ab + bc + ca = 1 . Chứng
5
minh rằng 2abc ( a + b + c ) ≤ + a 4b2 + b4 c2 + c4 a 2 .
9

Tài liệu sưu tầm và tổng hợp bản word đầy đủ liên hệ 0393732038 TÀI LIỆU TOÁN HỌC
14
Website:tailieumontoan.com
SỞ GIÁO DỤC VÀ ĐÀO TẠO KỲ THI VÀO LỚP 10 CHUYÊN KHOA HỌC TỰ NHIÊN
HÀ NỘI NĂM HỌC 2014 – 2015
Môn: TOÁN (VÒNG 2)
ĐỀ CHÍNH THỨC Thời gian làm bài: 150 phút
(Không kể thời gian giao đề)
Đề thi gồm 01 trang

Đề số 12

Câu 1.
y 2 y2 4 y4 8 y4
1. Giả sử x, y là những số thực dương thỏa mãn: + 2 + + =
4
x + y x + y 2 x 4 + y 4 x8 − y 4
Chứng minh rằng: 5y = 4x

2 x − 3 y + xy =
2 2
12
2. Giải hệ phương trình: 
6 x + x y =12 + 6 y + y x

2 2

Câu 2.
1. Cho x, y là những số nguyên lớn hơn 1 sao cho 4x2y2 – 7x + 7y là số chính phương.
Chứng minh rằng: x = y.
2. Giả sử x, y là những số thực không âm thỏa mãn: x3 + y3 + xy = x2 + y2 Tìm giá trị
1+ x 2 + x
lớn nhất và nhỏ nhất của biểu thức:
= P +
2 + y 1+ y
Câu 3. Cho tam giác ABC nội tiếp đường tròn (O) và điểm P nằm trong tam giác thỏa mãn
PB = PC. D là điểm thuộc cạnh BC (D khác B và D khác C) sao cho P nằm trong đường tròn
ngoại tiếp tam giác DAC và đường tròn ngoại tiếp tam giác DAC. Đường thẳng PB cắt
đường tròn ngoại tiếp tam giác DAB tại E khác B. Đường thẳng PC cắt đường tròn ngoại
tiếp tam giác DAC tại F khác C.
1. Chứng minh rằng bốn điểm A, E, B, F cùng thuộc một đường tròn.
2. Giả sử đường thẳng AD cắt đường tròn (O) tại Q khác A, đường thẳng AF cắt đường
thẳng QC tại L. Chứng minh rằng tam giác ABE đồng dạng với tam giác CLF.
3. Gọi K là giao điểm của đường thẳng AE và đường thẳng QB. Chứng minh rằng:
 + PAB
QKL  = QLK + PAC
.

Câu 4. Cho tập hợp A gồm 31 phần tử và dãy gồm m tập hợp của A thỏa mãn đồng thời
các điều kiện sau:
i) Mỗi tập hợp thuộc dãy có ít nhất hai phần tử.
ii) Nếu hai tập hợp thuộc dãy có chung nhau ít nhất hai phần tử thì số phần tử
của hai tập hợp này khác nhau.
Chứng minh rằng: m ≤ 900

Tài liệu sưu tầm và tổng hợp bản word đầy đủ liên hệ 0393732038 TÀI LIỆU TOÁN HỌC
15
Website:tailieumontoan.com
SỞ GIÁO DỤC VÀ ĐÀO TẠO KỲ THI VÀO LỚP 10 CHUYÊN KHOA HỌC TỰ NHIÊN
HÀ NỘI NĂM HỌC 2013 – 2014
Môn: TOÁN (VÒNG 1)
ĐỀ CHÍNH THỨC Thời gian làm bài: 150 phút
(Không kể thời gian giao đề)
Đề thi gồm 01 trang

Đề số 13

Câu 1. (3,0 điểm).


1). Giải phương trình
3x  1  2  x  3 .
2). Giải hệ phương trình

 1 1 9

 xy  

 x y 2
 .

 1 3  1  1

   x    xy 


 4 2  y  xy

Câu 2. (3,0 điểm).


1). Cho các số thực a; b; c  0 thỏa mãn a  bb  cc  a  8 abc . Chứng minh rằng

a b c 3 ab bc ca
      .
a  b b  c c  a 4 a  bb  c b  cc  a c  aa  b

2). Có bao nhiêu số nguyên dương có 5 chữ số abcde sao cho abc  10d  e chia hết cho
101 ?
Câu 3. (3,0 điểm).
Cho tam giác nhọn ABC nội tiếp đường tròn (O) với AB  AC. Đường phân giác
 cắt (O) tại điểm D khác A. Gọi M là trung điểm của AD và E là điểm đối
của góc BAC
xứng với D qua tâm O . Giả sử đường tròn ngoại tiếp tam giác ABM cắt đoạn thẳng AC
tại điểm F khác A.
1). Chứng minh rằng tam giác BDM và tam giác BCF đồng dạng.
2). Chứng minh rằng EF vuông góc với AC.
Câu 4. (1,0 điểm).
Giả sử a; b; c; d là các số thực dương thỏa mãn điều kiện abc  bcd  cda  dab  1.
Tìm giá trị nhỏ nhất của biểu thức
P  4  a 3  b 3  c 3   9d 3 .

Tài liệu sưu tầm và tổng hợp bản word đầy đủ liên hệ 0393732038 TÀI LIỆU TOÁN HỌC
16
Website:tailieumontoan.com
SỞ GIÁO DỤC VÀ ĐÀO TẠO KỲ THI VÀO LỚP 10 CHUYÊN KHOA HỌC TỰ NHIÊN
HÀ NỘI NĂM HỌC 2013 – 2014
Môn: TOÁN (VÒNG 2)
ĐỀ CHÍNH THỨC Thời gian làm bài: 150 phút
(Không kể thời gian giao đề)
Đề thi gồm 01 trang

Đề số 14

Câu 1. (3,0 điểm).


1). Giải hệ phương trình
x 3  y 3  1  y  x  xy
 .

7 xy  y  x  7

2). Giải phương trình

x  3  1  x2  3 x  1  1  x .
Câu 2. (3,0 điểm).
1). Tìm các cặp số nguyên  x; y thỏa mãn

5 x 2  8 y 2  20412 .

2). Với  x; y là các số thực dương thỏa mãn x  y  1, tìm giá trị nhỏ nhất của biểu thức
1 1
P     1  x 2 y 2 .
 x y 

Câu 3. (3,0 điểm). Cho tam giác nhọn ABC nội tiếp đường tròn (O) có trực tâm H . Gọi P
là điểm nằm trên đường tròn ngoại tiếp tam giác HBC ( P khác B, C và H ) và nằm trong
tam giác ABC . PB cắt (O) tại M khác B, PC cắt (O) tại N khác C . BM cắt AC tại E,
CN cắt AB tại F . Đường tròn ngoại tiếp tam giác AME và đường tròn ngoại tiếp tam
giác ANF cắt nhau tại Q khác A.
1). Chứng minh rằng ba điểm M ; N ; Q thẳng hàng.
 . Chứng minh rằng khi đó PQ đi qua trung điểm của
2). Giả sử AP là phân giác góc MAN
BC.
Câu 4. (1,0 điểm). Giả sử dãy số thực có thứ tự x1  x2    x192 thỏa mãn các điều kiện
x1  x2    x192  0 và x1  x2    x192  2013.

Chứng minh rằng


2013
x192  x1  .
96

Tài liệu sưu tầm và tổng hợp bản word đầy đủ liên hệ 0393732038 TÀI LIỆU TOÁN HỌC
17
Website:tailieumontoan.com
SỞ GIÁO DỤC VÀ ĐÀO TẠO KỲ THI VÀO LỚP 10 CHUYÊN KHOA HỌC TỰ NHIÊN
HÀ NỘI NĂM HỌC 2012 – 2013
Môn: TOÁN (VÒNG 1)
ĐỀ CHÍNH THỨC Thời gian làm bài: 150 phút
(Không kể thời gian giao đề)
Đề thi gồm 01 trang

Đề số 15

Câu 1. (3,0 điểm).

1). Giải phương trình: x  9  2012 x  6  2012  x  9x  6 .



x 2  y 2  2 y  4
2). Giải hệ phương trình  .

2 x  y  xy  4

Câu 2. (3,0 điểm).

1). Tìm tất cả các cặp số nguyên  x; y thỏa mãn đẳng thức

x  y  1xy  x  y  5  2  x  y .

2). Giả sử  x; y là các số thực dương thỏa mãn điều kiện  x 1 
y  1  4 . Tìm giá trị

nhỏ nhất của biểu thức

x2 y 2
P  .
y x

Câu 3. (3,0 điểm). Cho tam giác nhọn ABC nội tiếp đường tròn tâm O . Gọi M là một điểm
 ( M khác B; C và AM không đi qua O ). Giả sử P là một điểm thuộc đoạn
trên cung nhỏ BC

 tại điểm N khác M .


thẳng AM sao cho đường tròn đường kính MP cắt cung nhỏ BC

1). Gọi D là điểm đối xứng với điểm M qua O . Chứng minh rằng ba điểm N ; P; D thẳng

hàng.

2). Đường tròn đường kính MP cắt MD tại điểm Q khác M . Chứng minh rằng P là tâm

đường tròn nội tiếp tam giác AQN .

Câu 4. (1,0 điểm). Giả sử a; b; c là các số thực dương thỏa mãn: a  b  3  c ; c  b  1 ;

2 ab  a  b  c ab  1
a  b  c . Tìm giá trị nhỏ nhất của biểu thức: Q .
a  1b  1c  1

Tài liệu sưu tầm và tổng hợp bản word đầy đủ liên hệ 0393732038 TÀI LIỆU TOÁN HỌC
18
Website:tailieumontoan.com
SỞ GIÁO DỤC VÀ ĐÀO TẠO KỲ THI VÀO LỚP 10 CHUYÊN KHOA HỌC TỰ NHIÊN
HÀ NỘI NĂM HỌC 2012 – 2013
Môn: TOÁN (VÒNG 2)
ĐỀ CHÍNH THỨC Thời gian làm bài: 150 phút
(Không kể thời gian giao đề)
Đề thi gồm 01 trang

Đề số 16

Câu 1. (3,0 điểm).

1). Giải hệ phương trình



xy  x  y  2

 .




9 xy  3 x  y   6  26 x 3
 2 y 3

2). Giải phương trình

 x4 2  
4  x  2  2x .

Câu 2. (3,0 điểm).

1). Tìm hai chữ số cuối cùng của số

A  41106  57 2012 .

1 5
2). Tìm giá trị lớn nhất của hàm số y  3 2 x  1  x 5  4 x 2 , với x .
2 2

Câu 3. (3,0 điểm). Cho tam giác nhọn ABC ( AB  AC ) nội tiếp đường tròn (O) . Giả sử

M; N  sao cho MN song song với BC và tia AN nằm


là hai điểm thuộc cung nhỏ BC

giữa hai tia AM ; AB . Gọi P là hình chiếu vuông góc của điểm C trên AN và Q là hình

chiếu vuông góc của điểm M trên AB .

1). Giả sử CP cắt QM tại điểm T . Chứng minh T nằm trên đường tròn (O) .

2). Gọi giao điểm của NQ và (O) là R khác N . Giả sử AM cắt PQ tại S . Chứng minh

rằng bốn điểm A; R; Q; S cùng thuộc một đường tròn.

Câu 4. (1,0 điểm). Với mỗi số n nguyên lớn hơn hoặc bằng 2 cố định, xét các tập n số

thực đôi một khác nhau X  x1 ; x2 ;...; xn  . Kí hiệu C X  là số các giá trị khác nhau của tổng

xi  x j ( 1  i  j  n ). Tìm giá trị lớn nhất và giá trị nhỏ nhất của C X  .

Tài liệu sưu tầm và tổng hợp bản word đầy đủ liên hệ 0393732038 TÀI LIỆU TOÁN HỌC
19
Website:tailieumontoan.com
SỞ GIÁO DỤC VÀ ĐÀO TẠO KỲ THI VÀO LỚP 10 CHUYÊN KHOA HỌC TỰ NHIÊN
HÀ NỘI NĂM HỌC 2011 – 2012
Môn: TOÁN (VÒNG 1)
ĐỀ CHÍNH THỨC Thời gian làm bài: 150 phút
(Không kể thời gian giao đề)
Đề thi gồm 01 trang

Đề số 17

Câu 1 (3,0 điểm).


1). Giải hệ phương trình

  x  1 y  x  y  3
 2

 .

 ( y  2) x 2
 y  x  1

2). Giải phương trình

3 x2  7
x  .
x 2  x  1

Câu 2. (3,0 điểm).

1). Chứng minh rằng không tồn tại các bộ ba số nguyên  x; y; z thỏa mãn đẳng thức:

x4  y 4  7 z4  5 .

2). Tìm tất cả các cặp số nguyên  x; y thỏa mãn đẳng thức
4 4
 x  1   x  1   y 3 .
  90 . Đường phân giác của góc
Câu 3. (3,0 điểm). Cho hình bình hành ABCD với BAD

BCD cắt đường tròn ngoại tiếp tam giác BCD tại O khác C . Kẻ đường thẳng d đi qua
A và vuông góc với CO . Đường thẳng d lần lượt cắt các đường thẳng CB; CD tại E; F .

1). Chứng minh rằng OBE  ODC .


2). Chứng minh rằng O là tâm đường tròn ngoại tiếp tam giác CEF .
3). Gọi giao điểm của OC và BD là I , chứng minh rằng
IB.BE.EI  ID.DF.FI .
Câu 4. (1,0 điểm). Với x; y là những số thực dương, tìm giá trị nhỏ nhất của biểu thức

x3 4y3
P  .
x3  8 y 3 y 3   x  y
3

Tài liệu sưu tầm và tổng hợp bản word đầy đủ liên hệ 0393732038 TÀI LIỆU TOÁN HỌC
20
Website:tailieumontoan.com
SỞ GIÁO DỤC VÀ ĐÀO TẠO KỲ THI VÀO LỚP 10 CHUYÊN KHOA HỌC TỰ NHIÊN
HÀ NỘI NĂM HỌC 2011 – 2012
Môn: TOÁN (VÒNG 2)
ĐỀ CHÍNH THỨC Thời gian làm bài: 150 phút
(Không kể thời gian giao đề)
Đề thi gồm 01 trang

Đề số 18

Câu 1. (3,0 điểm).


1). Giải phương trình :  x3 x  
1 x  1  1 .

 2 2 2 2
x  y  2 x y

2). Giải hệ phương trình:  .



 x  y1  xy  4 x 2 y 2

Câu 2. (3,0 điểm).


1). Với mỗi số thực a ta gọi phần nguyên của a là số nguyên lớn nhất không vượt quá a
và ký hiệu là  a . Chứng minh rằng với mọi số nguyên dương n , biểu thức
2
 1 1
n   3 n    không biểu diễn được dưới dạng lập phương của một số nguyên dương.
 27 3 

2). Với x; y; z là các số thực dương thỏa mãn đẳng thức xy  yz  zx  5 , Tìm giá trị nhỏ
3x  3 y  2 z
nhất của biểu thức : P  .
6  x  5  6  y 2  5  6  z 2  5
2

 và CDA
Câu 3. (3,0 điểm). Cho hình thang ABCD với BC song song AD . Các góc BAD  là

các góc nhọn. Hai đường chéo AC và BD cắt nhau tại I . P là điểm bất kỳ trên đoạn thẳng
BC ( P không trùng với B; C ). Giả sử đường tròn ngoại tiếp tam giác BIP cắt đoạn thẳng
PA tại M khác P và đường tròn ngoại tiếp tam giác CIP cắt đoạn thẳng PD tại N khác P
1). Chứng minh rằng năm điểm A; M ; I ; N ; D cùng nằm trên một đường tròn. Gọi đường
tròn này là ( K ) .
2). Giả sử các đường thẳng BM và CN cắt nhau tại Q , chứng minh rằng Q cũng nằm
trên đường tròn ( K ) .
PB BD
3). Trong trường hợp P; I ; Q thẳng hàng, chứng minh rằng  .
PC CA
Câu 4. (1,0 điểm). Giả sử A là một tập con của tập các số tự nhiên  . Tập A có phần tử
nhỏ nhất là 1, phần tử lớn nhất là 100 và mỗi x thuộc A ( x  1 ) luôn tồn tại a; b cũng
thuộc A sao cho x  a  b ( a có thể bằng b ). Hãy tìm một tập A có số phần tử nhỏ nhất.

Tài liệu sưu tầm và tổng hợp bản word đầy đủ liên hệ 0393732038 TÀI LIỆU TOÁN HỌC
21
Website:tailieumontoan.com
SỞ GIÁO DỤC VÀ ĐÀO TẠO KỲ THI VÀO LỚP 10 CHUYÊN KHOA HỌC TỰ NHIÊN
HÀ NỘI NĂM HỌC 2010 – 2011
Môn: TOÁN (VÒNG 1)
ĐỀ CHÍNH THỨC Thời gian làm bài: 150 phút
(Không kể thời gian giao đề)
Đề thi gồm 01 trang

Đề số 19

Câu I.
1) Giải hệ phương trình
3 x 2 + 8 y 2 + 12 xy = 23
 2
 x + y 2 = 2.
2) Giải phương trình
2 x + 1 + 3 4 x 2 − 2 x + 1 = 3 + 8 x 3 + 1.
Câu II.
1) Tìm tất cả các số nguyên không âm (x, y) thoả mãn đẳng thức
( )( )
1 + x 2 1 + y 2 + 4 xy + 2(x + y )(1 + xy ) = 25.
2) Với mỗi số thực a, ta gọi phần nguyên của số a là số nguyên lớn nhất không
vượt quá a và ký hiệu là [a]. Chứng minh rằng với mọi n nguyên dương ta luôn
có.
 3 7 n 2 + n + 1
 + + ... =n
1.2 2.3 n(n + 1) 
Câu III.
Cho đường tròn (O) với đường kính AB = 2R. Trên đường thẳng tiếp xúc với đương
tròn (O) tại A ta lấy điểm C sao cho góc ACB = 30 0 . Gọi H là giao điểm thứ hai của đường
thăng BC với đường tròn (O).
1) Tính độ dài đương thẳng AC, BC và khoảng cách từ A đến đương thẳng BC
theo R.
2) Với mỗi điểm M trên đoạn thẳng AC, đường thẳng BM cắt đường tròn (O tại
điểm N (khác B). Chứng minh rằng bốn điểm C, M, N, H nằm trên cùng một
đường tròn và tâm đường tròn đó luôn chạy trên một đường thẳng cố định khi
M thay đổi trên đoạn thẳng AC.
Câu IV.
9
Với a,b là các số thực thoả mãn đẳng thức (1 + a )(1 + b) = , hãy tìm giá trị nhỏ nhất
4
của biểu thức P = 1 + a 4 + 1 + b 4 .

Tài liệu sưu tầm và tổng hợp bản word đầy đủ liên hệ 0393732038 TÀI LIỆU TOÁN HỌC
22
Website:tailieumontoan.com
SỞ GIÁO DỤC VÀ ĐÀO TẠO KỲ THI VÀO LỚP 10 CHUYÊN KHOA HỌC TỰ NHIÊN
HÀ NỘI NĂM HỌC 2010 – 2011
Môn: TOÁN (VÒNG 2)
ĐỀ CHÍNH THỨC Thời gian làm bài: 150 phút
(Không kể thời gian giao đề)
Đề thi gồm 01 trang

Đề số 20

Câu I.

1) Giải phương trình x + 3 + 3x + 1 =4


 5x2 + 2y2 + 2xy = 26
2) Giải hệ phương trình 
(
3x + 2x + y x − y = 11.)( )
Câu II.
1) Tìm tất cả các số nguyên dương n để n2 + 391 là số chính phương.
2) Giả sử x, y, z là những số thực dương thỏa mãn điều kiện x + y + z = 1. Chứng minh

xy + z + 2x2 + 2y2
rằng: ≥ 1.
1 + xy
Câu III.
Cho tam giác ABC có ba góc nhọn và M là điểm nằm trong tam giác. Kí hiệu H là
hình chiếu của M trên cạnh BC và P, Q, E, F lần lượt là hình chiếu của H trên các đường
thẳng MB, MC, AB, AC. Giả sử bốn điểm P, Q, E, F thẳng hàng.
1) Chứng minh rằng M là trực tâm của tam giác ABC.
2) Chứng minh rằng BEFC là tứ giác nội tiếp.
Câu IV.
Trong dãy số gồm 2010 số thực khác 0 được sắp xếp theo thứ tự a1 , a 2 , a 3 ,..., a 2010 ta
đánh dấu tất cả các số dương và tất cả các số mà tổng của nó với một số liên tiếp liền ngay
sau nó là một số dương.
Chứng minh rằng nếu trong dãy số đã cho có ít nhất một số dương thì tổng của tất cả các
số được đánh dấu là một số dương.

Tài liệu sưu tầm và tổng hợp bản word đầy đủ liên hệ 0393732038 TÀI LIỆU TOÁN HỌC
23
Website:tailieumontoan.com
SỞ GIÁO DỤC VÀ ĐÀO TẠO KỲ THI VÀO LỚP 10 CHUYÊN KHOA HỌC TỰ NHIÊN
HÀ NỘI NĂM HỌC 2009 – 2010
Môn: TOÁN (VÒNG 1)
ĐỀ CHÍNH THỨC Thời gian làm bài: 150 phút
(Không kể thời gian giao đề)
Đề thi gồm 01 trang

Đề số 21

Câu I. 1) Giải phương trình

x2 − x + 2 = 2 x2 − x +1
2) Giải hệ phương trình
 x 2 − y 2 + xy = 1

3 x + y = y 2 + 3

Câu II. 1) Tìm chữ số tận cùng của số 1313 + 6 6 + 2009 2009
2) Với a, b là những chữ số thực dương, tìm giá trị nhỏ nhất của biểu thức
a+b
P=
a (4a + 5b) + b(4b + 5a )
Câu III. Cho hình thoi ABCD. Gọi H là giao điểm của hai đường chéo AC và BD. Biết
rằng bán kính đường tròn ngoại tiếp tam giác ABC bằng a và bán kính đường tròn
ngoại tiếp tam giác ABD bằng b.
AH a
1) Chứng minh rằng =
BH b
2) Tính diện tích hình thoi ABCD theo các bán kính a, b
Câu IV. Với a, b, c là những số thực dương, chứng minh rằng
a2 b2 c2 a+b+c
+ + ≥
3a 2 + 8b 2 + 14ab 3b 2 + 8c 2 + 14bc 3c 2 + 8a 2 + 14ca 5

Tài liệu sưu tầm và tổng hợp bản word đầy đủ liên hệ 0393732038 TÀI LIỆU TOÁN HỌC
24
Website:tailieumontoan.com
SỞ GIÁO DỤC VÀ ĐÀO TẠO KỲ THI VÀO LỚP 10 CHUYÊN KHOA HỌC TỰ NHIÊN
HÀ NỘI NĂM HỌC 2009 – 2010
Môn: TOÁN (VÒNG 2)
ĐỀ CHÍNH THỨC Thời gian làm bài: 150 phút
(Không kể thời gian giao đề)
Đề thi gồm 01 trang

Đề số 22

Câu I. 1) Giải phương trình 14 x + 35 + 6 x + 1 = 84 + x 2 + 36 x + 35


1 3 2n − 1 n2
2) Chứng minh rằng + + ... + =
4 + 14 4 + 3 4 4 + (2n − 1) 4 4n 2 + 1

Với mọi n nguyên dương


Câu II. 1) Tìm số nguyên dương n sao cho tất cả các số
n + 1, n + 5, n + 7, n + 13, n + 17, n + 25, n + 37 đều là nguyên tố
2) Mỗi lần cho phép thay thế cặp số (a,b) thuộc tập hợp
M = {(16,2), (4,32), (6,62), (78,8)} bằng cặp số (a + c, b + d) trong đó cặp

số (c, d) cũng thuộc M.


Hỏi sau một số hữu hạn lần thay thế ta có thể nhận được tập hợp các cặp
số M 1 = {(2018,702), (844,2104), (1056,2176), (2240,912)} hay không?

Câu III. Cho đường tròn (O) và (O’) cắt nhau tại hai điểm A và B. Trên đường thẳng AB
ta lấy một điểm M bất kỳ sao cho điểm A nằm trong đoạn BM (M ≠ A) .Từ điểm M kẻ tới
đường tròn (O’) các tiếp tuyến MC và MD (C và D là các tiếp điểm, C nằm ngoài (O)).
Đường thẳng AC cắt lần thứ hai đường tròn (O) tại điểm P và đường thẳng AD cắt lần thứ
hai đường tròn (O) tại Q. Đường thẳng CD cắt PQ tại K.
1) Chứng minh rằng hai tam giác BCD và BPQ đồng dạng
2) Chứng minh rằng khi M thay đổi thì đường tròn ngoại tiếp tam giác KCP luôn đi qua
điểm cố định.
Câu IV. Giả sử x,y,z là những số thực thoả mãn điều kiện
0 ≤ x, y, z ≤ 2 và x+ y + z = 3

Tìm giá trị nhỏ nhất và lớn nhất của biểu thức :

M = x 4 + y 4 + z 4 + 12(1 − x )(1 − y )(1 − z )

Tài liệu sưu tầm và tổng hợp bản word đầy đủ liên hệ 0393732038 TÀI LIỆU TOÁN HỌC
25
Website:tailieumontoan.com

HƯỚNG DẪN GIẢI


Đề số 1

Bài 1.

5
a, Điều kiện x ≥ − đặt a = 26 x + 5 và b = x 2 + 30 (a ≥ 0, b > 0)
26

Phương trình trở thành

a2
+ 2a = 3b ⇔ ( a − b )( a + 3b ) = 0 ⇔ a = b ⇔ 26 x + 5 = x 2 + 30
b

Vậy x = 1 hoặc 25 là nghiệm của phương trình.

b, thay 2 = x2 + y2 vào phương trình thứ 2 ta được

(x + 2y)(x2 + y2 + 3y2 + 4xy) = 27

⇔ ( x + 2 y ) = 27
3

⇔ x = 3 - 2y thay vào PT thứ nhất ta được (3 - 2y)2 + y2 = 0

7
y = 1 hoặc y =
5

1
x = 1 hoặc
5

Bài 2.

a, từ biểu thức ( x 2 − x + 1)( y 2 + xy ) = 3 x − 1 ta nhận thấy 3x - 1 phải chia hết cho (x2 – x + 1)

ta có (3x - 1)(3x - 2) = 9x2 - 9x + 2 = 9(x2 – x + 1) - 7 cũng phải chia hết cho (x2 – x + 1)

suy ra 7 chia hết cho (x2 – x + 1)

(x2 – x + 1) = 1 hoặc 7

y = 0, 1, 3 và -2 lần lượt thay vào ta có y => (x,y) = (1, 1),(1, -2) và (-2, 1)

b, Từ giả thiết xy + 2 ≥ 2y => 4xy + 8 ≥ 8y

Mà ta lại có 4x2 + y2 ≥ 4xy

⇒ 4x2 + y2 + 8 ≥ 4xy + 8 ≥ 8y

Tài liệu sưu tầm và tổng hợp bản word đầy đủ liên hệ 0393732038 TÀI LIỆU TOÁN HỌC
26
Website:tailieumontoan.com

⇒ 4(x2 + 4) ≥ 8y + 8 - y2

⇒ 4(x2 + 4) ≥ 4(y2 + 1) + (5y + 2)(2 - y) ≥ 4(y2 + 1)

x2 + 4
⇒M = ≥1
y2 + 1

Dấu “=” xẩy ra khi x = 1 và y = 2, Mmin = 1.

Bài 3.

a, Do đường tron (O) nội tiếp hình vuông ABCD nên E


và F là trung điểm các cạnh AB và AD => ∆ ABF và ∆
BCE bằng nhau => góc EBG bằng góc BCG => góc BGC
vông => AEGF cùng nằm trên một đường tròn, mà
AEOF cũng nằm trên một đường tròn => AEGOF cùng
nằm trên một đường tròn.

b, Ta có AB là tiếp tuyến của đường tròn (O) nên

góc BEM = góc EFM,

lại có góc EAG và EFG cùng chắn cung EG nên góc EAG = EFG

suy ra EM//AG trong khi E là trung điểm của AB => M cũng là trung điểm của BG
Bài 4.

3
1 1 1 2 x y z 
+ + ≥  + + 
1+ x 1+ y 1+ z
2 2 2

3 1+ x 2
1+ y 2
1 + z2 
 

Ta có:

1 +x 2 = xy + yz + xz + x2 = (x + y)(x + z)

1 + y2 = xy + yz + xz + y2 = (x + y)(y + z)

1 + z2 = xy + yz + xz + z2 = (z + y)(x + z)

2( x + y + z )
VT=
( x + y )( y + z )( z + x)

Ta có:

2
 x y z   x y z 
 + +  ≤ ( x + y + z)  + + 2 
 1 + x2 1+ y 2
1 + z2  1+ x 1+ y 1+ z 
2 2
 
Tài liệu sưu tầm và tổng hợp bản word đầy đủ liên hệ 0393732038 TÀI LIỆU TOÁN HỌC
27
Website:tailieumontoan.com
 x y z 
= ( x + y + z)  + + 
 ( x + y )( x + z ) ( x + y )( y + z ) ( z + y )( x + z ) 

2( x + y + z )
=
( x + y )( y + z )( z + x)

Do đó

4( x + y + z )  x y z 
VP=  + + 
3( x + y )( y + z )( z + x)  1 + x 2
1+ y 2
1 + z2 
 

Bất đẳng thức trở thành

x y z 3
+ + ≤
1+ x 2
1+ y 2
1+ z 2 2

Ta có:

x x 1 x x 
= ≤  + 
1 + x2 ( x + y )( x + z ) 2 x+ y x+ z 

y y 1 y y 
= ≤  + 
1 + y2 ( x + y )( y + z ) 2 x+ y y + z 

z z 1 z z 
= ≤  + 
1 + z2 ( x + z )( y + z ) 2 x+ z y + z 

x y z 3
=> + + ≤
1 + x2 1+ y2 1+ z2 2

1
Dấu “=” xảy ra khi x = y = z =
3

Đề số 2

Câu 1.

a. Ta có
 3 x 2 + y 2 + 4 xy = 8  ( x + y )( 3 x + y ) = 8
 ⇔  .
( x + y ) ( x 2
+ xy + 2 ) =8 ( x + y ) ( x 2
+ xy + 2 ) = 8

Do phương trình thứ nhất nên x + y ≠ 0 do đó ta kết hợp hai phương trình
lại ta có

Tài liệu sưu tầm và tổng hợp bản word đầy đủ liên hệ 0393732038 TÀI LIỆU TOÁN HỌC
28
Website:tailieumontoan.com
 x =1
x 2 + xy + 2 = 3 x + y ⇔ ( x − 1)( x + y − 2 ) = 0 ⇔  .
 x= 2 − y
 y =1
TH1: x =1 ⇒ 3 + y 2 + 4 y = 8 ⇔  .
 y = −5
TH2: x= 2 − y thay vào phương trình thứ nhất ta có −4 ( y − 1) = 0 ⇔ y =1 .
Vậy hệ đã cho có hai cặp nghiệm ( x; y ) là (1;1) ; (1; −5 ) .
5
b. ĐK: x ≤ ; x 2 + x ≤ 5 .
2
Đặt a = 5 − ( x 2 + x ) và b =5 − 2 x  ( a, b ≥ 0 ) . Ta có

32 − a 2 32 − b 2
= . (1)
2+a 2+b
1 1
Ta thấy, nếu a > b ≥ 0 thì 32 − a 2 < 32 − b 2 và
< tức là VT<VP,
a+2 2+b
mâu thuẫn. Tương tự với a < b cũng mẫu thuẫn. Do đó a = b , tức là phương
trình ban đầu tương đương với
( ).
 x = 1 TM
5 − ( x 2 + x ) =5 − 2 x ⇔ 
( )
 x = 0 TM
Vậy phương trình có hai nghiệm = =
x 1, 0
x .
Câu 2.

a. Trước hết ta chứng minh rằng x 7 ≡ x  ( mod 42 ) , ∀x ∈  .


(1)
Thật vậy, ta có x 7 − x= x ( x − 1)( x + 1) ( x 4 + x 2 + 1) .
Dễ thầy x ( x − 1)( x + 1) là tích 3 số nguyên liên tiếp nên nó chia hết cho 6.
Theo định lí Ơle thì x 7 − x ≡ 0 ( mod 7 ) , ∀x ∈  , tức là x 7 − x chia hết cho 7 .
Vậy x 7 − x chia hết cho BCNN ( 6;7 ) = 42 . Khẳng định (1) được chứng minh.
Từ đó
7 7 7
( 27 n + 5 )7 + 10  + (10n + 27 )7 + 5 + ( 5n + 10 )7 + 27 
     
≡ ( 27 n + 5 ) + 10 + (10n + 27 ) + 5 + ( 5n + 10 ) + 27 ( mod 42 )
7 7 7

≡ 27 n + 5 + 10 + 10n + 27 + 5 + 5n + 10 + 27 ( mod 42 )
≡ 42 ( n + 1)( mod 42 )
≡ 0 ( mod 42 ) .
Từ đó ta có khẳng định của bài toán.
( x + y)
2
a2
b. Đặt a= x + y . Sử dụng bất đẳng thức AM-GM, ta có xy ≤ =
4 4
Tài liệu sưu tầm và tổng hợp bản word đầy đủ liên hệ 0393732038 TÀI LIỆU TOÁN HỌC
29
Website:tailieumontoan.com
2
5 
Hay  a + 1 ≥ 2 .
2 

Từ đó, ta có a ≥
2
5
(
2 − 1 . Suy ra)
9
( )
2
P=
17 x 2 + 17 y 2 + 16 xy =
17 a 2 − 18 xy ≥ 17 a 2 − a 2 ≥ 2 6−4 2 .
2 −1 =
2
2 −1
Dấu “=” xảy ra khi và chỉ khi x= y= .
5
Câu 3.

a. Sử dụng định lí Talet trong tam giác LKE với JH  EK , ta có


LH LJ
= .
HK JE
Sử dụng định lí Talet trong tam giác JHE với FL  EK , ta cùng có
FL LJ
= .
EK JE
Do đó
FL LH
= .
EK HK
FL LH
Hai tam giác FLH và EKH có ∠FLH = 900 và
∠EKH = = nên
EK HK
∆FLH ∽ ∆EKH ⇒ ∠LFH = ∠KEH . Mặt khác, ta lại có ∠LFH =∠FHJ (so le
trong) và ∠KEH =
∠EHJ (so le trong). Do đó HJ là phân giác của góc EHF .
b. Go HJ  FL nên S FJL = S FLH . Suy ra S BFJL = S BFL + S FLH = S BFH . (1)
Chứng minh tương tự, ta cùng có SCEJK = SCEH .
(2)
Theo chứng minh câu a, hai tam giác FTL và EKH đồng dạng nên
∠FHB =
∠FHL = ∠EHC .
∠EHK =
Hai tam giác FHB và EHC có ∠FBH =
∠ECH và ∠FHB =
∠ECH nên đông
S FBH BF 2
dạng với nhau. Suy ra = .
S ECH CE 2
Ta kết hợp (1) và (2), ta thu được
S1 S FBH BF 2
= = .
S 2 S ECH CE 2
Điều phải chứng minh.

Tài liệu sưu tầm và tổng hợp bản word đầy đủ liên hệ 0393732038 TÀI LIỆU TOÁN HỌC
30
Website:tailieumontoan.com

c. Không mất tính tổng quát, ta có thể giả sử P nằm cùng phíc với B so với
AD như hình vẽ ở trên. Gọi M là giao điểm của PJ và EK . Áp dụng định lí
Menelaus cho tam giác KFE với cát tuyến MJP , ta có
MK PE JF
. . = 1.
ME PF JK
Mà hai tam giác  BFH và CEH đồng dạng với nhau có FL và EK là hai
JF FL BF
đường cao tương ứng nên = = .
JK EK CE
MK PE BF
Suy ra . . =1.
ME PF CE
(3)
Để chứng minh ba điểm P, , 
J D thẳng hàng, ta chỉ cần chứng minh M , , 
DJ
thẳng hàng. Theo định lí Menelaus đảo áp dụng cho tam giác LKE , điều này
tương đương với ta phải chứng minh
MK JE DL
. . = 1.
ME JL DK
JE EK CE DL DL DC DB AF AC AF
Lại có = = = và  .= . = . .1 .
JL FL BF DK DB DK DC AB AE AE
MK CE AF
Do đó, chỉ cần chứng minh . . =1
ME BF AE
(4)
CE AF PE BF
Kết hợp (3) và (4) , ta đưa bài toán về chứng minh . = . .
BF AE PF CE
AF PF BF 2
Hay . = .
AE PE CE 2
(5)

Tài liệu sưu tầm và tổng hợp bản word đầy đủ liên hệ 0393732038 TÀI LIỆU TOÁN HỌC
31
Website:tailieumontoan.com
Gọi T, N lần lượt là tiếp điểm của đường tròn (I) với AB, AC . Đặt
= = AC=
a AB = CD=
x BD
,  = TF=
y PF
, ,  = EN . Ta sẽ chứng minh
z PE
x ( x + y )( x + z )
a= (6)
x 2 − yz
Thật vậy, sử dụng định lí cosin trong các tam giác ABC và AEF , ta có
AE 2 + AF 2 − EF 2 AB 2 + AC 2 − BC 2
=2 cos A = .
AE. AF AB. AC
Suy ra
(a − x − y) + (a − x − z) − ( y + z)
2 2 2
2a 2 − 4 x 2
= ,
( a − x − y )( a − x − z ) a2
Hay
(a − x − y) + (a − x − z) − ( y + z)
2 2 2
2a 2 − 4 x 2
2− =
2− .
( a − x − y )( a − x − z ) a2
Từ đây, ta có
yz x2
= ,
a 2 − ( 2 x + y + z ) a + ( x + y )( x + z ) a 2
Hay
(x 2
− yz ) a 2 − x 2 ( 2 x + y + z ) a + x 2 ( x + y )( x + z ) =.
0

Như thế, ta có ( a − x ) ( x 2 − yz ) a − x ( x + y )( x + z )  =


0.

Do a > x nên (6) được chứng minh, Sử dụng (6) vừa chứng minh ta có
x ( x + y )( x + z )
−x− y
(=x + y)
2
AF PF a − x − y y x 2 − yz y BF 2
= . = . = . .
AE PE a − x − z z x ( x + y )( x − z ) z ( x + z)
2
CE 2
−x−z
x − yz
2

Đẳng thức (5) được chứng minh. Ta có điều phải chứng minh.
Bài 4. Đặt M n = { x|x ∈ ,x ≤ 2n − 1} . Ta chứng minh mệnh đề tổng quát: Trong 2n + 1 số
phân biệt từ tập hợp M n , luôn tồn tại ba số phân biệt có tổng bẳng 0 . Ta chứng minh bằng
phương pháp phản chứng. Giả sử tồn tại số nguyên dương n sao cho thể chọn ra 2n + 1 số
phân biệt từ tập hợp M n mà trong đó không có ba số phân biệt nào có tổng bằng 0 . Gọi n
là số nhỏ nhất có tính chất như vây. Khi đó n > 1 ( vì với n = 1 thì mệnh đề đúng). Vì n là
số nhỏ nhất làm cho mệnh đề không đúng nên mệnh đề đúng với n − 1 . Nếu trong các số
được chọn có ít nhất 2n − 1 số thuộc M n −1 thì do mệnh đề đúng với n − 1 , sẽ tồn tại ba số
phân biệt trong các số được chọn có tổng bằng 0 . Mẫu thuẫn. Vậy có tối đa 2n − 2 số được
chọn thuộc M n −1 . Suy ra trong bốn số −2n + 2, −2n + 1, 2n − 2, 2n − 1, có ít nhất ba số được
chọn. Suy ra 0 không được chọn.

Tài liệu sưu tầm và tổng hợp bản word đầy đủ liên hệ 0393732038 TÀI LIỆU TOÁN HỌC
32
Website:tailieumontoan.com
• Nếu cả hai số của cặp ( −2n + 1, 2n − 1) được chọn. Chia tập
M n \ {−2n + 1, 2n − 1, 0} thành 2n − 2 cặp
(1; 2n − 2 ) , 2;
( 2n − 3) ,…, ( −1; −2n + 2 ) ,…, ( −n + 1, −n ) ta thấy từ mỗi cặp ta chỉ
chọn được tối đa một số. Suy ra chỉ lấy được tối đa 2 + 2n − 2 = 2n số. Mẫu
thuẫn.
• Nếu chỉ có một số của cặp ( −2n + 1, 2n − 1) được chọn thì theo lí luận ở trên,
cặp ( −2n + 2, 2n − 2 ) được chọn. Không mất tính tổng quát ta giả sử 2n − 1
được chọn còn 1 − 2n không được chọn. Lúc này chia các phần tử còn lại
thành 2n − 5 cặp
(1; 2n − 3) , 2;
( 2n − 4 ) ,…, ( n − 2; n ) , (−2; −2n + 3,…, ( −n + 3; −n − 1) , một bộ ba số
( −n + 2, −n + 1, −n ) và một phần tử lẻ cặp là n − 1 . Từ mỗi cặp ta lấy được tối
đa một số, từ bộ ba số ta cũng lấy được tối đa một số. Từ đó ta lấy được
tối đa 3 + 2n − 5 + 1 + 1 =2n số. Mẫu thuẫn.
Vậy trong mọi trường hợp đều dẫn đến mẫu thuẩn, tức điều giả sử sai. Mệnh đề
được chứng minh. Áp dụng mệnh đề cho n = 1010 ta có điều phải chứng minh.

Đề số 3

Bài 1.

a) Giải phương trình x 2 − x + 2 x 3 + 1= 2 x + 1 .

Điều kiện xác định của phương trình là x ≥ −1 . Để ý rằng x 2 − x + 1 > 0 .

Đặt a = x + 1; b = x 2 − x + 1 ( a ≥ 0; b > 0 ) . Khi đó phương trình đã cho được viết lại thành

b 2 − 1 + 2ab= 2a ⇔ ( b − 1)( b + 1 + 2a )= 0

Do a ≥ 0; b > 0 nên ta có b + 1 + 2a > 0 . Khi đó từ phương trình trên ta được b = 1 .

Do đó ta có phương trình x 2 − x + 1 = 1 ⇔ x 2 − x = 0 ⇔ x ∈ {0;1} .

Kết hợp với điều kiện xác định ta được tập nghiệm của phương trình là S = {0;1} .

xy + y 2 =1 + y

b) Giải hệ phương trình  2 2
.

 x + 2y + 2xy 4
=+ x
2xy + 2y 2 =+
 2 2y
Hệ phương trình đã cho được viết lại thành  2 2
.
x + 2y + 2xy =+
 4 x

Cộng theo vế hai phương trình của hệ phương trình trên ra thu được

Tài liệu sưu tầm và tổng hợp bản word đầy đủ liên hệ 0393732038 TÀI LIỆU TOÁN HỌC
33
Website:tailieumontoan.com

x 2 + 4y 2 + 4xy =6 + x + 2y ⇔ ( x + 2y ) =x + 2y + 6 ⇔ ( x + 2y ) − ( x + 2y ) − 6 =0
2 2

 x + 2y − 3 = 0  x = 3 − 2y
⇔ ( x + 2y − 3 )( x + 2y + 2 ) =0 ⇔  ⇔
 x + 2y + 2 =0  x =−2y − 2

+ Thế x= 3 − 2y vào phương trình thứ nhất của hệ đã cho ta được

y ( 3 − 2y ) + y 2 = y + 1 ⇔ y 2 − 2y + 1 = 0 ⇔ ( y − 1) = 0 ⇔ y = 1
2

Từ đó tương ứng ta được x = 1 .

+ Thế x =
−2y − 2 vào phương trình thứ nhất của hệ đã cho ta được

 −3 − 5 −3 + 5 
y ( −2y − 2 ) + y 2 = y + 1 ⇔ y 2 + 3y + 1 = 0 ⇔ y ∈  ; 
 2 2 

−3 − 5 −3 + 5
Từ đó với y = ta được x= 1 + 5 và với y = ta được x= 1 − 5
2 2

   
(1;1) ,  1 + 5; −3 −2 5  ,  1 − 5; −3 +2 5  .
Vậy hệ phương trình có các nghiệm ( x; y ) =
   

Bài 2.

a) Tìm tất cả các cặp số nguyên ( x; y ) thỏa mãn ( x + y )( 3x + 2y ) = 2x + y − 1 .


2

Để ý rằng 2x + y= ( 3x + 2y ) − ( x + y ) nên phương trình đã cho được viết lại thành


( x + y )( 3x + 2y ) = ( 3x + 2y ) − ( x + y ) − 1
2

Đặt a =+
x y; b =
3x + 2y . Khi đó ta có ab 2 = b − a − 1 hay a b 2 + 1 =b − 1 . ( )
Từ đó suy ra b − 1 chia hết cho b 2 + 1 . Do đó ta được b 2 + 1 − ( b − 1)( b + 1) chia hết cho

b 2 + 1 hay 2 chia hết cho b 2 + 1 . Suy ra b 2 + 1 ∈ {1; 2} nên b ∈ {−1; 0;1} .

+ Với b = −1 ta được a = −1 , khi đó ta được ( x; y=


) (1; −2 ) .
+ Với b = 0 ta được a = −1 , khi đó ta được ( x; y=
) ( 2; −3 ) .
+ Với b = 1 ta được a = 0 , khi đó ta được ( x; y=
) (1; −1) .
Vậy các cặp số nguyên ( x; y ) =
(1; −2 ) , (1; −2 ) , ( 2; −3 ) thỏa mãn yêu cầu bài toán.

Tài liệu sưu tầm và tổng hợp bản word đầy đủ liên hệ 0393732038 TÀI LIỆU TOÁN HỌC
34
Website:tailieumontoan.com

b
b) Với a, b là các số thực dương thay đổi thỏa mãn điều kiện a + 2b =2 + . Tìm giá trị
3
a b
nhỏ nhất của biểu thức
= M . +
a + 2b b + 2a
+ Lời giải 1. Ta sẽ chứng minh M ≥ 2 với dấu đẳng thức xẩy ra chẳng hạn khi a= b= 3 .

Thật vậy, áp dụng bất đẳng thức AM – GM ta có

b b 3b 2b 3b b 3b
= ≥ =
b + 2a 3b ( b + 2a ) 3b + b + 2a a + 2b

a b 3b
Như vậy ta cần chỉ ra được + ≥ 2.
a + 2b a + 2b

b
Đặt x = a + 2b; y =
3
( x ≥ 0; y ≥ 0 ) . Khi đó giả thiết được viết lại thành x − y =2 .
b 3y 2 ; =
Cũng từ trên ta có = a x 2 − 6y . Bất đẳng thức cần chứng minh trên được viết lại

x 2 − 6y 2 9y 3 x 2 − 6y 2 9y 3
thành + 2 ≥2⇔ + 2 ≥ x−y.
x x x x
Biến đổi tương đương bất đẳng thức trên ta được

( )
x x 2 − 6y 2 + 9y 3 ≥ x 2 ( x − y ) ⇔ x 3 − 6xy 2 + 9y 3 ≥ x 3 − x 2 y

(
⇔ 9y 3 − 6xy 2 + x 2 y ≥ 0 ⇔ y 9y 2 − 6xy + x 2 ≥ 0 ⇔ y ( 3y − x ) ≥ 0) 2

Bất đẳng thức cuối cùng trên luôn đúng. Vậy bài toán được giải quyết hoàn toàn.

b a b b
+ Lời giải 2. Xét biểu thức M + = + + .
3 a + 2b b + 2a 3

Áp dụng bất đẳng thức Cauchy – Schwarz dạng phân thức ta có

(a + b + b)
2
b a2 b2 b2
M += + + ≥
3 a a + 2b b b + 2a b 3b a a + 2b + b b + 2a + b 3b

Áp dụng tiếp bất đẳng thức Cauchy – Schwarz ta lại có

b b + 2a + b 3b= b ( b + 2a + 3b ≤ b ) (1 + 1)( b + 2a + 3b=) 2b a + 2b

(a + b + b) ( a + 2b )
2 2
a + 2b
Từ đó ≥ = a + 2b .
=
a a + 2b + b b + 2a + b 3b a a + 2b + 2b a + 2b a + 2b

Tài liệu sưu tầm và tổng hợp bản word đầy đủ liên hệ 0393732038 TÀI LIỆU TOÁN HỌC
35
Website:tailieumontoan.com

b b
Suy ra M + ≥ a + 2b nên M ≥ a + 2b − 2 . Vậy bài toán được giải quyết hoàn
=
3 3

toàn.

Bài 3.

a) Chứng minh hai tam giác BKM và DEF đồng dạng với nhau.

Đường tròn ( I ) nội tiếp tam giác ABC nên A

ta có BD và BF là các tiếp tuyến. Do đó BI là

đường trung trực của đoạn thẳng DF nên BI


E
vuông góc với DF tại M. Từ đó BMDK nội F
I

tiếp đường tròn, do đó BMK 
= BDK .
= CDE M
N

X
Cũng do CE là tiếp tuyến với đường tròn
C
( I ) tại E nên ta có CDE
 = DFE
 . Từ đó suy ra B D

BMK  . Mặt khác BKM


 = DFE  
= BDM 
= DEF J
L
nên hai tam giác BKM và DEF đồng dạng.

b) Chứng minh hai đường thẳng MK và NL song song với nhau.


 = DBK
Ta có các tứ giác BKMD và CLDN nội tiếp đường tròn nên suy ra DMK  và

 = DLN
DCN  . Mặt khác do BK song song với CN nên ta có DBK
 = DCN
 . Từ đó suy ra

 = DLN
DMK  nên MK song song với LN.

c) Chứng minh đường thẳng JX vuông góc với đường thẳng EF.
 =DCN
Ta có DMK  =90 0 − DFE
 =90 0 − CDN  =90 0 − DMN
 , do đó KMN
 = 90 0 . Do vậy tứ

giác KMNL là hình thang vuông. Ta có J là trung điểm của KL nên J nằm trên đường trung

1
trực của đoạn thẳng MN hay JM = JN . Mặt khác XM
= XN
= ID nên suy ra X nằm trên
2
đường trung trực của MN. Do đó XJ vuông góc với MN. Trong tam giác DEF thì MN là

đường trung bình nên ta có MN song song với EF. Do đó suy ra JX vuông góc với EF

Bài 4.

Tài liệu sưu tầm và tổng hợp bản word đầy đủ liên hệ 0393732038 TÀI LIỆU TOÁN HỌC
36
Website:tailieumontoan.com
Gọi m, n theo thứ tự là số đường thẳng đi qua P và Q. Gọi S số miền được tạo thành. Do

mỗi đường thẳng chỉ đi qua điểm P hặc điểm Q nên ta có m + n =


10 . Ta xét các trường
hợp sau.

+ Trường hợp 1. Nếu m = 0 hoặc n = 0 , chẳng hạn m = 0 thì tất cả 10 đường thẳng đã cho

cùng đồng quy tại P. Khi đó dễ thấy số miền được tạo ra trên mặt phẳng là 20. Do đó ta có

S = 20 .
+ Trường hợp 2. Nếu m > 0 và n > 0 , khi đó m ≥ 1 và n ≥ 1 . Từ mặt phẳng đã cho với hai

điểm P và Q ta vẽ thêm m đường thẳng đi qua điểm P, số miền được tạo thành là 2m.

Lần lượt vẽ thêm các đường thẳng đi qua điểm Q. Khi vẽ đường thẳng đầu tiên thì

đường thẳng này cắt m đường thẳng đi qua P tại m điểm phân biệt, m điểm phân biệt này

chia đường thẳng vừa vẽ thành m + 1 phần. Nói cách khác thì đường thẳng vừa vẽ đi qua

(vì thế chia đôi) đúng m + 1 miền trong 2m miền được tạo ra. Do đó lúc này số miền được

tạo ra là 2m + ( m + 1) .

Kể từ đường thẳng thứ hai đến đường thẳng thứ n đi qua điểm Q thì mỗi đường sẽ

cắt m đường thẳng phân biệt đi qua điểm P tại m điểm phân biệt khác Q. Các điểm phân

biệt đó cùng với điểm Q chia đường thẳng vừa vẽ thành m + 2 phần. Do đó mối lần vẽ

đường thẳng thì số miền tăng thêm m + 2 . Do đó số miền được tao ra từ các đường còn lại

đi qua Q là ( n − 1)( m + 2 ) . Như vậy ta có

S= 2m + ( m + 1) + ( n − 1)( m + 2 )= mn + 2m + 2n − 1= mn + 2 ( m + n ) − 1= mn + 19

1 1
( m + n)=
2
Áp dụng bất đẳng thức AM – GM ta có mn ≤ .100= 25 .
4 4
Từ đó ta được S ≤ 25 + 19 =
44 . Dấu bằng xẩy ra khi và chỉ khi m= n= 5 .
Vậy số miền được tạo ra tối đa là 44 khi số đường thẳng đi qua P là 5 và số đường thẳng đi
qua Q là 5.

Đề số 4

Câu 1.
 xy ( x + y ) =
 2
a) Giải hệ phương trình :  3
 x + y + x y + 7 ( x + 1)( y + 1) =
3 3 3
 31
Ta có hệ phương trình:

Tài liệu sưu tầm và tổng hợp bản word đầy đủ liên hệ 0393732038 TÀI LIỆU TOÁN HỌC
37
Website:tailieumontoan.com

 xy ( x + y ) = 2
⇔
( x + y )( x − xy + y ) + ( xy ) + 7( x + y + xy + 1) =
2 2 3
31
 xy ( x + y ) = 2
⇔
( x + y ) ( x + y ) − 3 xy  + ( xy ) + 7 ( x + y ) + xy + 1 =31
 2
 3

ab = 2
Đặt a =+
x y; b =xy thì hệ trên trở thành: 
a ( a − 3b ) + b + 7 ( a + b + 1) =
2 3
31

ab = 2
⇔ 3
a − 3ab + b + 7 ( a + b + 1) =
3
31

⇒ ( a + b ) ( a + b ) − 3ab  − 3ab + 7 ( a + b + 1) =
2
31
 
⇔ ( a + b ) − 3ab(a + b) − 3ab + 7(a + b) − 24 =
3
0
⇒ ( a + b ) − 6(a + b) − 3.2 + 7 ( a + b ) − 24 =
3
0
⇔ ( a + b ) + ( a + b ) − 30 =
3
0
⇔ ( a + b ) − 27 + (a + b) =
3
3
⇔ (a + b − 3) ( a + b ) + 3(a + b) + 10  =
2
0
 
=
⇒ a + b 3 do ( (a + b)
2
+ 3(a + b) + 10 > 0 )
 a=
+b 3 =
a 2
(do a 2 =( x + y ) ≥ 4 xy =4b)
2
⇒ ⇒
= ab 2= b 1
x + y = 2
⇒ ⇒ x = y =1
 xy = 1
Vậy hệ có nghiệm duy nhất ( x; y ) = (1;1)

b) Giải phương trình: 9 + 3 x ( 3 − 2 x ) = 7 x + 5 3 − 2 x


3
Điều kiện xác định: 0 ≤ x ≤
2
Đặt a =x,b =3 − 2 x ( a, b ≥ 0 ) . Khi đó phương trình tương đương với:
9 + 3ab =7 a + 5b
 2 ⇒ 2a 2 + b 2 + 6 + 3ab = 7 a + 5b
 2a + b =
2
3
⇔ 2a 2 + 2ab − 4a + ab + b 2 − 2b − 3a − 3b + 6 = 0
⇔ 2a (a + b − 2) + b ( a + b − 2 ) − 3 ( a + b − 2 ) =0
⇔ ( a + b − 2 )( 2a + b − 3) =0

Tài liệu sưu tầm và tổng hợp bản word đầy đủ liên hệ 0393732038 TÀI LIỆU TOÁN HỌC
38
Website:tailieumontoan.com

 a + b = 2 ⇒ b = 2 − a ⇒ 9 + 3a(2 − a) = 2a + 10
⇔
 2a + b = 3 ⇒ b = 3 − 2a ⇒ 9 + 3a ( 3 − 2a ) = 7 a + 5 ( 3 − 2a )
( 3a − 1)( a − 1) =  1 1
0
 a= ⇒x= (tm)
⇔ ⇔ 3 9
( a − 1) =
2
0 
 a =1 ⇒ x =1 (tm)
1 
Vậy phương trình trên có tập nghiệm S =  ;1
9 
Câu 2.
a) Cho x,y…..
(x 2
− 2 xy − y ) + ( xy − 2 y 2 − x ) =x 2 − xy − 2 y 2 − x
Ta có:
= x 2 + xy − ( 2 xy + y 2 ) − ( x + y ) = ( x + y )( x − 2 y − 1)
Lại có: x 2 − 2 xy − y, xy − 2 y 2 − x chia hết cho 5
⇒ ( x + y )( x − 2 y − 1) chia hết cho 5
TH1: Nếu x + y chia hết cho5 thì y ≡ − x ( mod 5 )
⇒ 0 ≡ x 2 − 2 xy − y ≡ x 2 + 2 x 2 + x= x ( 3 x + 1) (mod 5) , do vậy x chia hết cho 5 hoặc chia 5
dư 3.
+)Nếu x chia hết cho 5 thì y cũng vậy, bài toán được chứng minh
+)Nếu x chia cho 5 dư 3 thì y chia 5 dư 2, thì
2 x 2 + y 2 + 2 x + y ≡ 2.9 + 4 + 2.3 = 30 ≡ 0(mod 5)
Ta cũng có điều phải chứng minh.
TH2) Nếu x − 2 y − 1 chia hết cho 5 thì x ≡ 2 y + 1( mod 5 )

⇒ 0 ≡ x 2 − 2 xy − y ≡ ( 2 y + 1) − 2 y ( y + 1) − y = y + 1( mod 5 )
2

Do đó y chia 5 dư 4 và x cũng chia 5 dư 4 nên:


2 x 2 + y 2 + 2 x + y = 2.16 + 16 + 2.4 + 4 = 60 ≡ 0 ( mod 5 )
Vậy ta có điều phải chứng minh.
b) Cho………
Nếu tồn tại n :1 ≤ n ≤ 50 : a1 + a2 + ...... + an =
50 thì kết luận bài toán hiểu nhiên
a1 + a2 + ...... + an ≤ 49
Xét: 1 ≤ n ≤ 49 :  ⇒ an +1 ≥ 2
a1 + a2 + ....... + an +1 ≥ 51
TH 1: an+1 =2 ⇒ a1 + a2 + ..... + an =49
an+ 2 + an+3 + ...... + a50 =
49
Nên nếu n ≤ 24 ⇒ a1 ≤ an + 2 ; a2 ≤ an +3 ;.....; an ≤ a2 n +1
⇒ 49 =a1 + a2 + ..... + an ≤ an + 2 + an +3 + ..... + a2 n +1 < an + 2 + ...... + a49 + a50
Điều này vô lý nên:

Tài liệu sưu tầm và tổng hợp bản word đầy đủ liên hệ 0393732038 TÀI LIỆU TOÁN HỌC
39
Website:tailieumontoan.com
n ≥ 25 ⇒ 49 = a1 + a2 + .... + an ≥ na1 ≥ 25a1 ⇒ a < 2 ⇒ a = 1
⇒ a2 + .....
= an 48; a2 + ...... + =
an +1 50
TH2: an +1 ≥ 3

an + 2 + an +3 + ..... + a50 = 100 − ( a1 + a2 + ..... + an +1 ) ≤ 49


⇒ 49 ≥ ( 49 − n ) an + 2 ≥ ( 49 − n ) .3 ⇒ n ≥ 33
⇒ 49 ≥ ( a1 + a2 + .... + a16 ) + ( a17 + ..... + an ) ≥ 16 + ( n − 16 ) a17 ≥ 16 + 17 a17
⇒ a17 < 2 ⇒ a17 =1 ⇒ a1 = a2 ..... = a17 =1
Nếu an +1 < 18 đặt a1 + a2 + ..... + an +1 = 50 + k ( k ≥ 1)
⇒ 18 ≥ an +1 ≥ ( 50 + k ) − 49 =k + 1
⇒ k ≤ 17 ⇒ ak +1 + ..... + an +1 =
50
Nếu an +1 ≥ 19
⇒ 49 ≥ ( 49 − n ) an + 2 ≥ ( 49 − n )19 → n ≥ 47
⇒ a1 = a2 = ..... = a45 = 1
Vì nếu
a45 ≥ 2 ⇒ ( a1 + a2 + ....... + a44 ) + ( a45 + ....... + an ) ≥ 44 + ( n − 44 ) a45 ≥ 44 + ( 47 − 44 ) .2 > 49
Đặt an +1 = 50 − k ( 0 ≤ k ≤ 31) ⇒ a1 + ..... + ak + an +1 = 50 ( do a1 = ...... = ak = 1)
Vậy ta có điều phải chứng minh.

Câu 3:

a) Chứng minh 3 điểm K, M, Q thẳng hàng


 
= CBQ
Do các tứ giác BCKQ và BCDA nội tiếp nên: CKQ  ⇒ KQ / / AD. Mặt khác
= CAD
MK / / AD nên K, M, Q thẳng hàng
b) Đường tròn ngoại tiếp tam giác………….
Chứng minh tương tự ta có: R, M , L thẳng hàng
Tài liệu sưu tầm và tổng hợp bản word đầy đủ liên hệ 0393732038 TÀI LIỆU TOÁN HỌC
40
Website:tailieumontoan.com
 
= RLD
MQ / / AD nên RMQ  ⇒ tứ giác RTMQ nội tiếp
= ETD
Chứng minh tương tự RMSQ nội tiếp do đó: M , S , Q, R, T cùng thuộc một đường tròn
c) Chứng minh đường tròn ngoại tiếp..
Bổ đề: cho tam giác ABC, M nằm trên d / / BC lấy E khác M trên d, AM cắt BC
tại I. Đường qua M / / AB cắt BE tại J , khi đó IJ / / AE
MA AG
Chứng minh MJ cắt AE, AC tại S và T, ME cắt AC tại G. Ta có MG//BC suy ra = ,
MI GC
MS AP AG MA
ME cắt AB tại P ta có: = = = ⇒ AE / / IJ
MJ PB GC MI
Quay trở lại bài toán:
AM cắt BC, (O) tại I và J khác A. Áp dụng bổ đề ta có: IR / / AE , IQ / / AB . Do đó

= 
IRE = 
AEC AJC ⇒ nên RIJC là tứ giác nội tiếp . Chứng minh tương tự ta có DQIJ là
tứ giác nội tiếp
 + IJQ
Do đó: RJI  + RPD
= 2 PCD
 + CPD
= 1800 nên RPQJ nội tiếp. Kẻ tiếp tuyến Jx của
(O).
Ta có:
 = xJA
xJR  − RJA
 =  =
ADJ − PDC 
ADP + MAC
= =
ADP + PAD APB
 = MAC
⇒ PEJ  = PED 
Suy ra : Jx tiếp xúc với ( PQR ) hay ta thu được: ( PQR ) tiếp xúc với ( O )
Vậy ta có điều phải chứng minh
Câu 4: Áp dụng BĐT Cauchy-Schwarz ta có:

 ab bc   1 1   ab bc   1 1 
 +  +  ≤ 2.  + . 2  + 
 a+b b + c  a + b b+c  a+b b+c a+b b+c
 a c  b b   a c   b b 
= 2  +  +  ≤ + + + 
 a + b b + c  a + b b + c   a + b b + c   a + b b + c 
 a b   c b 
= + + + = 2
a+b a+b b+c b+c
Vậy ta có điều phải chứng minh
Dấu " = " xảy ra khi và chỉ khi a= b= c

Tài liệu sưu tầm và tổng hợp bản word đầy đủ liên hệ 0393732038 TÀI LIỆU TOÁN HỌC
41
Website:tailieumontoan.com

Đề số 5

Câu 1 (3.5 điểm).


x 2  y 2  xy  1

a) Giải hệ phương trình 
x  x 2y  2y 3


Phương trình thứ nhất của hệ được viết lại thành xy  x 2  y 2  1 . Thế vào phương trình

thứ hai của hệ ta được

 
x  x x 2  y 2  1  2y 3  x  x 3  xy 2  x  2y 3  x 3  xy 2  2y 3  0
3 3 2 3
 
 x  y  xy  y  0  x  y  x 2  xy  y 2  y 2 x  y   0
x  y  0 x  y
2
 2

 x  y  x  xy  2y  0   2
x  xy  2y 2
 0
 
x  y  0
 

Dễ thấy x  y  0 không thỏa mãn phương trình thứ nhất của hệ.

Do đó từ x  y thay vào phương trình thứ nhất của hệ ta được

x 2  1  x  1  x  y  1 .

Vậy hệ phương trình đã cho có các nghiệm là x ; y   1;1, 1; 1 .

b) Giải phương trình 2 x  1 x  1   


x  1  1 x 2  1 x2 
Điều kiện xác định của phương trình là 1  x  1 . Biến đổi tương đương phương trình

ta được

2 x  1 x  1   
x  1  1 x 2  1 x2 
 2 x  1 x  1  2 x  1  2 1  x  x  1 1  x   1  x  1  x 
2 2

 2 x  1 x  1  2 x  1  2 1  x  x  1 x  1  1  x  x  1
 x  1  
x 1  1x  2 1x

Đặt a  x  1;b  1  x a  0;b  0 , khi đó ta có a 2  b 2  2 .

Phương trình trên được viết lại thành a 2 a  b   2b . Từ đó ta có hẹ phương trình

a 2  b 2  2

 2 .
a a  b   2b

+ Xét trường hợp b  0 , hệ phương trình trên vô nghiệm.

Tài liệu sưu tầm và tổng hợp bản word đầy đủ liên hệ 0393732038 TÀI LIỆU TOÁN HỌC
42
Website:tailieumontoan.com
+ Xét trường hợp b  0 , khi đó phương trình thứ nhất của hệ tương đương với

 
b a 2  b 2  2b .

Khi đó ta có hệ hương trình  2


 
b a 2  b 2  2b
 a 2b  b 3  2b


 3 .



a a  b   2b 



a  a 2
b  2b

Từ đó ta được a 2b  b 3  a 3  a 2b  2b  a 3  b 3  a  b .

Thay vào phương trình thứ nhất của hệ và chú ý đến điều kiện ta được

a2  1  a  1  a  b  1 .

Từ đó ta được x  1  1  x  1  x  0 , thỏa mãn điều kiện xác định.

Vậy phương trình có nghiệm duy nhất là x  0 .

Câu 2(2.5 điểm).

a) Chứng minh rằng không tồn tại các số nguyên x, y thỏa mãn đẳng thức

12x 2  26xy  15y 2  4617

Trước hết ta chứng minh bổ đề: Với mọi số nguyên tố có dạng p  4k  3 thì ta luôn có

a  p
a 2  b 2  p   a,b  Z 
b  p

Thật vậy, ta xét hai trường hợp sau

+ Trường hợp 1. Nếu một trong hai số a và b chia hết cho p thì ta suy ra điều cần chứng

minh.

+ Trường hợp 2. Nếu cả hai số a và b cùng khoog chia hết cho p. Khi đó ta có

a; p   b; p   1 .
a p1  1 mod p  a 4k 2  1 mod p 
 
Theo định lí Fecmat ta có  p1   4k 2  a 4k 2  b 4k 2  2 mod p 
b  1 mod p  b  1 mod p 
 

Mặt khác ta có a 4k 2  b 4k 2  a 2   
k 1 2k 1
 b2 chia hết cho a 2  b 2 nên chia hết cho p.

Từ đó suy ra 2 chia hết cho p, mà p là số nguyên tố nên ta được p  2 . Điều này mâu

thuẫn vì p là số nguyên tố lẻ.

Như vậy trường hợp 2 không xẩy ra hay bổ đề được chứng minh.

Trở lại bài toán. Do 4617 chia hết cho 19 nên 12x 2  26xy  15y 2 19 hay ta được

Tài liệu sưu tầm và tổng hợp bản word đầy đủ liên hệ 0393732038 TÀI LIỆU TOÁN HỌC
43
Website:tailieumontoan.com

12x 2  12xy  15y 2  38xy 19  12x 2  12xy  15y 2 19


 
 3 4x 2  4xy  5y 2 19  4x 2  4xy  5y 2 19

 
 4x 2  4xy  y 2  4y 2 19  2x  y   2y  19
2 2

Do 19 là số nguyên tố có dạng 4k  3 nên áp dụng bổ đề trên ta suy ra được


2x  y 19 3x  2y 19 3x 19 x 19
      
   
2y 19 2y 19 2y 19 y 19
   

Từ đó ta được 4x 2  4xy  5y 2 192 . Điều này dẫn đến mâu thuẫn vì 4617 không chia hết

cho 192 .

Vậy không tồn tại cặp số nguyên x ; y  thỏa mãn yêu cầu bài toán.

b) Với a, b là các số thực dương. Tìm giá trị lớn nhát của biểu thức
 1 1  1
M  a  b  3  3 
a  b b  a  ab

Áp dụng bất đẳng thức Bunhiacopxki ta có


1   
a  
 b   b   a  b  ; b 3  a b1  a   a  b 
2 2
3

a 

1 1
b a
a b a b
Từ đó ta được 3 a ; 3 b . Do đó suy ra
a b a b b a a b

1 1 1 1
 1  b a a b  
1 a b a b
a  b a 3  b  a  b 3   a 3  b  b 3  a  aa  b  ba  b  a ab b
 

1 1
a b  1  ab a  b   a  b  a  b   ab a  b   1
a b  
Suy ra M 
a b ab a  b ab ab a  b 

Vậy giá trị lớn nhất của M là 1, đạt được tại a  b  1 .

Câu 3(3.0 điểm).


  900 . Đường tròn tâm I nội tiếp tam giác ABD tiếp
Cho hình thoi ABCD có BAD

xúc với BD và BA lần lượt tại J và L. Trên đường thẳng IJ lấy điểm K sao cho BK song song

ID.

Tài liệu sưu tầm và tổng hợp bản word đầy đủ liên hệ 0393732038 TÀI LIỆU TOÁN HỌC
44
Website:tailieumontoan.com
  ABI
a) Chứng minh rằng CBK .
B

Ta có L

  CBD
CBK   KBD
; ABI
  ABD
  IBD A J
C
I
  CBD
. Lại có ABD  . Mặt khác do ID
K
song song với BK nên ta có D
  IDB
IBD   DBK
 . Từ đó suy ra

  ABI
CBK 

b) Chứng minh rằng KC  KB .


  LJI
Dễ thấy KJC  . Lại có IJL
  IBJ  . Kết hợp với CBK
  LBI   ABI
 ta được

  CJK
CBK  nên tứ giác BCKJ nội tiếp đường tròn. Do đó BJC   900 hay
  BKC

KC  KB .

c) Chứng minh rằng bốn điểm C, K, I ,L cùng nằm trên một đường tròn.
  IJ
Tam giác IJL cân tại I nên ta có ILJ  L  IBJ   JBK
 . Mà ta có IBJ  nên ILJ
  JBK

  JCK
Mặt khác do tứ giác BCKJ nội tiếp đường tròn nên JBK .

  JBK
Từ đó ta được ILJ   JCK
 nên suy ra tứ giác CKIL nội tiếp đường tròn.

Câu 4. (1.0 điểm)

Trước hết ta chứng minh bổ đề: Với hợp số n  4 ta luôn có n  1 ! n .

Thật vậy, do n là hợp số và n  4 nên ta viết được n  a.b trong đó a, b  N ;1  a, b  n .

Khi đó ta suy ra được 2  a, b  n  1 . Từ đó dễ thấy ta luôn có n  1 ! n .


Trở lại bài toán. Ta thấy an  n vì nếu an  n thì ai  n i  1; n  1 . 
a .a ...a  n
Khi đó ta có  1 2 i , điều này mâu thuẫn với giả thiết của bài toán.
a1.a2 ...an  n

Do vậy an  n . Giả sử n là hợp số và n  4 , khi đó theo bổ đề ta có

a1.a2 ...an 1  n  1 ! n .

Mặt khác theo bài ra ta lại có a1.a2 ...an  n . Như vậy a1.a2 ...an 1 và a1.a2 ...an  n có cùng số dư

khi chia cho n. Điều này mâu thuẫn với giả thiết của bài toán.
Tài liệu sưu tầm và tổng hợp bản word đầy đủ liên hệ 0393732038 TÀI LIỆU TOÁN HỌC
45
Website:tailieumontoan.com
Từ đó suy ra n  4 . Mà do n là hợp số nên ta được n  4 .

Ta thấy với n  4 thì bộ số 1; 3;2; 4 viết được dãy số 1;1.3;1.3.2;1.3.2.4 chia cho 4 có số dư

lần lượt là 1; 3;2; 0 .

Đề số 6

Câu 1.

x  y   x  3y
2

1) Hệ phương trình tương đương với 


 x  y 2  3  xy
 
x  3
Do đó ta có phương trình x  3y  3  xy  x  3y  1  0  
y  1

x  3
 x  3

+ Với 
 2  
 2 , hệ phương trình vô nghiệm.


2
x  y  xy  3 
y  3y  6  0

 

y  1 y  1 x  1; y  1
+ Với  2    
x  2; y  1 .
x  y 2  xy  3 x 2  x  2  0
  

Vậy hệ phương trình đã cho có nghiệm là x ; y   1;1 .

2. Với a, b là các số thực dương thỏa mãn ab  a  b  1 . Chứng minh rằng:

a b 1  ab
 
  
2
1a 1  b2 2 1  a 2 1  b2

Cách 1. Do ab  a  b  1 nên ta được

a 2  1  a 2  ab  a  b  a  b a  1;b 2  1  b 2  ab  a  b  a  b b  1

Đẳng thức cần chứng minh tương đương với

a b 1  ab
 
a  b a  1 a  b b  1 2 a  b  a  1b  1 2

a b  1  b a  1 1  ab
 
a  b a  1b  1 2 a  b  a  1b  1
2

 a  1b  1  2 a  1b  1  a  1b  1  2  ab  a  b  1

Do đẳng thức cuối cùng luôn đúng nên đẳng thức cần chứng minh đúng.

Tài liệu sưu tầm và tổng hợp bản word đầy đủ liên hệ 0393732038 TÀI LIỆU TOÁN HỌC
46
Website:tailieumontoan.com
Cách 2. Đẳng thức cần chứng minh tương đương với

 
a b2  1  b a 2  1   1  ab
.
 
a 2  1 b2  1   2
2 a 1 b 1  2

   
Mà ta có a b 2  1  b a 2  1  a  b ab  1 nên đẳng thức trên tương đương với

a b 1
  
 2 a  b   a 2  1 b 2  1  a 2  b 2  4ab  a 2b 2  1
2

a 2

 1 b2  1  2

 a  b   ab  1  a  b  ab  1  ab  a  b  1
2 2

Do đẳng thức cuối cùng luôn đúng nên đẳng thức cần chứng minh đúng.

Câu 2.

1. Câu 2 (2.5 điểm).

1. Giả sử p và q là các số nguyên tố thỏa mãn đẳng thức p p  1  q q 2  1 .  


a) Chứng minh rằng tồn tại số nguyên dương k sao cho p  1  kq, q 2  1  kp .

p  q  0
Nếu p  q thì ta có p  1  q 2  1   , điều này vô lí vì p, q là các số nguyên tố.
 p  q  1

Do vậy p  q , khi đó do p và q là các số nguyên tố nên p  1q và q 2  1 p .

Như vậy tồn tại các số nguyên dương m, n thỏa mãn p  1  mq; q 2  1  np , thay vào


p  1  kq
đẳng thức đã cho ta được m  n . Do vậy tồn tại số nguyên dương k sao cho 
 2 .

q  1  kp

b) Tìm tất cả các số nguyên tố p, q thỏa mãn đẳng thức p p  1  q q 2  1 .  


Thế p  kq  1 vào hệ thức q 2  1  kp ta được q 2  1  k kq  1  q 2  k 2q  k  1  0 .

Xem phương trình là phương trình bậc hai ẩn q, khi đó để phương trình có nghiệm

nguyên dương thì

  k 4  4 k  1  k 4  4k  4 phải là số chính phương.

Ta có k 4  k 4  4k  4  k 2  2 nên ta được   k 2  1 .
2 2

Từ đó ta được k 4  4k  4  k 2  1  k  k 2  k  1 .
2

Thay vào hệ thức đã cho ta được q 2  q  2  0  q  2  p  3 .

Tài liệu sưu tầm và tổng hợp bản word đầy đủ liên hệ 0393732038 TÀI LIỆU TOÁN HỌC
47
Website:tailieumontoan.com
Vậy các số p  3; q  2 là các số nguyên tố cần tìm.

2. Với a, b, c là các số thực dương thỏa mãn ab  bc  ca  abc  2 . Tìm giá trị lớn nhất

của biểu thức

a 1 b 1 c 1
M  2
 2  2
a  2a  2 b  2b  2 c  2c  2

Biến đổi giả thiết ab  bc  ca  abc  2 ta được

1  a 1  b 1  c   1  a   1  b   1  c 
1 1 1
   1
1  a 1  b  1  a 1  c  1  b 1  c 
1 1 1
Đặt x  ;y  ;z  , khi đó ta thu được xy  yz  zx  1 .
1a 1 b 1c

Biểu thức M được viết lại thành

1 1 1
a 1 b 1 c 1 x y z
M      
a  1 b  1 c  1
2 2 2
1 1 1 1 1 1
1 1 1
x2 y2 z2
x y z
  2
2
 2
x 1 y 1 z 1

Để ý ta có x 2  1  x 2  xy  yz  zx  x  y x  z  . Áp dụng tương tự ta được

x y z
M   
x  y x  z  y  z x  y  x  z y  z 
x y  z   y z  x   z x  y  2 xy  yz  zx  2
  
x  y y  z z  x  x  y y  z z  x  x  y y  z z  x 
Ta chứng minh được 9 x  y y  z z  x   8 x  y  z xy  yz  zx  .

Vì x  y  z   3 xy  yz  zx   3 nên x  y  z  3 . Nên ta được


2

2 9 9 3 3
M   
8 4 x  y  z  4 3 4
9
x  y  z xy  yz  zx 
3 3 1
Vậy giá trị lớn nhất của M là , đạ được tại x  y  z   a  b  c  3  1.
4 3

Câu 3 (3.0 điểm).

Tài liệu sưu tầm và tổng hợp bản word đầy đủ liên hệ 0393732038 TÀI LIỆU TOÁN HỌC
48
Website:tailieumontoan.com
Cho tam giác ABC nhọn với AB  AC . Gọi E, F lần lượt là trung điểm của CA, AB.
 và nằm ngoài tam
Đường trung trực của EF cắt BC tại D. Giả sử P nằm trong góc EAF
 D
giác AEF sao cho PEC    DFE
EF và PEB  . Đường thẳng PA cắt đường tròn ngoại

tiếp tam giác PEF tại Q khác P.

A
I E
F
R
M X

P Y
F E
B C
D Z

N K
P

Hình 1 B
D
C

Hình 2
  BAC
a) Chứng minh rằng EQF   EDF
.

Vì tứ giác PEQF nội tiếp đường tròn nên ta có


  1800  EPF
EQF  P   DEC
EF  PFE   DFB

 EAD 
  EDA
  FAD
  FDA
   EDF
  BAC 

b) Tiếp tuyến tại P của đường tròn ngoại tiếp tam giác PEF cắt CA, AB lần lượt tại M, N.

Chứng minh rằng bốn điểm C, M, B, N cùng nằ trên một đường tròn. Gọi đường tròn này

là đường tròn K  .

Không mất tính tổng quát ta giả sử M nằm giữa A, C và N nằm trên tia đối của tia BA (các

trường hợp còn lại chứng minh tương tự). Khi đó ta có


  1800  NPF
MNB   PFN
  1800  P  
EF  DFE
 D
 1800  DEC  
EF  1800  C   ACB
EF  AEF 

Do đó tứ giác NCMB nội tiếp đường tròn K  .

Tài liệu sưu tầm và tổng hợp bản word đầy đủ liên hệ 0393732038 TÀI LIỆU TOÁN HỌC
49
Website:tailieumontoan.com

c) Chứng minh rằng đường tròn K  tiếp xúc với đường tròn ngại tiếp tam giác AEF.

  DAC
Ta có nhận xét: PAB  . Thật vậy, gọi X, Y, Z lần lượt là điểm đối xứng với P qua

 D
EF, AE, AF thì từ giả thiết PEC   D
EF suy ra DEY 
EX , mà ta có EX  EY nên

DX  DY . Tương tự thì ta có DX  DZ nên DX  DY  XZ , lạ có AY  AP  AZ ta


  DAY
được DAZ  . Kết hợp tính đối xứng ta được PAB
  DAC
.

Đường tròn ngoại tiếp tam giác PEM cắt đường tròn ngoại tiếp tam giác AEF tại R khác E.
  
Ta thấy RPN  R EM  RFA nên tứ giác PRFN nội tiếp đường tròn. Lại do tứ giác
  180
  1800  NBC 0  .
BNCM nội tiếp đường tròn nên BAC  CMN  AMN
    
Từ đây ta thu được A R E  AFE  ABC  AMN  1800  PRE nên ba điểm A, R, P

thẳng hàng.

Gọi giao điểm của EF và AD là I, theo tính chất đường trung bình thì I là trung điểm của

AD.
  AR
Ta lại có AEI  F , kết hợp với nhận xét ta được AEI ∽ ARF .

Từ đó ra suy ra được AED ∽ ARB . Ta thu được


  ADE
ABR   DEC
  DAE
 P  P
EF  PAB   P
EF  FER  
ER  RMP
Từ đó tứ giác NMRB nội tiếp đường tròn hay năm điểm M, N, R, B, C cùng nằm trên

đường tròn K  .


Lại có ERM  EPM 
  EF 
P  EF   RAE
R  RFP .
  RNM

Vậy đường tròn ngoại tiếp tam giác REF và đường tròn K  tiếp xúc nhau tại R.

Câu 4 (1.0 điểm).

Cho n là số nguyên dương với n  5 . Xét đa giác lồi n cạnh. Người ta muốn kẻ một

số đường chéo của đa giác mà các đường chéo này chia đa giác thành đúng k miền, mõi

miền là mọt ngũ giác lồi (hai miền bất kì không có điểm chung trong).

a) Chứng minh rằng ta có thể thực hiện được với n  2018, k  672 .

Kí hiệu đa giác 2018 cạnh là A1A2A3 ...A2018 , kẻ các đường chéo A1A5 ; A1A8 ; A1A11;...; A1A2015 khi

đó đa giác A1A2A3 ...A2018 được chia thành 672 ngũ giác lồi gồm:

A1A2A3A4A5 ; A1A5A6A7A8 ;...; A1A2012A2013A2014A2015 ; A1A2015A2016A2017A2018


Tài liệu sưu tầm và tổng hợp bản word đầy đủ liên hệ 0393732038 TÀI LIỆU TOÁN HỌC
50
Website:tailieumontoan.com
b) Với n  2017, k  672 ta có thể thực hiện được không? Hãy giải thích.

Giả sử ta có thể chia đa giác lồi 2017 cạnh thành 672 ngũ giác lồi bằng các đường chéo của

nó. Gọi p là số giao điểm của các đường chéo nằm trong đa giác. Do mỗi đỉnh của ngũ giác

lồi là là đỉnh của đa giác đã cho hoặc là một trong p giao điểm của các đường chéo nên

tổng số góc các ngũ giác này là

p.3600  2017  21800  2p  20151800

Mặt khác số ngũ giác lồi là 672, mỗi ngũ giác lồi có tổng số góc ở đỉnh là 3.1800 nên tổng

số góc của các ngũ giác lồi là 672.3.1800 .

1
Từ đó ta được 2p  20151800  672.3.1800  p  , vô lý.
2
Vậy ta không thể thực hiện được với n  2017, k  672 .

Đề số 7

Câu 1(3.5 điểm).

x  y  xy x  y   4

 3 3

a) Giải hệ phương trình 





 
xy  1 x 2  y 2  4 
 Phân tích và lời giải. Hệ phương trình đã cho là hệ phương trình đối xứng dạng 1, do

đó ta có thể sử dụng phép đặt ẩn phụ S  x  y; P  xy . Tuy nhiên để đơn giản hóa ta cần

biến đổi hệ phương trình trước.Biến đổi tương đương hệ phương trình ta được


   3xy x  y   xy x  y   4
3
x 3  y 3  xy x  y   4  x  y
 

  

xy  1 x 2  y 2  4 xy  1 x  y 2  2xy   4

 

  

 
x  y   2xy x  y   4
3

x  y   2xy x  y   4
3

 
   
xy  1 x  y   2xy   4
2
 
    2xy xy  1  4
2

 
 xy  1 x  y

  

Đến đây ta có thể sử dụng phép đặt ẩn phụ, tuy nhiên để ý đến vế phải hai phương trình

ta lấy hiệu theo vế của hai phương trình của hệ thì được

Tài liệu sưu tầm và tổng hợp bản word đầy đủ liên hệ 0393732038 TÀI LIỆU TOÁN HỌC
51
Website:tailieumontoan.com

x  y   2xy x  y   xy  1x  y   2xy xy  1


3 2

 x  y  x  y  xy  1  2xy x  y  xy  1  0
2

 
 x  y  xy  1 x  y   2xy   0
2

 
x  1

x  1y  1  0 
  
x  y  xy  1  0 y  1 x  1; y  1
  x 2  y 2  0   x  y  0
x  y   2xy  0
2

   
x  y  0

Thay vào hệ phương trình đã cho ta được x ; y   1;1 là nghiệm của hệ.

8x  3
b) Giải phương trình 7x  2  5  x  .
5

 Phân tích. Để ý rằng 7x  2  5  x   8x  3 , khi đó ta đặt 7x  2  a; 5  x  b

a 2  b2  a  b 
thì phương trình đã cho được viết lại thành a  b   a  b 1   0.
5  5 

2
 Lời giải. Điều kiện xác định của phương trình là  x  5.
7

Đặt 7x  2  a; 5  x  b a  0;b  0 . Khi đó ta có a 2  b 2  8x  3 .

Như vậy phương trình đã cho được viết lại thành

a 2  b2  a  b  a  b
a b   a  b 1  0  
5  5  a  b  5


3
+ Với a  b ta có phương trình 7x  2  5  x  x 
8

+ Với a  b  5 ta có phương trình 7x  2  5  x  5 hay ta được

7x  2  5  x  2 7x  2 5  x  25  33x  7x 2  10  9  3x
9  3x  0 

  x  3
2   x 1
33x  7x 2  10  9  3x  16x 2  87x  71  0
 

 3 
Kết hợp với điều kiện xác định ta được tập nghiệm của phương trình là S   ;1
 8 

Câu 2(2.5 điểm)

a) Tìm tất cả các giá trị của m sao cho tồn tại cặp số nguyên x ; y  thỏa mãn hệ phương

trình

Tài liệu sưu tầm và tổng hợp bản word đầy đủ liên hệ 0393732038 TÀI LIỆU TOÁN HỌC
52
Website:tailieumontoan.com
2  mxy 2  3m


 
2  m x 2  y 2  6m



Lấy hiệu theo vế hai phương trình của hệ ta được m xy 2  x 2  y 2  3m . 
+ Nếu m  0 , ta thấy hệ phương trình vô nghiệm.

+ Nếu m  0 , khi đó từ phương trình ta được


xy 2  x 2  y 2  3  y 2 x  1  x 2  1  2 
  
 y  x  1 x  1  2  y  x  1 1  x   2
2 2

Do x ; y  nhận xác giá trị nguyên nên ta xét các trường hợp sau

1  x  1 x  0
+ Trường hợp 1. Với  2   2 , hệ phương trình không có nghiệm
y  x  1  2 y  3
 
nguyên.
1  x  1 x  2 x  2
+ Trường hợp . Với  2   2  
y  x  1  2 y  1 y  1
  
1  x  2 x  1 x  1
+ Trường hợp 1. Với  2   2  
y  x  1  1 y  1 y  1
  
1  x  2 x  3
+ Trường hợp 1. Với  2   2 , hệ phương trình không có nghiệm
y  x  1  1 y  3
 
nguyên.
Vậy phương trình trên có các nghiệm nguyên là x ; y   1; 1, 1;1, 2; 1, 2;1 .

1
Thay các nghiệm trên vào hệ phương trình đã cho ta tìm được m  ; m  2 thỏa mãn.
2
 1 
Vậy m   ;2 thỏa mãn yêu cầu bài toán.
 2 

b) Với x, y là những số thực thỏa mãn các điều kiện 0  x  y  2;2x  y  2xy . Tìm giá trị

 
lớn nhất của biểu thức P  x 2 x 2  1  y2 y2  1 .  
1 2 1
  2 . Áp dụng bất đẳng thức a 2  b 2  a  b 
2
Từ giả thiết 2x  y  2xy ta được
x y 2
ta có
2
1 4 1  1 2  1 4 1 4
      2  2  2  2  2  2
x 2
y 2
2 x y   x y x y

Tài liệu sưu tầm và tổng hợp bản word đầy đủ liên hệ 0393732038 TÀI LIỆU TOÁN HỌC
53
Website:tailieumontoan.com
2
1 16 1  1 4 1 16
Hoàn toàn tương tự ta được 4  4   2  2   2  4  2  4 .
x y 2 x y  x y
 x 
2
4x 2
Do 0  x  y  2 nên ta có 1  2  y 2  4  0  y 2  4  x 2  2 .
 y  y
1 4
Từ đó kết hợp với 2
 2  2 ta được
x y
4x 2 4x 2 
2 4  1
y2  4  x 2   x 2
 y 2
 4  2 x 2
  4  x 2  2
  4  x 2. 2  5
y 2
y 2 
 y   x
 x4  16x 4
Hoàn toàn tương tự ta cũng có 1  4  y 4  16  0  y 4  16  x 4  4 .
 y  y

1 16
Từ đó kết hợp với 4
 2  4 ta được
x y
4x 4 16x 4 
4 16  1
y 4  16  x 4   x 4
 y 4
 16  2x 4
  16  x 2  4
  16  x 4 . 4  17
y 4
y 4 
 y  x
   
Do vậy P  x 2 x 2  1  y2 y2  1  x 4  y 4  x 2  y 2  17  5  22 . Dấu bằng xẩy ra khi

và chỉ khi x  1; y  2 .

Vậy giá trị lớn nhất của P là 22, đạt được tại x  1; y  2

Câu 3(3.0 điểm). Cho tam giác ABC nhọn nội tiếp đường tròn O  với AB  AC . Phân

 cắt BC tại D và cắt đường tròn


giác của góc BAC O  tại E khác A. Gọi M là trung điểm
của đoạn thẳng AD. Đường thẳng BM cắt đường tròn O  tại P khác B. Giả sử các đường

thẳng EP và AC cắt nhau tại N.

a) Chứng minh rằng tứ giác APNM nội tiếp và N là trung điểm của đoạn thẳng AC.
 nên ta
Do AE là phân giác của góc BAC
A P

có E là điểm chính giữa cung BC.


  ANP
Từ đó ta được AMP  nên tứ giác
M N
AMNP nội tiếp đường tròn.
O Q
  ANM
Do đó APM  . Lại có R

  ACB
APM  nên suy ra ANM
  ACB
. B D C

Từ đó dẫn đến MN dong song với BC,

mà M là trung điểm AD nên suy ra N là E

Tài liệu sưu tầm và tổng hợp bản word đầy đủ liên hệ 0393732038 TÀI LIỆU TOÁN HỌC
54
Website:tailieumontoan.com
trung điểm AC

b) Giả sử đường tròn K  ngoại tiếp

tam giác EMN cắt đường thẳng AC tại

Q khác N. Chứng minh B và Q đối xứng

nhau qua AE.

Không mất tính tổng quát ta giả sử Q nằm giữa N và C(các trương hợp còn lại chứng

minh tương tự).


  MNA
Do tứ giác EMNQ nội tiếp nên MEQ  . Mà ta có MNA
  ACB
 và ACB
  AEB

  AEB
nên ta suy ra được AEQ  . Lại có BAE  và AE chung nên suy ra
  CAE

ABE  ACE .

Do đó AB  AG và EB  EQ nên AE là đường trung trực của BQ, suy ra Q và B đối

xứng nhau qua đường thẳng AE.

c) Giả sử đường tròn K  cắt đường thẳng BM tại M. Chứng minh rằng RA vuông góc RC

  EMR
Tứ giác ERMN nội tiếp đường tròn nên ta có ENR   AMP
.

  ANP
Lại có ENC  nên ta được ERN
  AMP   ENC
.

  PMN
Ta có REN   PAN
  PEC   CEN
 và REN 

Kết hợp với cạnh NE chung ta suy ra được REN  CEN

1
Suy ra RN  NC  NA nên RN  AC , điều này dẫn đến tam giác RAC vuông tại R
2
hay ta được RA vuông góc với RC.

Câu 4(1.0 điểm). Số nguyên a được gọi là số “đẹp” nếu với mọi cách sắp xếp theo thứ tự

tùy ý của 100 số 1, 2, 3,…, 100 luôn tồn tại 10 số hạng liên tiếp có tổng lớn hơn hoặc bằng a.

Tìm số “đẹp” lớn nhất

1  100100
 Lời giải. Tổng của 100 số của dãy số là  5050 . Chia 100 số thành 10 bộ số
2
gồm 10 số liên tiếp thì trung bình tổng của 10 bộ số này là 505. Khi đó tồn tại ít nhất một

bộ số mà tổng 10 số đó lớn hơn hoặc bằng 505. Ta sẽ chứng minh a lớn nhất chỉa có thể

Tài liệu sưu tầm và tổng hợp bản word đầy đủ liên hệ 0393732038 TÀI LIỆU TOÁN HỌC
55
Website:tailieumontoan.com
bằng 505 bằng cách cách chọn ra ví dụ mà tổng 10 số liên tiếp bất kỳ nhỏ hơn hoặc bằng

505, khi đó mọi số a lớn hơn 505 đều không thỏa mãn.

Thật vậy, xét cách sắp xếp sau 100,1, 99,2, 98, 3, , 51, 50 (chia thành các cặp có tổng

bằng 101, viết số lớn đứng trước rồi xếp các cặp cạnh nhau theo thứ tự giảm dần của số

lớn hơn). Nếu 10 số liên tiếp gồm 5 cặp số như vậy thì tổng 10 số này là 505. Nếu không

10 số gồm số đầu nhỏ hơn trong một cặp và kết thúc là số lớn hơn trong một cặp khác. Các

số này thuộc sáu cặp khác nhau là x,101  x, x  1,102  x, , x  4,105 – x và 10 số được

chọn là các số được chọn là các số 101  x đến x  5 (trong dãy trên ). Dễ thấy tổng 10 số

liên tiếp bất kỳ đều không vượt quá 505. Vậy a  505

Đề số 8

Câu 1 (3.5 điểm).

x 2  4y 2  5

a) Giải hệ phương trình  2
4x  8xy 2  5x  10y  1


Biến đổi tương đương hệ phương trình ta có

x 2  4y 2  5   x  2y 2  4xy  5


x  2y   4xy  5  
2
   
 2
4xy x  2y   5 x  2y   1

4x  8xy  5x  10y  1
2
 4xy  5x  2y   1
 
Đặt a  x  2y;b  4xy , khi đó hệ phương trình trên trở thành

a 2  b  5 b  a 2  5 b  a 2  5 b  a 2  5 b  4


   
  2  3  3  
ab  5a  1
   
a a  5  5a  1 a  5a  5a  1

a  1

a  1

 x  1; y  1
x  2y  1 
Từ đó ta có hệ phương trình    .

xy  1 x  2; y   1
  2
 1
Vậy hệ phương trình đã cho có các nghiệm là x ; y   1;12;   .
 2 

64x 3  4x
b) giải phương trình 5x 2  6x  5 
5x 2  6x  6

Dễ thấy 5x 2  6x  5  5x 2  6x  5  2x 2  2  3 x  1  0 với mọi x. Do đó điều kiện


2

xác định của phương trình là x  R . Phương trình đã cho được viết lại thành

Tài liệu sưu tầm và tổng hợp bản word đầy đủ liên hệ 0393732038 TÀI LIỆU TOÁN HỌC
56
Website:tailieumontoan.com


5x 2  6x  5 5x 2  6x  5  1  4x   4x 
3

Đặt a  5x 2  6x  5;b  4x a  0 . Khi đó phương trình trên được viết lại thành

  
a a 2  1  b 3  b  a  b  a 2  ab  b 2  1  0  a  b 
Từ đó ta được 5x 2  6x  5  4x , phương trình tương đương với

x  0 x  0
   x 1

5x 2
 6x  5  16x 2
11x  6x  5  0
2

Vậy phương trình đã cho có nghiệm duy nhất là x  1 .

Câu 2(2.5 điểm).

x 2  1 y2  1
a) Với x, y là những số nguyên thỏa mãn đẳng thức  . Chứng minh
2 3

x 2  y 2  40 .

x 2  1 y2  1
Biến đổi giả thiết
2

3
  
 3 x 2  1  2 y 2  1  3x 2  2y 2  1 
Vì số chính phương chia 5 dư 0 hoặc 1 hoặc 4, mà 3x 2  2y2  1 nên x 2 và y 2 chia cho 5 có

cùng số dư 1, từ đó ta được x 2  y 2  5

Vì số chính phương chia 8 dư 0 hoặc 1 hoặc 4, mà 3x 2  2y2  1 nên x 2 và y 2 chia cho 8

có cùng số dư 1, từ đó ta được x 2  y 2  8

Do 5 và 8 nguyên tố cùng nhau nên ta được x 2  y 2  40

b) Tìm tất cả các cặp số nguyên x ; y  thỏa mãn đẳng thức sau x 4  2x 2  y 3 .

Ta có x 4  2x 2  y 3  x 4  2x 2  1  y 3  1  x 2  1  y  1y2 – y  1 .
2

 
Gọi d  y  1; y2  y  1 . Khi đó ta có y  1 d và y 2  y  1d nên ta được
2

y  1  y 
2
2
 y  1 d  3y d

Do d là nguyên tố nên ta có hai trường hợp

+ Khi 3 d ta được x 2  1  y  1y2 – y  1 9 nên x 2  1  9  x 2  1 3 . Điều này


2 2

vô lý vì số chính phương chia cho 3 không thể có số dư là 2.

+ Khi 3 d ta được y d , kết hợp với y  1d ta suy ra được d  1 .


Tài liệu sưu tầm và tổng hợp bản word đầy đủ liên hệ 0393732038 TÀI LIỆU TOÁN HỌC
57
Website:tailieumontoan.com


Do đó y  1; y2 – y  1  1 . 
 
Khi đó do y  1 y2 – y  1 là số chính phương nên ta đặt y  1  a 2 ; y2 – y  1  b2

trong đó a, b là các số nguyên dương và a;b   1 . Tứ đó ta được

      
2
b2  a 2  1 – a 2  1  4b2  4a 4  12a 2  12  2b – 2a 2  3 2b  2a 2  3  3

Vì 2b  2a 2  3  2b  2a 2  3 nên ta xét các trường hợp sau


2 2

 
2b  2a  3  1 b  1 , hệ không có nghiệm nguyên.
2

+ Trường hợp 1. Với  


2b  2a 2  3  3 a 2  2
 

2b  2a 2  3  3 b  1 a  1 y  1  1



+ Trường hợp 2. Với         2  x  y  0.
2b  2a 2  3  1 a 2  1 b  1 y – y  1  1

   
Thử lại vào phương trình ban đầu ta thấy thỏa mãn. Vậy phương trình có nghiệm duy

nhất là 0; 0 .

Câu 3 (3.0 điểm). Cho hình vuông ABCD nội tiếp đường tròn O  . P là điểm thuộc cung

nhỏ AD của đường tròn O  và P khác A, D. Các đường thẳng PB, PC lần lượt cắt AD tại

AD tại M, N. Đường trung trực của AM cắt đường thẳng AC, PB lần lượt tại E, K. Đường

trung trực DN cắt các đường thẳng BD, PC lần lượt tại F, L.

Lời giải.

Tài liệu sưu tầm và tổng hợp bản word đầy đủ liên hệ 0393732038 TÀI LIỆU TOÁN HỌC
58
Website:tailieumontoan.com

a) Chứng minh rằng ba điểm K, O, L thẳng hàng.


  KMA
Ta có KA  KM suy ra tam giác AKM cân. Do đó ta được KAM ,

  KBA
Mà ta lại có KMA   900 và KAB
  KAM
  900 nên suy ra   hay tam
KAB  KBA

giác AKB cân ta K. Do đó ta được KA  KB  KM .

Lại có OB  OD nên OK là đường trung bình của tam giác DKM, suy ra OK // MD.

Chứng minh tương tự ta có OL là đường trung bình của tam giác NCA, suy ra OL // AD

Theo tiên đề Ơclit thì ba điểm K, O, L thẳng hàng.

b) Chứng minh đường thẳng PO đi qua trung điểm của EF


  450 nên tam giác EAM vuông cân.
Ta có E thuộc đường trung trực AM và EAM

Do đó suy ra ME vuông góc với AC. Hoàn toàn tương tự ta cũng có NF vuông góc với BD.

PB PC
Ta có MN song song với BC nên theo định lí Talet ta có  .
MB NC
Hạ PX vuông góc với AC và PY vuông góc với BD, khi đó ta có PX, EM, BO cùng song

song với nhau.

XO PB
Do đó ta được  .
EO PM

Tài liệu sưu tầm và tổng hợp bản word đầy đủ liên hệ 0393732038 TÀI LIỆU TOÁN HỌC
59
Website:tailieumontoan.com
YO PC
Lại có PY, FN, CO cùng song song với nhau nên ta cũng có  .
FO NC

XO YO
Từ đó dẫn đến  nên suy ra XY và EF song song với nhau.
EO FO
  PYO
Ta có PXO   XOY
  900 nên tứ giác PXOY là hình chữ nhật, do đó PO đi qua

trung điểm của XY. Do XY song song với EF nên PO đi qua trung điểm của EF.

c) Giả sử đường thảng EK cắt đường thẳng FL và AC cắt nhau tại T. Đường thẳng ST cắt

các đường thẳng PB, PC lần lượt tại U và V. Chứng minh rằng bốn điểm K, L, V, U cùng

thuộc một đương tròn.


  OAD
Ta có LK song song với AD nên LK vuông góc với ES. Do đó KOA   450 nên

  450 .
KEO
  900 nên OK là phân giác của góc EOS
Mà ta có EOS .

Suy ra tam giác EOS cân nên ta có KS  KE , suy ra KL là đường trung trực của ES hay E

và S đối xứng với nhau qua KL. Hoàn toàn tương tự ta có F và T đối xứng qua KL.
  STO
Từ đó ta được EOF  SOT nên EFO .

Gọi giao điểm của OP và EF là I, ta có I là trung điểm của EF.

Do tam giác OEF cân nên ta có IO  IE  IE .


  IF
Suy ra tam giác IOF cân nên IOF  .
O  OTS

Mà IOE   EOF
  IOF   900 nên IOE
  OTS
  900 .

  IOE
Gọi giao điểm của OP và ST là H nên ta cóTOH   HTO
 , suy ra TOH   900 .

  900 hay PO vuông góc với ST.


Từ đó suy ra THO
  PCD
Ta có PLF  và PCD
  PBD
  BPO
 nên PLF
  PBO
  PVH
.

  HVP
Lại có PH vuông góc với UV nên VPH   900 .

  PLK
Mà ta lại có PLF   900 nên   .
  PLK
PLK  HVP
  UVP
Từ đó suy ra PLK  hay tứ giác KLUV nội tiếp đường tròn.

Tài liệu sưu tầm và tổng hợp bản word đầy đủ liên hệ 0393732038 TÀI LIỆU TOÁN HỌC
60
Website:tailieumontoan.com
Câu 4(1.0 điểm). Chứng minh rằng với mọi số tự nhiên n  3 luôn tồn tại cách xếp bộ n số

xi  xk
1,2, 3,...,n thành bộ số x 1, x 2 , x 3 ,..., x n sao cho x j  với mọi bộ chỉ số i; j ; k  mà
2
1i  j k n .

Lời giải.

a j  ak
Dãy a1 , a2 , a 3 ,..., as chiều dài s  3 tùy ý được gọi là dãy “tốt” nếu a j  với mọi
2
chỉ số (i, j, k) thỏa mãn ( 1  i  j  k  s ).

Nếu dãy a1 , a2 , a 3 ,...., as là dãy tốt thì dãy 2a1 , 2a2 , 2a 3 ,..., 2as và dãy

2a1  1, 2a2  1, 2a 3  1,..., 2as  1 cũng là dãy tốt.

Từ nhận xét trên ta suy ra nếu dãy x 1 , x 2 , x 3 ,..., x s là dãy tốt của các số 1, 2, 3, ... , s ( s  3 )

thì dãy 2x 1 , 2x 2 , 2x 3 ,..., 2x s , 2x 1  1, 2x 2  1,..., 2x s  1 là dãy tốt của các số 1, 2, 3, ..., 2s (chú ý

2x k  2x m  1
rằng không là số nguyên).
2
+) (1, 3, 2) là dãy tốt của các số 1, 2, 3.

+) Với n  3 luôn tồn tại k để 3.2k 1  n  3.2k . . Theo nhận xét trên, ta xây dựng được dãy

tốt từ các dãy tốt từ các số 1, 2, 3,..., 3.2k sau đó ta bỏ đi các số n  1, n  2, n  3,...., 3.2k

chúng ta nhận được dãy tốt từ các số 1, 2, 3, ..., n (trên dãy tốt ta bỏ đi các số hạng bất kì

thì dãy còn lại vẫn là dãy tốt).

Cách khác:

Với n  3 ta có cách sắp xếp 1, 3, 2.

Ta chứng minh rằng nếu bài toán đúng với n sẽ đúng với 2n.

Thật vậy, giả sử ta có cách xắp xếp đúng với n thì cách sắp xếp đó có dạng

xi  xk
x 1, x 2 , x 3 , x 4 ,..., x n thỏa mãn với mọi 1  i  j  k  n ta có x j  . Ta chứng minh
2
tồn tại dãy 2n thỏa mãn đề bài .

Xét dãy sau 2x1,2x 2 ,2x 3 ,...,2x n , 2x1  1,2x 2  1,2x 3  1,...,2x n  1 , dãy số gồm tất cả các số

từ 1 đến 2n. Xét a  b bất kỳ cùng thuộc dãy

a b
+ Nếu a, b khác tính chẵn lẻ thì không thuộc dãy (thỏa mãn)
2
Tài liệu sưu tầm và tổng hợp bản word đầy đủ liên hệ 0393732038 TÀI LIỆU TOÁN HỌC
61
Website:tailieumontoan.com
a b
+ Nếu a, b cùng tính chẵn lẻ thì thuộc dãy
2
Nếu a, b cùng chẵn (trường hợp a, b cùng lẻ chứng minh tương tự )

a b a b
Trường hợp 1. Khi lẻ thì không thể nằm giữa a, b do cách xây dựng dãy (thỏa
2 2
mãn)

a b
Trường hợp 2. Khi chẵn.
2

a b a b
Giả sử rằng nằm giữa a, b trong dãy khi đó a  2x i , b  2x k ,  2x j với
2 2
i  j  k suy ra dãy ban đầu x1,x 2 ,x 3 ,..., x n là cách sắp xếp không thỏa mãn đề bài (mâu

thuẫn )

a b
Vậy điều giả sử là sai nên không nằm giữa a và b trong dãy.
2
Vậy với mọi trường hợp trung bình cộng của a, b không thể nằm giữa a, b suy ra đã xây

dựng được cách xếp thỏa mãn cho trường hợp 2n. Như vậy đã chứng minh được rằng nếu

bài toán đúng với n thì đúng với 2n.

Mặt khác bài toán đúng với n thì đúng với n  1 . Nên theo nguyên lí quy nạp ta có điều

phải chứng minh.

Đề số 9

Câu 1.
1). Giả sử a; b là hai số thực phân biệt thỏa mãn
 2
 a  3b  2
a).  2  a 2  b2  3a  b  0 .
b  3a  2

 a  ba  b  3a  b  0  a  ba  b  3  0
 a  b  0 ( l)

 a  b  3
.

3
b). Với a  b  3  a  b  27
 a 3  b3  3ab a  b  27  a 3  b3  9 ab  27
2
 a 2  3a  b2  3b  4  a  b  2 ab  3a  b  4  ab  2 .
Vậy a 3  b3  45 .

Tài liệu sưu tầm và tổng hợp bản word đầy đủ liên hệ 0393732038 TÀI LIỆU TOÁN HỌC
62
Website:tailieumontoan.com
Nhận xét: Đây là bài toán hết sức cơ bản, đề yêu cầu gì ta làm đó, với một vài phép biến
đổi tương đương cùng kết hợp hằng đẳng thức quen thuộc ta sẽ suy ra được điều phải
chứng minh.
Nhắc lại kiến thức và phương pháp:
2
• Hằng đẳng thức a  b  a2  2ab  b2
3
và a  b  a3  3a2 b  3ab2  b3 .
ma 2  nb  p (1)

• Hệ phương trình đối xứng dạng  2 .

mb  na  p (2)
Lấy (1) + (2), ta có: ma2  b2   na  b  2 p .
a  b

Lấy (1) – (2), ta có: ma  b   na  b  
2 2
.
a  b  n
 m
Từ đó suy ra:
   n2  n
2 p  m a  b  2 ab  na  b  m 2  2 ab  n.  ab  ...
2

   m  m
Bài tập kết thúc.
Bài tập tương tự:
1. Cho a; b thỏa mãn a2  4b  3; b2  4a  3 . Tính a5  b5 .
2. Cho a; b thỏa mãn a2  2b2  4b  7; b2  2a2  4a  7 . Tính giá trị của biểu thức
a3  b3  ab .

2). Ta thấy x  y  0 là nghiệm của phương trình.


Nếu y  0 , nhân hai vế của phương trình với y , ta được
2 xy  3 y 2  5 xy 2 2 x  3 y  5 xy
 2  2
4 x  y 2  5 xy 2 4 x  y 2  5 xy 2
 

2 x  3 y  5 xy 2 x  3 y  5 xy

  2  
 2

 2 x  xy  y 2
 0  2
4 x  y  5 xy
2
 
2 x  3 y  5 xy 2 x  3 y  5 xy
   
 x  y2 x  y  0  x  y2 x  y  0
 
2 x  3 y  5 xy
  x y1
 x  y  0
 
 .
2 x  3 y  5 xy 2  4
 
2 x  y  0  x  5 , y  5

Nhận xét: Bài toán sử dụng phương pháp thế tích từ phương trình một xuống phương
trình hai của hệ, và đưa về phương trình đẳng cấp bậc hai để tìm mối liên hệ giữa hai biến
rồi thế ngược lại phương trình một, tìm nghiệm của hệ phương trình.
Nhắc lại kiến thức và phương pháp:
• Phương trình đẳng cấp bậc hai ẩn x , y là ax 2  bxy  cy 2  0 (*).
Làm nháp: Chia phương trình (*) cho y 2 ta có:

Tài liệu sưu tầm và tổng hợp bản word đầy đủ liên hệ 0393732038 TÀI LIỆU TOÁN HỌC
63
Website:tailieumontoan.com
2
 x  x x x
a    b    c  0 . Coi đây là một phương trình bậc hai ẩn , giả sử tìm được  m
 y   y  y y
x
hoặc  n , do đó ta phân tích được
y
 x  my
(*)  ax 2  bxy  cy 2  0   x  my x  ny  0   .

 x  ny
Ý tưởng: Dạng hệ sử dụng phương pháp thế khá là đặc trưng nhưng cái khó là tìm các đại
lượng biểu diễn nhân tử chung giữa hai biến x; y . Nhìn vào vế trái của từng phương
trình, với phương trình một là bậc nhất, còn phương trình hai là bậc hai, tương tự ở vế
phải, bậc của phương trình hai cũng lớn hơn một bậc so với phương trình một. Điều nó sẽ
làm ta nghĩ đến phép nhân để đồng bậc hóa chúng, nhân ở đây sẽ là nhân chéo từng
phương trình. Tuy nhiên nếu quan sát kỹ hơn một chút, cả hai phương trình đều xuất hiện
5xy nên ta sẽ thế 5 xy  2 x  3 y xuống phương trình hai (thực chất đây cũng là việc nhân
chéo nhưng ta đã giảm biến y , khi đó hệ phương trình đã cho trở thành:
2 x  3 y  5 xy
2 x  3 y  5 xy 2 x  3 y  5 xy 
 2    
 x  y .
4 x  y 2  2 x  3 y y 2 x 2  xy  y 2  0 
   2 x  y  0
Việc còn lại chỉ là tìm nghiệm của hệ phương trình đã cho.
Bài toán kết thúc.
Bài tập tương tự:

x  3 y  5 xy
1. Giải hệ phương trình  2 .

2 2
x  y  5 xy

2 1  1
Đáp số:  x; y  0; 0 ,  ;   , 1;  .
5 5  2

3 x  3 y  6 xy
2. Giải hệ phương trình  2 .
 2 2
9 x  y  6 xy

1   5 5
Đáp số:  x; y  0; 0 ,  ;  1 ,  ;  .
3  6 4
Câu 2.
1). Ta có xy  1   x  1 y  1 , suy ra xy  1  xy  1  x  y .
Mà xy  1  x  y  xy  1  x  y   x  1   y  1   x  1 y  1 , suy ra x  1  y  1 và y  1  x  1 ,
nên x  y
2
x 2  1   x  1 ta có x  1  x  1 , suy ra 2  x  1 , nên x  2 hoặc x  3 .
Nhận xét: Bài toán tìm số nguyên thỏa mãn một điều kiện, sử dụng tính chất chia hết của
một tổng, hiệu, tích,…
Nhắc lại kiến thức và phương pháp:
• Biến đổi đại số.
Ta có ( x − 1)( y − 1) = xy − x − y + 1 nên:
( xy − 1)  ( x − 1)( y − 1) ⇔ ( xy − 1)  ( xy − x − y + 1) .
• Một số luôn chia hết cho chính nó.
Tài liệu sưu tầm và tổng hợp bản word đầy đủ liên hệ 0393732038 TÀI LIỆU TOÁN HỌC
64
Website:tailieumontoan.com

Ta có ( xy − x − y + 1)  ( xy − x − y + 1) .
• Hai số chia hết cho một số thì hiệu của hai số đó.
( xy − 1)  ( xy − x − y + 1)
 ⇒ ( x + y − 2 )  ( xy − x − y + 1)
( xy − x − y + 1)  ( xy − x − y + 1)
⇔ ( x − 1) + ( y − 1)  ( xy − x − y + 1) ⇔ ( x − 1) + ( y − 1)  ( x − 1)( y − 1) .
• Tính chất chia hết a  b thì ka  b với a; b; k là các số nguyên.
( x − 1) + ( y − 1)  ( x − 1)( y − 1) ⇒ ( x − 1) + ( x − 1)( y − 1)  ( x − 1)( y − 1) .
2

• Một tổng hai số hạng chia hết cho một số trong đó có một số hạng chia hết cho số đó
thì số hạng còn lại cũng chia hết.
( x − 1)2 + ( x − 1)( y − 1)  ( x − 1)( y − 1) 2
   x  1   x  1 y  1   x  1   y  1 ;
( x − 1)( y − 1)  ( x − 1)( y − 1)
hoàn toàn tương tự ta có ( y − 1)  ( x − 1) .
• Có hai số thỏa mãn: Số thứ nhất chia số thứ hai và số thứ hai chia hết cho số thứ nhất
thì hai số bằng nhau.

 x  1   y  1
Ta có   x 1  y 1  x  y .
 y  1   x  1



Thay lại vào đề bài ta có ( x 2 − 1)  ( x − 1) ⇔ ( x − 1)( x + 1)  ( x − 1)
2 2

⇔ ( x + 1)  ( x − 1) ⇔ ( x − 1) + 2   ( x − 1) ⇒ 2  ( x − 1) .
• Số thứ nhất chia hết cho số thứ hai thì số thứ hai là ước của số thứ nhất.
2  ( x − 1) ⇒ ( x − 1) ∈ U( 2 ) ={±1; ± 2} .

vì x ≥ 2 ⇔ x − 1 ≥ 1 suy ra ( x − 1) ∈ {1; 2}
x − 1 = 1 x = 2 ⇒ y = 2
⇔ ⇔ .
x − 1 = 2 x = 3 ⇒ y = 3
Vậy x= y= 2 hoặc x= y= 3 .
x 2 y 2  1
2). Ta có x 2 y 2  2 y  1  0  2 y  x 2 y 2  1  y 
2
xy xy
P 
3x y  1  2 3 x 2 y 2  1
2 2

 3 Px 2 y 2  2 xy  P  0 .
Ta có  4  12P 2 .
Phương trình có nghiệm khi  0
1 1 1
 4  12 P 2  0  1  3 P  1  P P .
3 3 3
3 12 3 7
Vậy MaxP = ⇔ x =− và y = − .
3 21 6
Nhận xét: Bài toán tìm giá trị của một biểu thức sau khi biến đổi biểu thức trở thành tham
số của một phương trình bậc hai theo một ẩn
Nhắc lại kiến thức và phương pháp:

Tài liệu sưu tầm và tổng hợp bản word đầy đủ liên hệ 0393732038 TÀI LIỆU TOÁN HỌC
65
Website:tailieumontoan.com
• Biến đổi giả thiết.
x 2 y 2  1
Ta có x 2 y 2  2 y  1  0  2 y  x 2 y 2  1  y  .
2

• Đưa biểu thức đã cho về một ẩn hoặc một biểu thức cố định.
−x2 y 2 − 1 xy
Thay y = vào biểu thức P ta có P = 2 2
2 −x y − 1
3 +1
2
2 xy 2 xy
= = .
( 2 2
3 − x y − 1 + 2 −3 x y − 1
2 2
)
2a
Đặt xy = a ta được P = .
−3a 2 − 1
• Tìm giá trị lớn nhất, nhỏ nhất của một biểu thức bằng cách đưa biểu thức đó trở thành
tham số của một phương trình bậc hai
2a
=P 2
+ P 0 Khi đó ta có ∆′ = 12 − 3P 2 .
⇒ 3 Pa 2 − 2 a=
−3a − 1
• Phương trình bậc hai có nghiệm khi và chỉ khi ∆ hoặc ∆ ' không âm.
Ta có Vì phương trình có nghiệm nên ∆′ ≥ 0 ⇔ 1 − 3P 2 ≥ 0
( )(
⇔ 1 − 3P 1 + 3P ≥ 0 ⇔ ) ( 3P − 1 )( )
3P + 1 ≥ 0

−1 1 1
⇔ −1 ≤ 3 P ≤ 1 ⇔ ≤P≤ ⇒P≤ .
3 3 3
• Khi xét đến giá trị nhỏ nhất, giá trị lớn nhất của biểu thức cần phải xét xem “dấu bằng
xảy ra khi nào?”.
1
Ta có dấu bằng xảy ra khi vào chỉ khi P = ⇔ ∆ ' = 0 ⇒ Phương trình bậc hai có
3
2
nghiệm kép a = , thay vào biểu thức ban đầu ta có:
3
2
2
 2 
   2 y  1  0  2 y   7  y   7  x  xy  3   12 .
 3  3 6 y 7 7 3

6
3 12 3 7
Vậy MaxP = ⇔ x =− và y = − .
3 21 6
Câu 3.

Tài liệu sưu tầm và tổng hợp bản word đầy đủ liên hệ 0393732038 TÀI LIỆU TOÁN HỌC
66
Website:tailieumontoan.com

BD AB
1). Ta có AD là phân giác   mà BED; CDF là tam giác cân,
DC AC
BE AB
   BC  FE .
CF AC
Nhận xét. Bài toán chứng minh hai đường thẳng song song sử dụng định lý Ta-lét đảo
Nhắc lại kiến thức và phương pháp.
• Tâm đường tròn nội tiếp tam giác là giao điểm của ba đường phân giác.
Trong ∆ABC có I là tâm đường tròn nội tiếp nên suy ra AI hay AD là phân giác của
.
BAC
• Tính chất đường phân giác trong tam giác.
DB AB
Trong ∆ABC có AD là phân giác nên = .
DC AC
• Hai điểm đối xứng nhau qua một đường thẳng thì đường thẳng đó là trung trực của
đoạn thẳng nối hai điểm.
+ E là điểm đối xứng của D qua BI nên BI là trung trực của DE ;
+ F là điểm đối xứng của D qua CI nên CI là trung trực của DF .
• Một điểm thuộc trung trực của một đoạn thẳng thì điểm đó cách đều hai đầu mút của
đoạn thẳng đó.
+ B ∈ BI mà BI là trung trực của DE nên BD = BE ;
+ C ∈ CI mà CI là trung trực của DF nên CD = CF ;
BD BE BE AB BE CF
suy ra = do đó = ⇔ =
CD CF CF AC BA CA
• Định lý Ta-lét đảo: Hai điểm thuộc hai đoạn thẳng, chia hai đoạn thành các đoạn
thẳng tương ứng tỷ lệ thì hai đường thẳng chứa hai đoạn thẳng đó song song.
BE CF
Theo định lý Ta-lét đảo ta có
= thì EF  BC .
BA CA
2). Ta có   EDB
BC  EF  EFD   BED
 .
  180 0  AEM
Mà APM   BED
  APM
  DEF
.
  APN
Tương tự: DFE   APN
  APM
  DFE
  FED
  MPN
.
  MDN
Mà MJN   EDF
  MJN
  MPN
  180 0  MPNJ nội tiếp.

Tài liệu sưu tầm và tổng hợp bản word đầy đủ liên hệ 0393732038 TÀI LIỆU TOÁN HỌC
67
Website:tailieumontoan.com
Nhận xét: Bài toán chứng minh tứ giác nội tiếp bằng cách chứng minh tổng hai góc trong
bằng 180° .
Nhắc lại kiến thức và phương pháp:
• Hai đường thẳng song song tạo ra các góc ở vị trí so le trong bằng nhau.
 = EDB
Ta có EF  BC , suy ra FED .
• Tam giác có hai cạnh bằng nhau là tam giác cân, tam giác cân có hai góc kề cạnh đáy
bằng nhau.
 = BED

Tam giác ∆BDE có BD = BE nên ∆BDE cân tại B suy ra BDE , suy ra

FED 
= EDB 
= BED .
• Tứ giác nội tiếp có tổng hai góc đối diện bằng 180° .
Tứ giác APME là tứ giác nội tiếp nên  + AEM
APM = 180 0
 =180 0 − AEM
⇔ APM  =BED
 ⇒ APM
 =DEF
.

Hoàn toàn tương tự có


  APN
DFE   APN
  APM
  DFE
  FED
  MPN
.
• Trung điểm; Đường trung bình của tam giác.
 MJ  DF

+ MJ là đường trung bình của ∆DFE nên  1 ;
 MJ = DF
 2
DF
+ N là trung điểm của DF nên DN = ;
2
 MJ  DN ( DF )
suy ra  .
=MJ DN =( DF )
• Tứ giác có một cặp cạnh song song và bằng nhau là hình bình hành.
Tứ giác MJND có MJ  DN và MJ = DN nên MJND là hình bình hành.
• Hình bình hành có hai góc đối diện bằng nhau.
  MDN
+ MJND là hình bình hành nên MJN ;
  EDF
+ MDN  (hai góc so le trong của MN  FE suy ra tính chất đường trung bình);
  MPN
suy ra MJN   180 0 .

• Tứ giác có tổng hai góc trong đối diện bằng 180° là tứ giác nội tiếp.
  MPN
Tứ giác MPNJ có MJN   180 0 nên tứ giác MPNJ là tứ giác nội tiếp.

  DEF
3). Ta có APM   JNM
 và JPM   JEM
  JPM
  APM
 , suy ra 3 điểm A; P ; J thẳng

hàng.
Nhận xét: Bài toán chứng minh ba điểm thẳng hàng bằng cách chứng minh ba điểm cùng
thuộc một đường thẳng đặc biệt
Nhắc lại kiến thức và phương pháp:
• Tứ giác nội tiếp có góc ngoài và góc trong không kề với nó bằng nhau.
 = BED
Tứ giác MPAE là tứ giác nội tiếp nên MPA  = DEF
 mà BED  ( = BDE
 ), suy ra
 = DEF
MPA .
• Trung điểm, Đường trung bình, Hình bình hành,…(đã nhắc lại phần trên).

Tài liệu sưu tầm và tổng hợp bản word đầy đủ liên hệ 0393732038 TÀI LIỆU TOÁN HỌC
68
Website:tailieumontoan.com
 MN  FE

+ MN là đường trung bình của ∆DEF nên  1 ;
 MN = FE
 2
1
+ J là trung điểm FE nên EJ = EF ; suy ra MN  JE và JE = MN nên MNJE là hình
2
    (hai góc nội tiếp cùng chắn cung MJ
bình hành nên MEJ = MNI mà MEJ = MNP  của
 , suy ra 
  DEF
đường tròn ngoại tiếp tứ giác MNPJ ) suy ra JPM  nên
JPM = APM
PJ ≡ PA hay ba điểm A; P ; J thẳng hàng.
Câu 4.
1). Theo đề bài, các số nguyên dương được sắp xếp theo từng hàng chéo của bảng: Hàng
chéo thứ nhất có 1 số, hàng chéo thứ hai có 2 số,……
Giả sử số x nằm ở hàng chéo thứ k thì ta có:
k  k  1 k  k  1 1  1  8 x 1  1  8x
x  k
2 2 2 2
1  1  8 x
k .
2

1  1  8.2015
Áp dụng x  2015 ta có k   63 .
2
k  k  1
Số đầu tiên ở hàng chéo thứ k  63 là  1  1954 .
2
Như vậy số 2015 nằm ở vị trí thứ 2015  1954  1  62 của hàng chéo thứ 63 (vị trí áp chót)
Tọa độ của nó là 2,62 .
2). Theo Côsi cho 4 số ta có
4  abc  ab  bc  ac  4 4 a 3 b3 c 3  1  abc  a  b  c  3 3 abc  3 3 a 2 b2 c 2 .
BĐT tương đương a2  b2  c 2  3 3 a2 b2 c 2  2 ab  bc  ac (1).
Đặt 3
a 2  x; 3 b2  y ; 3 c 2  z ( x; y ; z  0 ).
 x 3  y 3  z 3  3 xyz  2 x 3 y 3  2 z 3 x 3  2 z 3 y 3 .
Áp dụng bất đẳng thức Schur bậc 3:
x 3  y 3  z 3  3 xyz  xy  x  y  yz  y  z  xz  x  z
 x  x  y x  z  y  y  x y  z  z  z  x z  y  0
với mọi số thực không âm x; y; z .
Chứng minh bất đẳng thức
Do vai trò x; y; z như nhau, giả sử x  y  z  z  z  x z  y  0 .
Ta xét x  x  z  y  y  z  x 2  xz  yz  y 2   x  y x  y  z  0
 x  x  z x  y  y  y  z x  y  0  x  x  z x  y  y  y  z y  x  0
 x  x  y x  z  y  y  x y  z  z  z  x z  y  0 .
(điều phải chứng minh).
Ta có x  y  z  3xyz  xy  x  y  yz  y  z  xz  x  z
3 3 3

Tài liệu sưu tầm và tổng hợp bản word đầy đủ liên hệ 0393732038 TÀI LIỆU TOÁN HỌC
69
Website:tailieumontoan.com

 2 x3 y 3  2 z3 x3  2 z3 y 3 .
xyz 
Dấu “=” xảy ra khi   a b c 1.
 x  y; z  0
Nhận xét: Bài toán sử dụng bất đẳng thức Cosi, phép đặt ẩn phụ đồng thời chứng minh
qua bất đẳng thức trung gian hay là bất đẳng thức bổ đề (bất đẳng thức Schur bậc ba) để
suy ra điều phải chứng minh.
Nhắc lại kiến thức và phương pháp:
• Bất đẳng thức Cosi cho ba số thực dương a  b  c  3 3 abc .
• Bất đẳng thức Schur bậc ba cho các số thực dương
x 3  y 3  z 3  3 xyz  xy  x  y  yz  y  z  xz  x  z .
Bài toán kết thúc.
Bài tập tương tự:
1. Chứng minh rằng với mọi a; b; c  0 , ta luôn có:
2 a 2  b2  c 2   abc  8  5a  b  c .
2. Cho a; b; c  0 là các số thực dương thỏa mãn a  b  c  3 . Chứng minh rằng
 1 1 1 12
5     3.
 a b c  abc

ĐỀ SỐ 10

Câu 1.
3a  b  c  x

1). Đặt 3b  c  a  y .

3c  a  b  z
3 3 3 3
Ta có 3a  3b  3c  24  3a  b  c  3b  c  a  3c  a  b
3
  x  y  z  24  x 3  y 3  z 3
3 3
  x  y  z  24   x  y  z  3 x  y y  z x  z
 24  3 x  y y  z x  z  0  24  32 a  4b2b  ac2c  4 a  0
 24  24 a  2bb  2cc  2 a  0  a  2bb  2cc  2 a  1 .
Nhận xét: Bài toán sử dụng phương pháp đặt ẩn phụ, khai thác biến đổi từ giả thiết, điểm
nhấn ở đây chính là hằng đẳng thức bậc ba để suy ra điều phải chứng minh.
Nhắc lại kiến thức và phương pháp:
3
• Hằng đẳng thức  x  y  x 3  3x 2 y  3xy 2  y 3
• Hằng đẳng thức
3 3 2
 x  y  z   x  y  3 x  y z  3 x  y z 2  z 3
2
 x 3  3 x 2 y  3 xy 2  y 3  3 x  y z  3 x  y z  z 3
2
 x 3  y 3  z 3  3 xy  x  y  3 x  y z  3 x  y z
 x 3  y 3  z 3  3 x  y xy  xz  yz  z 2 

Tài liệu sưu tầm và tổng hợp bản word đầy đủ liên hệ 0393732038 TÀI LIỆU TOÁN HỌC
70
Website:tailieumontoan.com
 x 3  y 3  z 3  3 x  y y  z z  x .
Ý tưởng: Bài toán cho giả thiết khá là cồng kềnh, phức tạp thậm chí bậc to, nhưng quan sát
các đại lượng bên vế phải, ta thấy các biểu thức 3a  b  c ,
3b  c  a và 3c  a  b độc lập so với nhau mặt khác:
3a  b  c  3b  c  a  3c  a  b  3a  3b  3c
Vì thế nếu đặt x  3a  b  c; y  3b  c  a; z  3c  a  b thì giả thiết bài cho tương đương
3
với  x  y  z  24  x 3  y 3  z 3 . Khoan nói đến chuyện biến đổi nói, bây giờ ta sẽ quan
sát đến biểu thức cần chứng minh, với phép ẩn phụ trên ta có thể hoàn toàn rút a; b; c theo
x; y ; z . Xét biểu thức a  2b biểu thức này không chứa c vậy nên ta cần tìm mối liên hệ
giữa x; y; z sao cho c triệt tiêu, dễ thấy ở x chứa c còn y chứa c nên suy ra
x  y  2 a  4b  2 a  2b , tương tự y  z  2 b  2c ; z  x  2 c  2 a . Do đó ta cần chứng
minh  x  y y  z z  x  8 . Bây giờ ta quay ngược lại với giả thiết, xuất hiện hằng đẳng
3
thức bậc ba, nên ta sẽ khai triển nó, ta được:  x  y  z  x 3  y 3  z 3  3 x  y y  z z  x
nên suy ra:
x 3  y 3  z 3  3 x  y y  z z  x  24  x 3  y 3  z 3
  x  y y  z z  x  8  điều phải chứng minh.
Bài toán kết thúc.
Bài tập tương tự:
1. Cho a; b; c là ba số thực dương thỏa mãn điều kiện
3 3 3 3
 a  b  c  12  a  b  c  b  c  a  a  c  b . Chứng minh rằng 2 abc  1 .


2 x  2 y  xy  5
2). Ta có 
27  x  y  y  7  26 x 3  27 x 2  9 x
3


 x  2 y  2  9


 


 27 x  y  y 3  7  26 x 3  27 x 2  9 x
 y 3  x 3  7  3 x  y x  2 y  2  27 x 3  27 x 2  9 x
2 3
 y 3  x 3  8  3 xy  x  y  12  x  y  6  x  y  3 x  1
3 3
  x  y  2  3 x  1  x  y  2  3 x  1
x  1  y  1

 y  1  2 x   x  22 x  1  9   .
 x   7  y  8
 2
 7 
Vậy  x; y  1; 1 ,  ;  8 .
 2 
Nhận xét: Bài hệ phương trình rất hay ở chỗ kết hợp cả hai phương trình, sau đó nhóm lại
hằng đẳng thức đưa được về phương trình biểu diễn mối liên hệ giữa x; y và thế ngược
lại tìm nghiệm của hệ phương trình.
Nhắc lại kiến thức và phương pháp:
3
• Hằng đẳng thức  x  y  x 3  3x 2 y  3xy 2  y 3 .

Tài liệu sưu tầm và tổng hợp bản word đầy đủ liên hệ 0393732038 TÀI LIỆU TOÁN HỌC
71
Website:tailieumontoan.com
3 3 2
• Hằng đẳng thức  x  y  z   x  y  3 x  y z  3 x  y z 2  z 3
2
 x 3  3 x 2 y  3 xy 2  y 3  3 x  y z  3 x  y z  z 3
2
 x 3  y 3  z 3  3 xy  x  y  3 x  y z  3 x  y z
 x 3  y 3  z 3  3 x  y xy  xz  yz  z 2 
 x 3  y 3  z 3  3 x  y y  z z  x
3 3
• Giải phương trình tổng quát  f  x   g  x
  f  x  g  x  f 2  x  f  x g  x  g 2  x  0  f  x  g  x .
  
Ý tưởng: Phương trình thứ hai trong hệ khá phức tạp, ta sẽ đi khai thác nó trước, để ý bên
vế phải của phương trình hai, xuất hiện các tổng 27 x 2  9 x mà ta thấy
2 3
27 x 2  9 x  3.3 x  3.3 x có bóng dáng của hằng đẳng thức nếu thêm đại lượng 3 x  13
3
khi đó : 26 x 3  27 x 2  9 x  3x  1  x 3  1
Chuyển đại lượng x 3  1 sang vế trái của phương trình hai, ta được:
3
x 3  y 3  8  27  x  y  3 x  1 ( i ).
Điều còn lại là ta chưa khai thác tới phương trình một, nhưng quan sát ( i ) một chút, ta
thấy vế phải là một lũy thừa bậc ba, ta sẽ biến đổi vế trái ( i ) cũng về dạng đó. Bởi lẽ
3
x 3  y 3  8  x 3  y 3  2 3 ta sẽ nghĩ đến chuyện đưa vế trái ( i ) về dạng  x  y  2 . Hay nói
cách khác, kết hợp phương trình một của hệ, ta cần chứng minh:
3
x 3  y 3  8  27  x  y   x  y  2 .
Sẽ có hai cách để chứng minh, một là đi xuôi, hai là chứng minh ngược:
• Từ phương trình một của hệ, có  x  2 y  2  9 thế vào vế trái của ( i ), ta được:
3
x 3  y 3  2 3  3 x  y x  2 y  2   x  y  2 .
3
• Ta có  x  y  2  x 3  y 3  2 3  3 x  y x  2 y  2
 3 x  y x  2 y  2  27  x  y  2 x  2 y  xy  5
3 3
Khi đó ta có  x  y  2  3x  1  x  y  2  3x  1  y  2 x  1 , thế ngược lại phương
trình một của hệ ta sẽ tìm được nghiệm của hệ phương trình ban đầu.
Bài toán kết thúc.
Bài tập tương tự:

2 x  2 y  xy  5
1. Giải hệ phương trình  .
27  x  y  y  7  7 x 3  12 x 2  6 x
3

Câu 2.

n  5  x2

1). Đặt  ( x , y  ?; x , y  0 ).
 2
n  30  y

 y 2  x 2  25   y  x y  x  1.25 vì ( x , y  ?; x , y  0 ).
y  x  1
  y  13

Lại có y  x  y  x nên  
 .

 y  x  25
 
x  12

Tài liệu sưu tầm và tổng hợp bản word đầy đủ liên hệ 0393732038 TÀI LIỆU TOÁN HỌC
72
Website:tailieumontoan.com
Thay vào ta tính được n  139 thảo mãn.
Nhận xét: Bài toán số học sử dụng tính chất số chính phương, phương trình ước số
Nhắc lại kiến thức và phương pháp:
• Số chính phương được viết dưới dạng a 2 với a là số nguyên.
+ n + 5 là số chính phương nên có dạng x 2 với x là số nguyên.
+ n + 30 là số chính phương nên có dạng y 2 với y là số nguyên
• Hằng đẳng thức A 2 − B2 = ( A + B )( A − B ) .

n  5  x 2
Trừ vế theo vế của  cho nhau ta được
n  30  y 2


y 2 − x 2 = ( n + 30 ) − ( n + 5 ) ⇔ ( y − x )( y + x ) = 25 .
• Phương trình ước số.
( y − x )( y + x ) =25 =1.25 =−
( 1) . ( −25 ) =5.5 =−
( 5 ) . ( −5 ) ;
= y − x 1 =  2 y 26 =   y 13  =n + 30 169
   
  y + x = 25 x = y − 1 =   x 12  n + 5 144
=
=   y − x 25 =  2 y 26 =   y 13  =n + 30 169
   
y + x = 1   x = y − 25   x =−12  n + 5 =144
   
= y − x 5 =  2 y 10 = y 5  =n + 30 25
y + x = 5 x = y − 5 = x 0  
   = n + 5 0
suy ra  ⇔ ⇔ ⇔
  y − x =−5  2 y =−10 y = −5  n + 30 =25
   
  y + x =−5   x =y + 5 = x 0  n + 5 0
=
  y − x =−1  2 y =−26 y = −13  n + 30 =169
   
  y + x =−25   x =y + 1   x =−12  n + 5 =144
   
  y − x =−25  2 y =−26 y = −13  n + 30 =169
  y + x =−1   x =y + 25=   x 12  
    n + 5 144
=
 =n + 30 169 =  n 139
 
= n + 5 144= n 139 n = 139
⇔ ⇔  ⇒ 139 (do n là số tự nhiên).
⇒n=
n + 30 = 25 n =−5  n = 139
 
 n + 5 =0  n =−5

2). Ta thấy 1  x  y  3  x  y và x , y  ?  x , y là số chính phương.


 x  y  3; x ; y  ?
Đặt x  a; y  b; x  y  3  c a , b , c  ?

 a b  c 1

 
a  b  c  1
 x  y  a 2  b 2  
  2

  2 2
c  a  b  3
 x  y  3  c 2 


2
 a  b  1  a 2  b2  3  2 a  2b 2 ab  3
 
a  2  x  4
b  3  y  9
 a  1b  1  2   
 
a  3 x  9
 
 
b  2  y  4
Tài liệu sưu tầm và tổng hợp bản word đầy đủ liên hệ 0393732038 TÀI LIỆU TOÁN HỌC
73
Website:tailieumontoan.com

Nhận xét: Bài toán số học sử dụng tính chất số chính phương, phương trình ước số.
Nhắc lại kiến thức và phương pháp:
• Khi a là số nguyên ta có được a là số chính phương.
Ta có x; y; x + y + 3 là các số nguyên nên ta có thể biển diễn như sau
 x = a ⇒ x = a2

 y =b ⇒ y =b
2
với a , b , c là các số nguyên dương.
 2
 x + y + 3 = c ⇒ x + y + 3 = c
Từ đây ta có

 a  b  c 1
 
 a  b  c  1  a  b  12  a 2  b2  3 .

x  y  a 2  b 2  

  2 2 2

 x  y  3  c 2 c  a  b  3



• Hằng đẳng thức ( x + y − z ) = x 2 + y 2 + z 2 + 2 xy − 2 xz − 2 yz
2

( a + b − 1)
2
− a 2 − b2 =3 ⇔ a 2 + b2 + 1 + 2 ab − 2 a − 2b − a 2 − b2 =3
⇔ 2 ab − 2 a − 2b =2 ⇔ ab − a − b =1 ⇔ a ( b − 1) − ( b − 1) =2
⇔ ( a − 1)( b − 1) =
2.
• Phương trình ước số ( a − 1)( b − 1) =2 =2.1 =1.2 =( −2 ) . ( −1) =( −1) . ( −2 ) .
Vì a; b ≥ 0 nên ( a − 1) ; ( b − 1) ≥ −1 .
=a − 1  a 3 =
2 = x 9
  
= b −1 1 = b 2 = y 4
Ta có   
⇔ ⇒  .
a − 1
= 1 = a 2 = x 4
  
b − 1
=  b 3 =
2 =   y 9
Vậy ( x; y ) ∈ ( 9; 4 ) , ( 4; 9 ) .
x y z
3). Ta có P   
y  z4 z x4 x y4
4x 4y 4z
P  
4 y  z4 4 z x4 4 x y4
4x 4y 4z
  
y  z44 x z44 x y44
 x y z 
 4     6 .
 y  z x  z x  y 
x  4
Dấu = xảy ra khi x  y  z  4   .
 y  9
Nhận xét: Bài toán sử dụng kỹ thuật chọn điểm rơi, kết hợp với bất đẳng thức Cosi đưa về
được một bất đẳng thức quen thuộc gọi là bất đẳng thức Nesbitt, bất đẳng thức có tới 45
cách chứng minh từ đó tìm ra giá trị nhỏ nhất của biểu thức đã cho.
Nhắc lại kiến thức và phương pháp:
• Bất đẳng thức Cosi cho hai số thực dương a  b  2 ab .

Tài liệu sưu tầm và tổng hợp bản word đầy đủ liên hệ 0393732038 TÀI LIỆU TOÁN HỌC
74
Website:tailieumontoan.com

• Bất đẳng thức Cosi cho ba số thực dương a  b  c  3 3 abc .


• Bất đẳng thức Nesbitt cho ba số thực dương a; b; c là
a b c 3
   .
bc ca ab 2
Chứng minh: Bất đẳng thức đã cho tương đương với:
 a   b   c  9
  1    1    1 
 b  c   c  a   a  b  2
abc bca cab 9
   
bc ca ab 2
 1 1 1  9
 a  b  c   
 a  b b  c c  a  2
 1 1 1 
 a  b  b  c  c  a     9 .
  a  b b  c c  a 
Áp dụng bất đẳng thức Cosi cho ba số thực dương, ta có:
a  b  b  c  c  a  3 3 a  bb  cc  a
1 1 1 3
  
ab bc ca 3 a  bb  cc  a
Nhân hai bất đẳng thức trên với nhau suy ra điều phải chứng minh.
Ý tưởng: Bài toán là một bất đẳng thức đối xứng, vai trò các biến x; y; z là như nhau,
không khó để thấy nếu P  m , dấu đẳng thức xảy ra khi và chỉ khi x  y  z  k . Với giá trị
x  y  z  k thay ngược lại P , ta có:
3k
P  m  9 k 2  2 m2 k  4 m2  0 (*).
2k  4
Để bất phương trình (*) có nghiệm với mọi m , k thì (*)  0 khi đó ta được
m4  36 m2  0  m2 m2  36  0  m2  36  m  6 . Do đó ta tìm được P  6 và dấu đẳng
thức xảy ra khi k  4  x  y  z  4 . Bây giờ, quan sát biểu thức P , chứa ba phân thức
đồng thời căn thức xuất hiện ở mẫu số mỗi phân thức, bằng cách nào đó ta sẽ đánh giá
khử căn bậc hai. Với điểm rơi đã tìm được là x  y  z  4 , ta thấy y  z  4  4 , vì vậy đã
khử căn bậc hai ta áp dụng bất đẳng thức Cosi như sau:
yz
 y  z  4  4  2 4. y  z  4  4 y  z  4  y  z  4  .
4
Tương tự cho các biểu thức còn lại, ta suy ra:
 x y z  x y z 3
P  4      6     (*).
 y  z z  x x  y  yz zx xy 2
Và nếu chứng minh được bất đẳng thức (*) bài toán sẽ được hoàn thành. Nhận thấy nếu
lấy tử cộng mẫu trong từng phân thức của (*) ta sẽ được đại lượng x  y  z chung, chính
vì thế, mỗi phân thức ta sẽ cộng thêm 1 khi đó, ta có (*)
xyz xyz xyz 9
   
yz zx xy 2
 1 1 1  9
  x  y  z   
 x  y y  z z  x  2
Tài liệu sưu tầm và tổng hợp bản word đầy đủ liên hệ 0393732038 TÀI LIỆU TOÁN HỌC
75
Website:tailieumontoan.com
 1 1 1 
  x  y   y  z   z  x     9 .
   x  y y  z z  x 
Đặt a  x  y; b  y  z; c  z  x ta có:
1 1 1 1
a  b  c    3 3 abc .3 3  9  (*) luôn đúng (điều phải chứng minh).
a b c abc
Bài toán kết thúc.
Bài tập tương tự:
1. Cho x; y; z là các số thực dương lớn hơn 1 . Tìm giá trị nhỏ nhất của biểu thức
x y z
P   .
yz2 z x2 x y2
2. Cho x; y; z là các số thực dương lớn hơn 3 . Tìm giá trị nhỏ nhất của biểu thức
2x y z
P   .
y  z6 z  2x  6 2x  y  6

Câu 3.

1). Gọi P là điểm đối xứng của A qua M


 HP  HM  MB  2 HM  AH  HN
 H là trung điểm của NP .
Mà BH  NP , suy ra tam giác PNB cân tại B  BN  BP .
Mặc khác lại có M là trung điểm của BC ; AP . Do đó tứ giác ACPB là hình bình hành, suy
ra AC  BP  AC  BN .
Nhận xét: Chứng minh hai đoạn thẳng bằng nhau ta chứng minh cho chúng cùng bằng
một đoạn thẳng thứ ba
Nhắc lại kiến thức và phương pháp:
• Hai điểm đối xứng nhau qua một điểm thì điểm đó là trung điểm của đoạn thẳng nối
hai điểm đã cho.
Tài liệu sưu tầm và tổng hợp bản word đầy đủ liên hệ 0393732038 TÀI LIỆU TOÁN HỌC
76
Website:tailieumontoan.com
P là điểm đối xứng với A qua M nên M là trung điểm của AP do đó ta có
PM = AM ⇔ PM + HM = AM + HM
⇔ PH = AH + HM + HM = AH + 2 HM = AH + AN = HN
⇒ H là trung điểm của PN .
• Tam giác có đường trung tuyến cũng là đường cao thì tam giác đó là tam giác cân.
Tam giác ∆PNB có BH vừa là đường trung tuyến H là trung điểm của PN ) vừa là
đường cao ( BH ⊥ NP ) nên ∆PNB cân tại B
• Tam giác cân có hai cạnh bên bằng nhau.
Tam giác ∆PNB cân tại B nên BP = BN .
• Tứ giác có hai đường chéo cắt nhau tại trung điểm mỗi đường là hình bình hành.
Tứ giác ACPB có M vừa là trung điểm của BC vừa là trung điểm của AP nên ACPB
là hình bình hành.
• Hình bình hành có các cạnh đối diện bằng nhau.
 AC = PB
ACPB là hình bình hành nên AC = PB , suy ra  BN (điều phải chứng
⇒ AC =
 BP = BN
minh).
  APB
2). Do tứ giác ACPB là hình bình hành, suy ra PAC .
  ANB
Mà tam giác PBN cân tại B  APB   ANB
  PAC

  BNQ
 CAN 

Ta có AC  NB; NQ  AN
  NCD
BNQ CAN  NBD   N ; B; C ; D cùng thuộc một đường tròn C ; G là giao điểm
  BQG
.
   
DQG với DBC , suy ra CAG

 GBQ ∽GCA  GA  GQ  GA  GQ
  GCA
Mà GBQ .
AC QB NB NC
  BDC
Mà BNC  NBC ∽GAQ
  AGQ
  NCB
 GQA   GDC
  NCB   GC  NB  NG  BC .
Nhận xét: Chứng minh hai đoạn thẳng bằng nhau ta chứng minh cho chúng cùng bằng
một đoạn thẳng thứ ba.
Nhắc lại kiến thức và phương pháp:
• Hình bình hành có hai cạnh đối diện song song.
  APB
ACPB là hình bình hành nên AC  PB suy ra PAC  (hai góc so le trong).
• Tam giác cân có hai góc ở đáy bằng nhau.
 = ANB
Tam giác PBN cân tại B nên APB  , suy ra PAC = ANB
 hay CAN = BNQ
.

Có AC  NB (chứng minh phần 1).


NQ  AN ( Q đối xứng với A qua N ).
• Hai tam giác có một góc bằng nhau và hai cạnh kề góc tương tứng bằng nhau thì bằng
nhau theo trường hợp “cạnh - góc - cạnh” (c – g – c).
Xét ∆CAN và ∆BNQ có:
+ CA = NB ;
 = BNQ
+ CAN ;

Tài liệu sưu tầm và tổng hợp bản word đầy đủ liên hệ 0393732038 TÀI LIỆU TOÁN HỌC
77
Website:tailieumontoan.com
  BQG
+ AN = NQ ; CAG 

Suy ra ∆CAN =    = NBD


∆BNQ (c – g – c), suy ra ACN = NBQ hay DCN .

• Tứ giác có hai đỉnh liên tiếp cùng nhìn một cạnh dưới hai góc bằng nhau là tứ giác nội
tiếp.
 = NBD
Tứ giác NBCD có hai đỉnh B; C liên tiếp cùng nhìn cạnh DN với hai góc DCN 

nên NBCD là tứ giác nội tiếp hay N ; B; C ; D cùng thuộc một đường tròn.
  BQG
3). Ta có CAG .

 GBQ ∽GCA  GA  GQ  GA  GQ ;
  GCA
Mà GBQ
AC QB NB NC
  BDC
và BNC   AGQ
 NBC ∽GAQ ;
  NCB
 GQA   NCB   GC  NB  NG  BC .
  GDC
Nhận xét: Chứng minh hai đoạn thẳng bằng nhau ta chứng minh cho chúng cùng bằng
một đoạn thẳng thứ ba
Nhắc lại kiến thức và phương pháp:
• Tứ giác nội tiếp có góc trong bằng góc ngoài tại đỉnh đối diện.
  BQG
DNGB là tứ giác nội tiếp nên CAG .

• Hai góc nội tiếp cùng chắn một cung thì bằng nhau.
  GCA
GBQ  (hai góc nội tiếp cùng chắn cung DG
 của đường tròn ngoại tiếp tứ giác

BCGD ).
• Hai tam giác có hai cặp góc bằng nhau thì đồng dạng theo trường hợp “góc - góc” (g –
g); Tam giác đồng dạng có các cặp cặp tương ứng tỷ lệ và các góc tương ứng bằng
nhau.
Xét GBQ và GCA có:
  BQG
+ CAG ;
  GCA
+ GBQ ;

GA GQ GA GQ
Suy ra GBQ ∽GCA (g – g), suy ra    .
AC QB NB NC
• Hai tam giác có hai góc bằng nhau và hai cặp cạnh kề góc tương ứng tỷ lệ thì đồng
dạng theo trường hợp “cạnh - góc - cạnh” (c – g – c); Tam giác đồng dạng có các cặp
cặp tương ứng tỷ lệ và các góc tương ứng bằng nhau.
Xét NBC và GAQ có:
GA GQ
+  ;
NB NC
  AGQ
+ BNC   BDC

;  
  NCB
Suy ra NBC ∽GAQ (c – g – c), suy ra GQA ,
  GDC
 NCB   GC  NB  NG  BC .

Câu 4. Giả sử trên mặt phẳng có n điểm thẳng hàng thì tồn tại một đường thẳng.Theo bài
ra các điểm đã cho không cùng nằm trên một đường thẳng nên tồn tại ít nhất một điểm
không cùng nằm trên đường thẳng đó nối điểm đó với n  1 điểm đã cho ta được n  1

Tài liệu sưu tầm và tổng hợp bản word đầy đủ liên hệ 0393732038 TÀI LIỆU TOÁN HỌC
78
Website:tailieumontoan.com
đường thẳng với đường thẳng đi qua n  1 điểm ta được n đường thẳng, thay n  2015 thì
tồn tại ít nhất 2015 đường thẳng.
Nhận xét: Bài toàn tư duy từ việc lập đường thẳng đi qua các điểm kết hợp với tính toán
số đường thẳng được tạo thành.
Nhắc lại kiến thức và phương pháp:
• Với n điểm thẳng hàng luôn tồn tại một đường thẳng duy nhất đi qua n điểm đó.
• Với n 1 điểm thẳng hàng và 1 điểm nằm ngoài đường thẳng đi qua n 1 điểm đó.
Khi đó từ 1 điểm đó ta kẻ được n 1 đường thẳng tới n 1 điểm. Trường hợp này ta
có n đường thẳng được tạo thành. Đây là trường hợp có ít đường thẳng được tạo ra
nhất.
Thay n  2015 ta có 2015 đường thẳng phân biệt được tạo thành.

Đề số 11

Câu 1. 1) Điều kiện: −1 ≤ x ≤ 1.

( )
2
Ta có: 1+ x + 1− x = 2 + 2 1 − x2 .

Do đó phương trình đã cho tương đương với

( )
3
1+ x + 1− x = 8 ⇔ 1+ x + 1− x = 2

⇔ 2 + 2 1 − x 2 =4 ⇔ 1 − x 2 =1 ⇔ x 2 =0
⇔ x = 0 ∈ [ −1,1] .

Vậy phương trình đã cho có nghiệm duy nhất x = 0.

2) Từ HPT ra ta có:
4 ( x 2 − xy + y 2 ) =x 2 + xy + 2 y 2 ⇔ 3 x 2 − 5 xy + 3 y 2 =0 ⇔ ( x − y )( 3 x − 2 y ) =0.

Với x – y = 0 hay x = y, thay vào PT thứ nhất của HPT ta có x 2 − x 2 + x 2 =⇔


1 x=±1.

3
Với 3x – 2y = 0 hay y =
x , thay vào PT thứ nhất của HPT ta có:
2
3 9 2 3
x 2 − x 2 + x 2 =⇔
1 x= ± ⇒y= ± .
2 4 7 7

 2 3   2 3 
Vậy HPT có bốn nghiệm (x; y) là: (1;1) , ( −1; −1) ,  ; , − ;− .
 7 7  7 7

1 1 1
Câu 2. 1) Đặt=a = ,b = ,c .
x y z

Từ giả thiết ta có ab + bc + ca = 1. Do đó, đẳng thức cần chứng minh tương đương với:

Tài liệu sưu tầm và tổng hợp bản word đầy đủ liên hệ 0393732038 TÀI LIỆU TOÁN HỌC
79
Website:tailieumontoan.com
a 2b 3c 5bc + 4ca + 3ab
+ + =
1+ a 1+ b 1+ c
2 2 2
( a + b )( b + c )( c + a )
a 2b 3c 5bc + 4ca + 3ab
⇔ + + =
( a + b )( a + c ) ( b + c )( b + a ) ( c + a )( c + b ) ( a + b )( b + c )( c + a )

⇔ a ( b + c ) + 2b ( c + a ) + 3c ( a + b ) 5bc + 4ca + 3ab (luôn đúng)


Vậy đẳng thức được chứng minh.
2) Đặt u = x + y, v = xy phương trình đã cho trở thành:
v+3
v2u + u = 3 + v ⇔ u = 2 .
v +1
Do x; y ∈ Z + nên u; v ∈ Z+ suy ra v 2 + 1 là ước của v + 3 ⇒ v 2 + 1 là ước của v 2 − 9 ⇒ v 2 + 1 là
ước của v 2 + 1 − 10 ⇒ v 2 + 1 là ước của 10.

Lần lượt xét các ước nguyên của 10 để xác định v, u rồi tìm nghiệm nguyên dương x, y
tương ứng, thử lại với PT ban đầu.

Phương trình có ba nghiệm nguyên dương (x; y) là: (0; 3), (3; 0), (1; 1).

Câu 3.

1) Ta có ∆ABF và ∆ACE đồng dạng do chúng lần lượt cân tại F, E và


∠FBA = ∠BAD = ∠DAC = ∠ECA.

GF BF AB DB
2) Gọi G là giao điểm của BE và CF. Ta có: = = = ⇒ DG / / BF . Mặt khác,
GC CE AC DC
DA//BF suy ra A, D, G thẳng hàng, suy ra đpcm.

3) Ta có ∠BQG = ∠QGA = ∠GAE = ∠GAC + ∠CAE = ∠GAB + ∠BAF = ∠GAF suy ra AGQF
là tứ giác nội tiếp. Mặt khác, ∠QPG = ∠GFQ nên QGPF là tứ giác nội tiếp.
∠GCE =

Tài liệu sưu tầm và tổng hợp bản word đầy đủ liên hệ 0393732038 TÀI LIỆU TOÁN HỌC
80
Website:tailieumontoan.com
Suy ra đpcm

Câu 4. Áp dụng BĐT Cauchy ta có:

1 1
a 4 b 2 + abc 2 + ca ≥ a 2 bc;
3 9
1 1
b 4 c 2 + bca 2 + ab ≥ b 2 ca;
3 9
1 1
c 4 a 2 + cab 2 + bc ≥ c 2 ab.
3 9

Cộng theo từng vế của ba BĐT trên thay ab + bc + ca = 1 vào và rút gọn ta được:

2 1
abc ( a + b + c ) ≤ a 4 b 2 + b 4 c 2 + c 4 a 2 + . (1)
3 9
Ta có
1 1 4 4
abc ( a + b + c ) = ab.ca + bc.ab + ca.bc ≤ ( ab + bc + ca ) = ⇒ abc ( a + b + c ) ≤ . ( 2)
2

3 3 3 9

Cộng theo từng vế (1) và (2) ta có đpcm.

3
Đẳng thức xảy ra ⇔ a = b = c = .
3

Đề số 12

Câu 1.
1). Dễ thấy đẳng thức sau đúng với a  b .
b b 2 b 2 b b 2b 2
  , suy ra   .
a  b a  b a2  b2 a  b a  b a2  b2
Do đó đẳng thức đã cho tương đương với
y 2y2  y2 2 y 4   y4 2 y 8  8 y8
 
  2     4  4    4
x  y x2  y 2  x 2  y 2 x 4  y 4   x  y 4 x8  y 8  x8  y 8
y
  4  y  4 x  4 y  5 y  4 x , điều phải chứng minh.
xy
Nhận xét: Bài toán sử dụng đẳng thức (bổ đề luôn đúng) để ghép vào biểu thức đã cho để
chứng minh.
Ý tưởng: Trước hết, xét ở đẳng thức cần chứng minh ta có 5 y  4 x vì thế ta sẽ tách thành
y
y  4x  4 y   4 . Việc tách này là có cơ sở vì số 4 xuất hiện ở vế phải của giả thiết,
xy
y
cũng như các mối liên hệ giữa mẫu số các phân thức ở vế trái. Với 4  thế ngược lại
xy
y 2y2 4y4 8 y8 y
giả thiết của bài toán, tức là ta sẽ cần chứng minh  2   
xy x y 2 4
x y 4 8
x y 8
xy
(*).

Tài liệu sưu tầm và tổng hợp bản word đầy đủ liên hệ 0393732038 TÀI LIỆU TOÁN HỌC
81
Website:tailieumontoan.com
Đẳng thức (*) hoàn toàn có thể chứng minh bằng phương pháp biến đổi tương đương đó
y y 2y2 4y4 8 y8
là:   2   0
x  y x  y x  y 2 x 4  y 4 x8  y 8
2y2 2y2 4y4 8 y8
    0
x2  y 2 x2  y 2 x4  y 4 x8  y 8
4y4 4y4 8 y8 8 y8 8 y8
    0    0 .
x4  y 4 x4  y 4 x8  y 8 x8  y 8 x8  y 8
Với cách biến đổi trên, để làm xuôi ngược lại thì ta đã sử dụng một bổ đề đẳng thức rất
b b 2 b 2
đẹp đó là   2 .
a  b a  b a  b2
Bài toán kết thúc.


 x  y2 x  3 y  12
2). Hệ đã cho tương đương với  ,
 


6 x  y  xy  x  y  12
x  y  0
suy ra  x  y2 x  3 y   x  y6  xy  
 2 x  3 y  6  xy. (loại)
x  3
Ta có 2 x  3 y  6  xy   x  3 y  2  0   .
 y  2
+ Với x  3 , thay vào phương trình đầu của hệ ta có
 y  1
18  3 y 2  3 y  12   .
y  2

+ Với y  2 , thay vào phương trình đầu của hệ ta có
x  3
2 x 2  2 x  12  12   .
 x  4

Vậy hệ có nghiệm  x; y  3;  1 , 3; 2 , 4; 2 .
Nhận xét: Bài toán sử dụng phương pháp thế hằng số ở cả hai phương trình, sau đó
phương trình thu được phân tích thành nhân tử và thế ngược lại một trong hai phương
trình của hệ tìm nghiệm của hệ phương trình.
Ý tưởng: Thoạt nhìn, ta sẽ nghĩ đến hướng xét delta ẩn x hoặc y ở phương trình thứ hai
của hệ và mong muốn đenta chính phương. Nhưng hướng đi này sẽ thất bại, vì dễ thấy
cũng từ phương trình hai, ta tách được rằng 6  x  y  xy  x  y  12   x  y xy  6  12 .
Mặt khác, xét vế trái của phương trình một nếu coi đây là một phương trình đẳng cấp bậc
hai, ta sẽ có được 2 x 2  3 y 2  xy   x  y2 x  3 y . Khi đó hệ phương trình đã cho tương

 x  y2 x  3 y  12
đương với  .



 x  y xy  6  12
Đây là một hệ rất đẹp vì nhân tử x  y; 12 đều xuất hiện ở cả hai phương trình, chính vì
thế suy ra:
x  y
 x  y2 x  3 y   x  y xy  6  
 2 x  3 y  xy  6 (*)

Tài liệu sưu tầm và tổng hợp bản word đầy đủ liên hệ 0393732038 TÀI LIỆU TOÁN HỌC
82
Website:tailieumontoan.com
x  3
Với (*), dễ thấy nhân tử như sau: (*)   x  3 y  2  0   .
y  2
Việc còn lại là thế ngược lại tìm nghiệm của hệ phương trình.
Bài toán kết thúc.

Câu 2.
1). Do x; y là các số nguyên lớn hơn 1 nên x; y  2
 4 xy  1  7 x  7 y  4 xy  1
 4 x 2 y 2  4 xy  1  4 x 2 y 2  7 x  7 y  4 x 2 y 2  4 xy  1
2 2
 2 xy  1  4 x 2 y 2  7 x  7 y  2 xy  1 .

Mà 4 x 2 y 2  7 x  7 y là số chính phương và 1  2 xy  1  2 xy  1 ;
2
nên ta có 4 x 2 y 2  7 x  7 y  2 xy  x  y , điều phải chứng minh.
Nhận xét. Bài toán chứng minh đẳng thức từ những điều kiện đã cho.
Nhắc lại kiến thức và phương pháp.
• Xét x  y
x  y  0 7 x  7 y  0
+ Với x  y ta có    7 x  7 y  4 xy  1 .
xy  0 4 xy  1  0
+ Với x; y  2 ta có 4 xy  1  8 x  1  7 x  x  1  7 x  y  1  7 x  7 y
Suy ra 4 xy  1  7 x  7 y  4 xy  1
 4 x 2 y 2  4 xy  1  4 x 2 y 2  7 x  7 y  4 x 2 y 2  4 xy  1
2 2
 2 xy  1  4 x 2 y 2  7 x  7 y  2 xy  1
+ Bình phương của hai số nguyên có giá trị tuyệt đối là hai số tự nhiên liên tiếp được
2 2
gọi là hai số chính phương liên tiếp. Giữa n 1 và n  1 có duy nhất số chính
phương n2 với n là số nguyên.
2 2 2
+ Ta có giữa 2 xy  1 và 2 xy  1 có duy nhất số chính phương 2xy mà
2 2
2 xy  1  4 x 2 y 2  7 x  7 y  2 xy  1 suy ra
2
4 x 2 y 2  7 x  7 y  2 xy  4 x 2 y 2  7 x  7 y  4 x 2 y 2
 7 x  7 y  0  x  y (trái với điều kiện x  y )
• Xét y  x
Chứng minh hoàn toàn tương tự như trên, ta được x  y (trái với điều kiện y  x ).
Mối quan hệ giữa các số nguyên a  b hoặc a  b hoặc a  b .
Ta có x  y và y  x đều không tồn tại nên chỉ có x  y (điều phải chứng minh).

2). Ta có x 3  y 3  x 2  y 2  xy
  
  x  y x 2  y 2  xy  x 2  y 2  xy   x  y  1 x 2  y 2  xy  0 

Tài liệu sưu tầm và tổng hợp bản word đầy đủ liên hệ 0393732038 TÀI LIỆU TOÁN HỌC
83
Website:tailieumontoan.com
 x 2  y 2  xy  0  x  y  0
  
x  y  1 .
 x  y  1  0 
5
+ Với x  y  0  P  .
2
+ Với x  y  1  0  x; y  1 ,
1 1 2 1
suy ra P    4 , Dấu “=” xảy ra khi và chỉ khi x  1; y  0 .
2 0 1 0
1 0 2 0 4
P   , Dấu “=” xảy ra khi và chỉ khi x  0; y  1 .
2  1 1 1 3
4
Vậy Pmax  4  x  1; y  0 và Pm in   x  0; y  1 .
3
Nhận xét: Khai thác giả thiết cũng như biểu thức bài cho, tìm điều kiện chặn của biến để
tìm giá trị nhỏ nhất, giá trị lớn nhất của biểu thức.
Nhắc lại kiến thức và phương pháp:
• Hằng đẳng thức a3  b3  a  ba2  ab  b2 
m  p f  x
• Xét biểu thức P  , với 0  a  f  x  b , ta có:
n  q f  x
mp a mp a mp b mp b
P  Pmin  ; P  Pmax  .
nq b nq b nq a nq a
Ý tưởng: Đi từ giả thiết của bài toán, sự xuất hiện của x 3  y 3 làm ta nghĩ đến hằng đẳng
thức x 3  y 3   x  y x 2  xy  y 2  , khi đó giả thiết trở thành:
x 3  y 3  x 2  xy  y 2   x  y x 2  xy  y 2   x 2  xy  y 2
x  y  0
  x  y  1 x 2  xy  y 2   0  
x  y  1

Ta chỉ cần xét với x  y  1 , mặt khác kết hợp với điều kiện ở biểu thức ta có được điều

0  x  1
kiện chặn của x , y là 0  x , y  1   .

0  y  1


Suy ra
1 
 x 1 
 x 2
 2  1 x 2 x 4
•  ; P   .
 y 3  y 2 2 y 1 y 3
2 

 1 



 2 
x 2  x 3
1   1 x 2 x
•  ; P  4 .
 y 2  y 1 2 y 1 y
2 

 1 


4
Do đó Pmax  4  x  1; y  0 và Pm in   x  0; y  1 .
3
Bài toán kết thúc.

Câu 3.

Tài liệu sưu tầm và tổng hợp bản word đầy đủ liên hệ 0393732038 TÀI LIỆU TOÁN HỌC
84
Website:tailieumontoan.com

F O
E
P

B D C

K
  AEB
1). Ta có AFC   ADC
  ADB
  180 suy ra tứ giác AEPF nội tiếp.
Nhận xét. Để chứng minh bốn điểm cùng thuộc một đường tròn ta đưa về chứng minh tứ
giác AEPF bằng cách chỉ ra tứ giác này có tổng hai góc đối diện bằng 180 .
Nhắc lại kiến thức và phương pháp.
• Các góc nội tiếp cùng chắn một cung của một đường tròn thì bằng nhau.
  ADC
+ AFC  (hai góc nội tiếp cùng chắn cung AC của đường tròn ngoại tiếp tam
giác ACD ).
  ADB
+ AEB  (hai góc nội tiếp cùng chắn cung AB của đường tròn ngoại tiếp tam giác
ABD ).
• Tứ giác có tổng hai góc đối diện bằng 180 là tứ giác nội tiếp.
  ADB
ADC   180 (hai góc kề bù) nên AFC   AEB
  180 , suy ra tứ giác AEPF là tứ
giác nội tiếp hay bốn điểm A ; E ; P ; F cùng thuộc một đường tròn.

 = LFC
2). Từ tứ giác AEPF nội tiếp, suy ra AEB  (1).
 = FCB
Ta lại có FCL   BAQ
 = PBC
  BCL  = DAE
  BAQ = BAE
 (2).

Từ (1) và (2), suy ra FCL ∽ EAB .


Nhận xét. Nhớ lại các trường hợp đồng dạng của hai tam giác kết hợp với dữ kiện đề bài
cùng các ý đã chứng minh được từ ý trên để tìm ra hướng chứng minh của bài toán. Đối
với bài toán này, ta chứng minh hai tam giác đồng dạng theo trường hợp “góc - góc”
Nhắc lại kiến thức và phương pháp.
• Tứ giác nội tiếp có góc trong bằng góc ngoài tại đỉnh đối diện.
  LFC
Tứ giác AEPF là tứ giác nội tiếp (chứng minh trên) nên AEB 
• Các góc nội tiếp cùng chắn một cung của một đường tròn thì bằng nhau.

Tài liệu sưu tầm và tổng hợp bản word đầy đủ liên hệ 0393732038 TÀI LIỆU TOÁN HỌC
85
Website:tailieumontoan.com
  BAQ
+ BCL  (hai góc nội tiếp cùng chắn cung BQ của đường tròn (O) ).
  EBC
+ EAD  (hai góc nội tiếp cùng chắn cung DE của đường tròn ngoại tiếp tam giác
ABD ).
• Tam giác cân có hai góc kề đáy bằng nhau.
  PCB
Tam giác PBC có PB  PC nên tam giác PBC cân tại P suy ra PBC .
  FCB
Kết hợp lại, ta được BCL   ABQ
  QAE
  FCL  BAE
.

• Hai tam giác có hai góc tương ứng bằng nhau thì đồng dạng.
Xét FCL và EAB có: AEB   LFC (chứng minh trên) và FCL
  BAE
 (chứng minh
trên), suy ra FCL ∽ EAB (g – g).
FL FC
3). Từ FCL ∽ EAB , suy ra = hay FL.EA = FC.EB (3).
BE AE
Chứng minh tương tự EK.FA = FC.EB (4).
FL EK
Từ (3) và (4), suy ra FL.EA = EK.FA hay = , suy ra EF  KL .
FA EA
Ta lại có  = ALK
QLK   ALQ
  AFE  ABE
  APE
  ABE
  PAB  .
  PAC
Tương tự ta có QKL .
  PAB
Suy ra QKL   QLK
  PAC
.

Nhận xét. Với các bài toán chứng minh hai tổng bằng nhau, ta dựa vào mối quan hệ giữa
các góc để đưa các vế cùng bằng một lượng nào đó.
Nhắc lại kiến thức và phương pháp.
• Tính chất hai tam giác đồng dạng .
 FL FC
 =  FL. AE  FC.EB
FCL ∽ EAB   BE AE .
  
FLC  ABE
Chứng minh hoàn toàn tương tự ta có EK.FA = FC.EB
• Định lý Ta-lét đảo.
FL EK
Từ chứng minh trên, ta được FL.AE  EK.FA  = , suy ra EF  KL .
FA EA
• Đường thẳng cắt hai đường thẳng song song tạo ra các góc đồng vị bằng nhau.
  ALK
EF  KL (chứng minh trên), suy ra AFE .

• Các góc nội tiếp cùng chắn một cung của một đường tròn thì bằng nhau.
  APE
AFE  (hai góc nội tiếp cùng chắn cung AE của đường tròn ngoại tiếp tứ giác
AFPE ).
Kết hợp các ý chứng minh trên ta có được điều đã trình bày trong bài giải trên
 = ALK
QLK   ALQ   ABE
  AFE   APE  ABE  PAB
 . Hoàn toàn chứng minh tương tự ta
  PAC
có được QKL  . Từ đó ta có điều cần chứng minh QKL
  PAB
  QLK
  PAC
.

Câu 4.
Từ giả thiết dễ thấy m tập con thuộc dãy là phân biệt. Vì A có 31 phần tử nên số tập con
31  30
có đúng 2 phần tử của A là  Ký hiệu ak 2  k  31 là số các tập có đúng k phần
2

Tài liệu sưu tầm và tổng hợp bản word đầy đủ liên hệ 0393732038 TÀI LIỆU TOÁN HỌC
86
Website:tailieumontoan.com
tử, nằm trong dãy đã cho, suy ra m  a2  a3    a31 . Xét một tập hợp có k phần tử suy
k  k  1 k  k  1
ra số các tập con có 2 phần tử của tập đó là  ak tập này sẽ có ak tập con 2
2 2
phần tử. Mà theo giả thiết với 2 phần tử bất kỳ của A thì chúng không thể đồng thời
thuộc 2 tập có k phần tử của dãy  các tập con 2 phần tử nói trên là phân biệt.
k  k  1 31  30 1
Suy ra ak   ak  31  30 
2 2 k  k  1
 1 1 1 
 a2  a3    a31  31.30    
 1.2 2.3 30.31
 1 1 1 1 1
 m  31.30 1        .
 2 2 3 30 31
Vậy m  900, điều phải chứng minh.
Nhận xét. Bài toán về phần nguyên
Nhắc lại kiến thức và phương pháp.
• Bài toán cách chọn số từ một tập hợp số: “Trong n số bất kỳ ta chọn n cách chọn số thứ
n n 1
nhất, n 1 cách chọn số thứ hai, khi đó có cách chọn hai số khác nhau từ n số
2
đã cho”.
Từ giả thiết dễ thấy m tập con thuộc dãy là phân biệt. Vì A có 31 phần tử nên số tập
31  30
con có đúng 2 phần tử của A là  Ký hiệu ak 2  k  31 là số các tập có đúng k
2
phần tử, nằm trong dãy đã cho, suy ra m  a2  a3    a31 . Xét một tập hợp có k phần
k  k  1 k  k  1
tử suy ra số các tập con có 2 phần tử của tập đó là  ak tập này sẽ có ak
2 2
tập con 2 phần tử. Mà theo giả thiết với 2 phần tử bất kỳ của A thì chúng không thể
đồng thời thuộc 2 tập có k phần tử của dãy  các tập con 2 phần tử nói trên là phân
biệt.
• Phân số có mẫu là tích hai số tự nhiên liên tiếp: “Với x là số tự nhiên thỏa mãn điều
1 1 1
kiện xác định ta có   ”
x  x  1 x x  1
k  k  1 31  30 1
ak   ak  31  30 
2 2 k  k  1
 1 1 1 
 a2  a3    a31  31.30    
 1.2 2.3 30.31
 1 1 1 1 1  1
 m  31.30 1          31.30.1    900 .
 2 2 3 30 31  31

Tài liệu sưu tầm và tổng hợp bản word đầy đủ liên hệ 0393732038 TÀI LIỆU TOÁN HỌC
87
Website:tailieumontoan.com

Đề số 13

Câu 1.
1
1). Điều kiện:   x  2 .
3
Phương trình đã cho tương đương với 2 x  3  2 3x  12  x  9

3  x  0
 3 x 2  5 x  2  3  x  

 2 2
3 x  5 x  2  x  6 x  9

x  1

2
 4 x  11x  7  0   .
x  7
 4
7
Đối chiếu với điều kiện ta được được nghiệm: x  1; x  .
4
Nhận xét: Bài toán sử dụng phương pháp nâng lũy thừa (bình phương) hai vế tìm nghiệm
của phương trình.
Nhắc lại kiến thức và phương pháp:

 
2
• Phương trình dạng f  x  g  x  m  0  f  x  g  x  m 2

 f  x   g  x  2 f  x g  x  m 2  2 f  x g  x  m 2  f  x  g  x

 m 2  f  x  g  x  x  x1


   .
  
4 f  x g  x   m  f  x  g  x
2
2 x  x


    2

• Phương trình trên có cách giải khác như sau:


f  x  g  x  m  f  x  m  g  x
m  f  x m  f  x
   
 f x  m2  2 m g x  g x

      2m g  x  g  x  m2  f  x

m  f  x ; g  x  m 2  f  x

  x  x1
   .
 
 x  x2
2
4 m2 g  x   g  x  m2  f  x


Ý tưởng: Đây là một bài phương trình cơ bản, dạng toán một vế chứa hai căn thức vế còn
lại là một hằng số thì phương pháp nâng lũy thừa hai vế là một phương pháp tối ưu nhất.
Bài toán kết thúc.
Bài tập tương tự:
1. Giải phương trình 3x  1  x  1  8 .
Đáp số: x  8 .
2. Giải phương trình 7 x  4  x  1  3 .
Đáp số: x  3 .

Tài liệu sưu tầm và tổng hợp bản word đầy đủ liên hệ 0393732038 TÀI LIỆU TOÁN HỌC
88
Website:tailieumontoan.com
 1   1  9

 x  y    y  x   2
 
2). Hệ phương trình tương đương với  .
 1 3  1   1  1 
   x     x   y    2
 4 2  y   y  x

 1

u x
 y
Đặt  .

 1

 vy

 x

 9 
 9
u  v  
 v  u

  2
Hệ phương trình trở thành  2 
 1 3

 9 3u 9  .
  u  uv  2 
  
 u  u
  2

4 2
 
4
 2 
9 3u 9u 9
Suy ra    u2  u2  3u   0
4 2 2 4

 1 3
2  3 
 x 
 3 u   y 2
 u    0   2 

 
2  
 1
v  3 
 y 3

 x

 3y  1

 xy  1  3y y 2 y  1  x 
2.
2
 2   3x  x   y   3  y  3 y  2  0  

 2 2 y 
xy  1  3 x
  y  2  x  1
1 
Hệ phương trình có nghiệm  x; y   ; 1 , 1; 2 .
2 
Nhận xét: Bài toán sử dụng phương pháp đặt hai ẩn phụ, đưa về hệ phương trình bậc hai
cơ bản giải bằng phương pháp thế. Sau đó từ nghiệm ẩn phụ suy ngược lại nghiệm của hệ
phương trình.
Ý tưởng: Hình thức bài toán khá phực tạp vì sự xuất hiện của phân thức, quan sát ta thấy
1
ở cả hai phương trình của hệ đều xuất hiện biểu thức x  . Ta sẽ nghĩ đến chuyện thế
y
1 9  1 1
x    y   xuống phương trình hai nhưng còn đại lượng xy  chưa biết xử lý
y 2  x xy
như thế nào. Có lẽ tác giá đã gợi mở theo con đường đặt ẩn phụ, nếu đặt
1 1 1
u  x  ; v  y  thì bây giờ ta chỉ cần biểu diễn xy  qua u; v thì hệ phương trình đã
y x xy

 1

u  x   uy  xy  1
 y
cho sẽ được giải quyết. Ta có 

 1

 v  y   vx  xy  1

 x
2 1
 uvxy   xy  1  uv  xy  2.
xy

Tài liệu sưu tầm và tổng hợp bản word đầy đủ liên hệ 0393732038 TÀI LIỆU TOÁN HỌC
89
Website:tailieumontoan.com

 9

uv 
 2
Khi đó, hệ phương trình đã cho tương đương với  .

 1 3
  u  uv  2

4 2

Hệ phương trình trên là hệ phương trình cơ bản, hoàn toàn giải quyết được bằng phương
pháp thế.
Bài toán kết thúc.
Bài tập tương tự:
x 2 1  y 2   2

1. Giải hệ phương trình  .
 2 2 2
1  xy  x y  3 x


 7 5   7 5 

Đáp số:  x; y  1; 1 , 1;  1 ,  ;  ,  ; .
 4 7   4 7 

 2 2

 x  1  y  1  27 xy
2. Giải hệ phương trình  2 .

 x  1 y 2  1  10 xy


1   1 
2 
 
Đáp số:  x; y   ; 2  3  , 2; 2  3 , 2  3;

.
2 
Câu 2.
1). Cách 1: Đẳng thức cần chứng minh tương đương với
a  b  b  c  c  a  3
1    1    1  
a  b b  c b  c  c  a c  a  a  b  4
ac ba cb 3
   
a  bb  c b  cc  a c  aa  b 4
3
 ac a  c  ba b  a  cb c  b  a  bb  cc  a
4
 ac a  c  ba b  a  cb c  b  6 abc
 ac a  c  b2 a  c  ba 2  abc  c 2 b  abc  8 abc

 a  cac  b2  ab  bc  8 abc

 a  cc  bb  a  8 abc , điều phải chứng minh.


 
x  a 1  x  b
 a  b  ba
 
b c
Cách 2: Đặt  y   1  y  .
 b  c  bc
 
z  c 1  z  a
 c  a  ca

1
Từ điều kiện suy ra xyz  , ta được
8
xyz  1  x1  y1  z  1   x  y  z   xy  yz  zx  xy
 2 xyz  1   x  y  z   xy  yz  zx
3
 xyz  xy  yz  zx .
4

Tài liệu sưu tầm và tổng hợp bản word đầy đủ liên hệ 0393732038 TÀI LIỆU TOÁN HỌC
90
Website:tailieumontoan.com
Nhận xét: bài toán sử dụng phép biến đổi tương đương hoặc ẩn phụ để chứng minh đẳng
thức đã cho.
Ý tưởng: Nhìn đẳng thức cần chứng minh ( gọi là (*)) khá là cồng kềnh, tuy nhiên nếu tinh
ý một chút, ta thấy rằng bên vế trái (*) có tổng của ba thừa số, đồng thời vế phải (*) xuất
hiện tổng hoán vị của tích hai thừa số . Vì thế nếu chuyển vế ta sẽ nhóm được nhân tử
chung là:
a  b  b  c  c  a  3
1  
 1  
 1  
a  b b  c b  c  c  a c  a  a  b  4
ac ba cb 3
    ( i ).
a  bb  c b  cc  a c  aa  b 4
Với biểu thức ( i ), hướng tối ưu nhất có lẽ là quy đồng mẫu số và biến đổi tương đương,
kết hợp với giải thiết a  bb  cc  a  8 abc thì ta có:
3
( i )  ac a  c  ba b  a  cb c  b  a  bb  cc  a
4
 ac a  c  ba b  a  cb c  b  6 abc
 ac a  c  b2 a  c  ba 2  abc  c 2 b  abc  8 abc

 a  cac  b2  ab  bc  8 abc .


Hoặc, ta có thể đi với hướng tư duy ẩn phụ hóa để đơn giản bài toán hơn một chút. Vẫn là
a b c
hướng phát hiện như bên trên, ta sẽ đặt ẩn phụ các thừa số là x  ; y ; z .
ab bc ca
1 3
Khi đó giả thiết  xyz  và ta cần chứng minh x  y  z   xy  yz  zx (**). Nếu chỉ
8 4
dựa vào giả thiết để chứng minh (**) là chưa đủ, ta cần phải khéo léo kết hợp với giải thiết
b c a
bài toán như sau 1  x  ; 1 y  ; 1 z  . Và từ đó suy ra được đẳng thức
ba bc ca
xyz  1  x1  y1  z . Khai triển tích số, ta sẽ được điểu phải chứng minh.
Bài toán kết thúc.

2). Ta có abcde  abc00  de  abc  100  de


 abc 101  1  de  abc  101  abc  de
Suy ra abcde chia hết cho 101  abc  de  abc  10d  e chia hết cho 101 .
99999 9
Ta có 101  m  99999  m   990 
101 101
Vậy số có 5 chữ số lớn nhất chia hết cho 101 là 990  101
999
Ta có 101  n  9999  n   99
101
Vậy số có 5 chữ số nhỏ nhất chia hết cho 101 là 100  101
Số các số có 5 chữ số thỏa mãn yêu cầu của bài toán là: 990  100  1  891.
Đáp số: 891 số.
Nhận xét. Bài toán chứng minh đẳng thức từ những điều kiện đã cho.
Nhắc lại kiến thức và phương pháp.

Tài liệu sưu tầm và tổng hợp bản word đầy đủ liên hệ 0393732038 TÀI LIỆU TOÁN HỌC
91
Website:tailieumontoan.com
• Cấu tạo số
abcde  abc 00  de  abc  100  de  abc 101  1  de  abc  101  abc  de
• Tính chất chia hết của một tích: Trong một tích có một thừa số chia hết cho một số thì
tích chia hết cho số đó.
Ta có 101  101  abc  101  101
• Tính chất chia hết của một tổng: Tổng của hai số hạng, trong đó có một số hạng chia
hết cho một số thì số hạng còn lại chia hết cho số đó.
 
 abc  101  101

Ta có  , suy ra de  abc   101
  
abcde   abc.101  de  abc
  101


 
 abc  10d  e  101 .
• Số lớn nhất có năm chữ số chia hết cho 101.
99999 9
Ta có 101  m  99999  m   990  suy ra số có năm chữ số lớn nhất chia hết
101 101
cho 101 là 990  101 .
• Số bé nhất có năm chữ số chia hết cho 101 .
999
Ta có 101  n  9999  n   99 suy ra số có năm chữ số nhỏ nhất chia hết cho 101 là
101
100  101
cd
• Số các số của dãy số viết theo quy luật được tính theo công thức s   1 trong đó
h
c là số cuối, d là số đầu, h khoảng cách giữa 2 số liên tiếp của dãy.
Số các số có 5 chữ số thỏa mãn yêu cầu của bài toán là:
990.101  100.101
 1  990  101  1  891 .
101
Câu 3.
E
A
F

M O

N
C
B

1). Ta có góc nội tiếp bằng nhau   BCF


BDM  (1) và   BFA
BMA  suy ra
  180 0  BFA
180 0  BMA  hay BMD
  BFC
 (2).

Tài liệu sưu tầm và tổng hợp bản word đầy đủ liên hệ 0393732038 TÀI LIỆU TOÁN HỌC
92
Website:tailieumontoan.com
Từ (1) và (2) , suy ra BDM và BCF đồng dạng (g - g).
Nhận xét. Đây là bài toán tương đối cơ bản và thường gặp trong các bài toán chứng minh
tam giác đồng dạng ứng dụng của góc nội tiếp.
Nhắc lại kiến thức và phương pháp.
• Các góc nội tiếp cùng chắn một cung của một đường tròn thì bằng nhau.
  BCF
+ BDM  (hai góc nội tiếp cùng chắn cung AB của đường tròn (O) ).
  BFA
+ BMA  (hai góc nội tiếp cùng chắn cung AB của đường tròn ngoại tiếp tam giác
ABM )
  180 0  BFA
 180 0  BMA   BMD
  BFC
.
• Hai tam giác có hai cặp góc tương ứng bằng nhau thì đồng dạng.
  BCF
+ BDM và BCF có BDM  và BMD
  BFC
 , suy ra BDM ∽ BCF (g – g).

 suy ra DB  DC vậy DE vuông góc với BC tại trung điểm N


2). Từ AD là phân giác BAC
của BC .
DM BD
Từ 1). BDM ∽ BCF , ta có  .
CF BC
DA 2 DM 2 BD CD DE
Vậy ta có biến đổi sau     (3).
CF CF BC CN CE
  FCE
Ta lại có góc nội tiếp ADE  (4).

Từ (3) và (4) , suy ra EAD ∽ EFC suy ra EFC  EAD


  90 .

Vậy EF  AC .
Nhận xét. Với bài toán này ta đưa về chứng minh EF tạo với AC một góc vuông. Dựa vào
các góc đã biết và kết nối bởi tam giác đồng dạng.
Nhắc lại kiến thức và phương pháp.
• Hai góc nội tiếp bằng nhau chắn hai cung thì hai cung đó bằng nhau và hai dây cung
của cung đó bằng nhau.
Từ AD là phân giác BAC suy ra BAD
  DAC suy ra DB  DC kết hợp với OB  OC (
 R ) suy ra DO hay DE là trung trực của BC hay DE vuông góc với BC tại trung
điểm N của BC .
• Các góc nội tiếp cùng chắn một cung của đường tròn thì bằng nhau.
  FCE
ADE  (hai góc nội tiếp cùng chắn cung AE của đường tròn (O) )

• Các dữ liệu được suy ra, biến đổi từ những điều đã chứng minh.
DM BD 2 DM 2 BD DA CD
BDM ∽ BCF      
CF BC CF 2CN CF CN
• Hệ thức lượng trong tam giác vuông “Cạnh huyền  Đường cao = Tích hai cạnh góc
vuông”.
Áp dụng hệ thức lượng vào tam giác CDE vuông tại C , đường cao CN ta có
CD DE DA DE
CN .DE  CE.CD   kết hợp với chứng minh trên ta được  kết hợp
CN CE CF CE
  EFC
với trên, ta suy ra EAD ∽ EFC (g – c – g)  EAD .
• Góc nội tiếp chắn nửa đường tròn là góc vuông.

Tài liệu sưu tầm và tổng hợp bản word đầy đủ liên hệ 0393732038 TÀI LIỆU TOÁN HỌC
93
Website:tailieumontoan.com
 chắn nửa đường tròn (O) được chia bởi đường kính ED nên EAD
Góc EAD   90 suy
  90 hay EF vuông góc với AC.
ra EFC

Câu 4.
Với  là số thực dương ta có
d3 a3 b3 dab d 3 b3 c3 dbc d 3 c3 a3 dca a 3  b3  c 3 abc
 3 3 2 ;  3 3 2 ;  3 3 2 ;  2 .
3 3 3  3 3 3  3 3 3  3 2 
Cộng bốn đẳng thức trên ta thu được
 2 1  1
d 3   3  2 a 3  b3  c 3   2 dab  dbc  dca  abc .
 3 3  
2 1 4 4
Ta tìm   0 sao cho  2   2     3  4 3  3  6
3 3
3 9 3
3
1 1 1 1 3 1
Chọn    x   , ta được  x     x    6
2 x 2  x  2  x
1 1  3 1 3  1
  x 3  3    x     x    6  x 6  12 x 3  1  0 .
2  x  2  x  2  x
Ta có nghiệm dương là
1 
x  3 6  35 ; x  3 6  35     3 6  35  3 6  35 
2 
Với  xác định như trên ta thu được
4 3 1
d3   a  b3  c 3   2
9 
9 36
 9d 3  4  a 3  b 3  c 3   2  2
.
 3 3 
 6  35  6  35 
 
 1
Đẳng thức xảy ra khi a  b  c  3 ; d= 3 3
3   3 2
36
Vậy giá trị nhỏ nhất của P là 2 .
3 
 6  35  3 6  35 
 

Đề số 14

Câu 1.
1). Cộng hai phương trình của hệ ta thu được
3
x 3  y 3  6 xy  8  x 3  y 3  2  3 xy 2  0

 
  x  y  2 x 2  y 2  4  xy  2 y  2 x  0
2
Ta luôn có x 2  y 2  2  xy  2 y  2 x đẳng thức xảy ra khi và chỉ khi x  y  2 .
Vậy nếu x 2  y 2  4  xy  2 y  2 x  0 ta suy ra x  y  2 (loại) vì không thỏa mãn phương
trình 7 xy  y  x  7 .

Tài liệu sưu tầm và tổng hợp bản word đầy đủ liên hệ 0393732038 TÀI LIỆU TOÁN HỌC
94
Website:tailieumontoan.com

x  y  2  y  2  x
Vậy thu được hệ  .

7 xy  y  x  7

x  1  y  1
2 
Suy ra 7 x 2  x  2  2 x  7  7 x  12 x  5  0   .
x  5  y  9
 7 7
Nhận xét: Bài toán sử dụng phương pháp thế (hay cộng vế) để ra được phương trình có
mối liên hệ giữa các biến. Sau đó thế ngược lại tìm nghiệm của hệ phương trình.
Nhắc lại kiến thức và phương pháp:
• Tổng các đại lượng không âm:
2 2 2
a  b  b  c  a  c  0  a2  b2  c 2  ab  bc  ca
• Đẳng thức:

a3  b3  c 3  3abc  a  b  c a2  b2  c 2 ab  bc  ca 
Ý tưởng: Cả hai phương trình của hệ, đều xuất hiện nhân tử x  y vì thế ta sẽ nghĩ đến
chuyện thế x  y từ phương trình một vào phương trình hai (hoặc ngược lại), do đó ta có
được x 3  y 3  6 xy  8  0 ( i ). Đến đây ta mong muốn sẽ biểu diễn mối quan hệ giữa x , y ,
3
quan sát phương trình ( i ), ta thấy rằng 2  8 và 6 xy  3xy 2 do đó nếu đặt z  2
thì ( i )  x 3  y 3  z 3  3xyz  0 . Một biểu thức đối xứng rất đẹp, bằng cách nhóm nhân tử,
ta có:
3
x 3  y 3  z 3  3 xyz  0   x  y  z 3  3 xy  x  y  z  0

 
  x  y  z x 2  2 xy  y 2  xz  yz  z 2  3 xy  x  y  z  0

 
  x  y  z x 2  y 2  z 2  xy  yz  xz  0 (*).

1 2 2 2
Dễ thấy x 2  y 2  z 2  xy  yz  xz   x  y   y  z   z  x  do đó, phương trình
2  
x  y  z  0 x  y  2
   . Công việc còn lại là thay z  2 suy ra  . Nhưng x  y  2
x  y  z  x  y  2
loại vì không thỏa mãn phương trình hai trong hệ. Với x  y  2 thay xuống phương trình
5 9 
hai, ta tìm được nghiệm của hệ phương trình là  x; y  1; 1 ,  ; .
 7 7 
Bài toán kết thúc.
Bài tập tương tự:
x 3  y 3  1  3 xy
1. Giải hệ phương trình  2 .
x  y 2  x  2 y  3  0

 1  33 5  33   1  33 5  33 
  , 
Đáp số:  x; y  1; 1 ,  ;
  4
;  .

 4 4   4 

x 3  y 3  x  2 xy  3
2. Giải hệ phương trình  .
x  xy  4


Đáp số: vô nghiệm

Tài liệu sưu tầm và tổng hợp bản word đầy đủ liên hệ 0393732038 TÀI LIỆU TOÁN HỌC
95
Website:tailieumontoan.com
2). Điều kiện 1  x  1 .
Phương trình tương đương với
2
2  x 1   1  x2  x  1  1  x  2 x  1

  
x  1  1 x  2 x  1  2  x  1  x  1  1 x    x  1  1 x  2  
x  1 1  0 .

+ Giải x  1  1  x  2  2  2 1  x2  4  1  x2  1  x  0 .
+ Giải x 1 1 x  0 .
Đáp số x  0 .
Nhận xét: bài toán sử dụng phương pháp nhóm nhân tử chung, sau đó nâng lũy thừa bậc
hai để tìm nghiệm của phương trình.
Nhắc lại kiến thức và phương pháp:
 f  x  0
• Giải phương trình f  x.g  x  0   .
 g  x  0

• Giải phương trình a  f  x  a  f  x  b



a  f  x  a

a  f  x  a
    2 2 .
2 a  2 a  f  x  b 4  a  f  x  b  2 a
2 2 2 2 2

Ý tưởng: Bài toán xuất hiện ba căn thức, nhưng có điều đặc biệt ở đây là căn thức còn lại là
tích của hai căn thức kia. Mặt khác 1  x 2 , 1  x có sự đồng nhất hệ số, do đó ta sẽ nhóm

hai căn này lại nên ta được nhân tử chung như sau: 1  x2  1  x  1  x  
x  1  1 . Và

ta mong muốn biểu thức x  3  3 x  1 sẽ phân tích được biểu thức có chứa x  1  1.
Thật vậy, nếu coi h  x  x  3  3 x  1 là một phương trình bậc hai ẩn x  1 ta sẽ thấy:

h  x  x  1  3 x  1  2   x 1 1  
x  1  2 . Chính vì thế bài toán của ta được giải

quyết như sau:


x  3  1  x2  3 x  1  1  x
  x 1 1  
x  1  2  1 x  
x 1 1  0

 x11
  x 1 1  x  1  1  x  2  0   
 x  1  1  x  2
.

Phần còn lại chỉ là việc bình phương các phương trình và tìm nghiệm như ở trên đã nêu.
Ta được nghiệm của phương trình là x  0.
Bài toán kết thúc.
Bài tập tương tự:
1. Giải phương trình x  4  4  x 2  3 x  2  2  x .
Đáp số: x  1 .
2. Giải phương trình 2 x  3  1  4 x 2  3 2 x  1  1  2 x .

Tài liệu sưu tầm và tổng hợp bản word đầy đủ liên hệ 0393732038 TÀI LIỆU TOÁN HỌC
96
Website:tailieumontoan.com
Đáp số: x  0 .
Câu 2.
1). Nhận xét: a; b là các số nguyên thỏa mãn a2  b2  3 thì a; b 3 thật vậy, vì
a 2  0,1mod 3 ; b2  0,1mod 3 .
 2
a  0 mod 3

suy ra a2  b2  0 mod 3    a , b 3 .

b 2
 0  mod 3

  
Phương trình tương đương với 6 x 2  9 y 2  x 2  y 2  28  9 3 . 

 x 2  0 mod 3
2 2 

suy ra x  y  0 mod 3    x  3 x1 ; y  3 y1 ( x1 ; y1   ).

 y 2
 0  mod 3


Thay vào phương trình ta thu được 5  9 x12  8  9  y12  28  9 3
 5  x12  8  y12  28  9 2 .
Lập luận tương tự ta thu được x1  3x2 ; y1  3 y2 ( x2 ; y2   ).
Và nhận được phương trình 5  9 x22  8  9  y22  28  9 2
 5  x22  8  y22  28  9 .
Tương tự ta có x2  3x3 ; y2  3 y3 ( x3 ; y3   ) và thu được 5  x32  8  y32  28 .
28
Từ phương trình suy ra y32   22 .
8
 2 28
 y3  0  x32  2 2 2
Suy ra 
 5  x3  2 ; y 3  1 .
 y 2  1  x2  22
 3 3

 x22  9  2 2 ; y22  9 .
 x12  9 2  2 2 ; y12  9 2  x 2  9 3  2 2 ; y 2  9 3 .
Đáp số: x  2  33 ; y  33 , x  2  33 ; y  33 , x  2  33 ; y  33 , x  2  33 ; y  33 .
Nhận xét. Bài toán nghiệm nguyên giải bằng phương pháp xét số dư hay đồng dư thức
Nhắc lại kiến thức và phương pháp.
• Một số chính phương chia hết cho 3 chỉ tồn tại dư 0 hoặc 1. Do đó tổng của hai số
chính phương chia cho 3 dư 0 hoặc dư 2. Nên a; b là các số nguyên thỏa mãn a2  b2  3
thì a; b  3 ,

 a 2  0 mod 3
2 2 

suy ra a  b  0 mod 3    a , b 3 .

b 2
 0  mod 3


• Một số chia hết cho 3 có dạng x = 3 k .
  
Phương trình tương đương với 6 x 2  9 y 2  x 2  y 2  28  9 3 . 
Suy ra

 x 2  0 mod 3
2 2 

x  y  0 mod 3    x  3 x1 ; y  3 y1 ( x1 ; y1   ).

 y 2
 0  mod 3


Tài liệu sưu tầm và tổng hợp bản word đầy đủ liên hệ 0393732038 TÀI LIỆU TOÁN HỌC
97
Website:tailieumontoan.com

Thay vào phương trình ta thu được 5  9 x12  8  9  y12  28  9 3


 5  x12  8  y12  28  9 2 .
Lập luận tương tự ta thu được
x1  3 x2 ; y1  3 y2 ( x2 ; y2   ).
Và nhận được phương trình 5  9 x22  8  9  y22  28  9 2
 5  x22  8  y22  28  9 .
Tương tự ta có x2  3x3 ; y2  3 y3 ( x3 ; y3   ) và thu được 5  x32  8  y32  28 .
• Hai số hạng không âm luôn nhỏ hơn hoặc bằng tổng.
28
Từ phương trình suy ra y32   22 ,
8
 2 28
 y3  0  x32  2 2 2 2 2 2
suy ra  5  x3  2 , y 3  1  x2  9  2 ; y 2  9
 y 2  1  x2  22
 3 3

 x12  9 2  2 2 ; y12  9 2  x 2  9 3  2 2 ; y 2  9 3 .
Đáp số: x  2  33 ; y  33 , x  2  33 ; y  33 , x  2  33 ; y  33 , x  2  33 ; y  33 .
2 1
2). Ta có P  1  x 2 y 2  2 xy  .
xy xy

 x  y 2 1
Đặt t  xy     .
 2  4
P 1 1 15 17
Ta thu được  t   16t   15t  2 16    P  17 .
2 t t 4 2
Dấu “=” xảy ra khi
1
xy  Pmin  17 .
2
Nhận xét: Bài toán sử dụng bất đẳng thức Cosi kết hợp với giả thiết tìm giá trị nhỏ nhất
của thức bài cho.
Nhắc lại kiến thức và phương pháp:
• Bất đẳng thức Cosi cho hai số thực dương a  b  2 ab .
2
1 1 2  a  b 
• Các hệ quả từ bất đẳng thức Cosi   ; ab    .
a b ab  2 
Ý tưởng: Bài toán có sự đối xứng giữa hai biến x; y ( vai trò của chúng như nhau ) vì thế
1
điểm rơi ta khẳng định là x  y , kết hợp với giả thiết suy ra được tại x  y  thì Pmin . Mặt
2
khác, xét với biểu thức P , có xuất hiện đại lượng xy trong căn, vì vậy ra nghĩ đến bất
1 1
đẳng thức Cosi để đánh giá đại lượng  về f  xy .
x y
1 1 2 2 1  x2 y 2 1
Ta có   P 2  xy
x y xy xy xy

Tài liệu sưu tầm và tổng hợp bản word đầy đủ liên hệ 0393732038 TÀI LIỆU TOÁN HỌC
98
Website:tailieumontoan.com
Và khai thác giả thiết, ta cũng sẽ đánh giá về xy , với điểm rơi x  y thì ta có đánh giá
2
2  x  y  1
 x  y  0  xy     . Và nếu đặt t  xy thì ta cần tìm giá trị nhỏ nhất của biểu
 2  4
1 1
thức P  2  t với t  . Bằng sự khéo léo trong chọn điểm rơi, ta đánh giá như sau:
t 4
1 1 15
2 16t   15t  2 2 16t.  15t  2 2 16   17
t t 4
 Pmin  17 .
1
Dấu “=” xảy ra khi x  y  .
2
Bài toán kết thúc.
Bài tập tương tự:
1. Cho x; y là các số thực dương thỏa mãn x  y  2 . Tìm giá trị nhỏ nhất của biểu thức
1 1
P     1  x 2 y 2 .
 x y 
2. Cho x; y là các số thực dương thỏa mãn x  y  2 . Tìm giá trị nhỏ nhất của biểu
4x 2 y 2 x2 y2
thức P  2
  2 .
x 2
 y2  y2 x

Câu 3.

M
Q
Q M
N N E
E F O
F
O H
H P
P

B B C
C K

1). Ta có   BHC
BPC   180  BAC   BEC
 , suy ra tứ giác AEPF nội tiếp, nên BFC   1800 .

Mặt khác từ các tứ giác AQFN ; AQEM nội tiếp ta có


  MQA
MQN   NAQ
  MEA
  NFA
  1800 .

Vậy M ; N ; Q thẳng hàng.


Nhận xét. Từ ba điểm bất kỳ tạo thành hai tia có chung gốc. Khi hai tia đó tạo với nhau
một góc tù thì ba điểm đã cho thẳng hàng.
Nhắc lại kiến thức và phương pháp.

Tài liệu sưu tầm và tổng hợp bản word đầy đủ liên hệ 0393732038 TÀI LIỆU TOÁN HỌC
99
Website:tailieumontoan.com
• Hai góc nội tiếp cùng chắn một cung của một đường tròn thì bằng nhau.
  BHC
+ BPC  của đường tròn ngoại tiếp tam giác
 (hai góc nội tiếp cùng chắn cung BC

HBC ).
  AFN
+ AQN  (hai góc nội tiếp cùng chắn cung AN
 của đường tròn ngoại tiếp tam

giác FAN ).
  AEM
+ AQM  (hai góc nội tiếp cùng chắn cung AM
 của đường tròn ngoại tiếp tam

giác AME ).
• Tứ giác có tổng hai góc đối diện bằng 180 là tứ giác nội tiếp. Tứ giác nội tiếp có tổng
hai góc đối diện bằng 180 .
Gọi chân đường cao từ đỉnh B và C của tam giác ABC là E và F  .
 
Tứ giác AE HF  có AE H  AF  H  90  90  180 , suy ra AE HF  là tứ giác nội tiếp, khi
     (hai góc đối đỉnh) nên
 mà F  HE  BHC
đó F  AE  F  HE  180  F  HE  180  BAC
  180 BAC
BHC  kết hợp với trên ta được BPC
  180 BAC
  BPC
  BAC
  180 suy

ra tứ giác AEPF là tứ giác nội tiếp.


  AEP
Suy ra AFP   180  180 AFP

  180 AEP
  180
  
  BEC
 BFC   180  NFA
  AEM
  180 kết hợp với trên, ta có
  AQM
NAQ   180  NQM
  180 hay ba điểm N ; Q; M thẳng hàng.

  ANQ
2). Ta có các góc nội tiếp bằng nhau AFQ   ANM
  ABM
 suy ra FQ  BE . Tương tự

EQ  CF .
  QFP
Từ đó tứ giác EQFP là hình bình hành, suy ra QAN   QAM
  QEP  hay AQ là phân

.
giác MAN
 thì A , P , Q thẳng hàng.
Nếu AP là phân giác MAN
  QAC
Từ đó nếu PQ giao BC tại K thì KAC   QME
  NMB
  PCK

Vậy AKC ∽ CKP , suy ra KC 2  KP.KA .


Tương tự KB2  KP.KA .
Từ đó KB  KC hay K là trung điểm.
Nhận xét. Chứng minh một đường thẳng đi qua trung điểm của một đoạn thẳng khác ta
chứng minh giao điểm của đường thẳng với đoạn thẳng là trung điểm của đoạn thẳng.
Nhắc lại kiến thức và phương pháp.
• Một đường thẳng cắt hai đường thẳng khác tạo ra cặp góc đồng vị bằng nhau thì hai
đường thẳng song song.
  ANQ
Từ kiến thức ở trên, ta có AFQ   ANM   ABM  và ABM
 mà AFQ  ở vị trí đồng

vị của hai đường thẳng FQ và BE nên suy ra FQ  BE . Hoàn toàn tương tự ta có


EQ  CF .
• Tứ giác có hai cặp cạnh song song là hình bình hành.

Tài liệu sưu tầm và tổng hợp bản word đầy đủ liên hệ 0393732038 TÀI LIỆU TOÁN HỌC
100
Website:tailieumontoan.com
Tứ giác EQFP có FQ  BE và EQ  CF suy ra EQFP là hình bình hành khi đó
  QFP
QAN   QEP
  QAM
 suy ra AQ là phân giác MAN
 . Do đó A , Q , P thẳng hàng.
  QME
KAC  (hai góc nội tiếp cùng chắn cung QE
 của đường ngoại tiếp tam giác

AME ).
  PCK
QME  (hai góc nội tiếp cùng chắn cung NB
 của đường tròn O ),

  PCK
 KAC  chung của hai tam giác AKC và CKP nên suy ra
 mà lại có góc PKC
AKC ∽ CKP (g – g)
AK KC 2
   KC 2  AK.KP hoàn toàn tương tự ta có KB  KP.KA . Từ đó suy ra
CK KP
KC 2  KB2  KC  KB hay K là trung điểm của BC .

Câu 4.
Giả sử k là chỉ số mà x1  x2    xk  0  xk1    x192 .
Ký hiệu S  x1  x2    xk ; S  xk1  xk2    x192
2013
 S  S  0  S  S  2013  S  S  .
2
Do x1  x2    x192 suy ra S  kx1 ; S  192  k x192
S S S S
 x1    x1  ; x192 
k k k 192  k
S S 2013 2013 2013  192
 x192  x1      .
192  k k 2 192  k  2k 2 k 192  k

 192  k  k  2 192 2
Ta có 2 k 192  k  2  
 
 2  2
2013  192 2013
 x192  x1   .
192 2 96
2
Dấu “=” xảy ra khi
2013 2013
x1  x2    x96   ; x97  x98    x192 
192 192

Đề số 15

Câu 1.
1). Điều kiện x  6.
Phương trình đã cho tương đương với  x  9  2012  
x  6 1  0 .
2
+ Giải x  9  2012  0  x  2012  9  4048135 .

+ Giải x  6  1  0  x  5 .
Vậy phương trình có hai nghiệm: x  4048135; x  5 .

Tài liệu sưu tầm và tổng hợp bản word đầy đủ liên hệ 0393732038 TÀI LIỆU TOÁN HỌC
101
Website:tailieumontoan.com
Nhận xét: Bài toán sử dụng phương pháp nhóm nhân tử chung và nâng lũy thừa tìm
nghiệm của phương trình.
Nhắc lại kiến thức và phương pháp:
• Cách giải phương trình dạng
 f x m 
   f  x  m 2
 
f  x  m . 
g  x  n  0  
 g  x  n
 
 g  x  n
2
.

Ý tưởng: Bài toán cho hết sức đơn giản, với sự xuất hiện của hai căn thức
x  9; x  6; x  9x  6 nên không khó để nhóm được nhân tử chung như sau:
x  9  2012 x  6  2012  x  9x  6
 x9  x  9x  6  2012 x  6  2012  0
 
 x  9 1  x  6  2012 1  x  6  0  

 1 x  6  
x  9  2012  0 .
Bài toán kết thúc.
Bài tập tương tự:
1. Giải phương trình 2 x  6  x  7  2  x 2  13x  42 .
Đáp số: x  3; x  5 .
2. Giải phương trình x  4  2 x  3  2  x 2  7 x  12 .
Đáp số: x  0; x  2 .
 2 2
x   y  1  5
2). Cách 1: Hệ đã cho tương đương với  .
x  y  1  x   y  1  5


u  x   y  1
Đặt 
2 2
 x 2   y  1   x  y  1  2 x  y  1  u2  2 v .


 v  x  y  1


u2  2 v  5
Thu được 

u  v  5

u  3  v  2
 u2  2 5  u  5  u2  2u  15  0   .

u  5  v  10

x   y  1  3  x  y  1
+ Giải  

.
 


 x y  1  2  x  2, y  0

x   y  1  5
+ Giải  (vô nghiệm).
 


 x y  1  10

Vậy hệ phương trình có hai nghiệm 1; 1 và 2; 0 .



x 2  y 2  2 y  4
Cách 2: Hệ tương đương với  .

4 x  2 y  2 xy  8

Cộng vế với vế hai phương trình ta thu được

Tài liệu sưu tầm và tổng hợp bản word đầy đủ liên hệ 0393732038 TÀI LIỆU TOÁN HỌC
102
Website:tailieumontoan.com
2 x  y  2
 x  y  4  x  y  12  0  
 x  y  6
.

x  y  2 x  y  2 x  1  y  1
+ Giải     .
2 x  y  xy  4 x  x 2  x  2  x  2  y  0

x  y  6 x  y  6
+ Giải    (vô nghiệm).
2 x  y  xy  4 x  x 6  x  10

Vậy hệ phương trình có nghiệm  x; y  1; 1 , 2; 0 .

Nhận xét: Bài toán sử dụng phương pháp ẩn phụ sau đó từ ẩn phụ tìm ngược lại nghiệm
của hệ phương trình.
Ý tưởng: Sự xuất hiện của y 2  2 y ở phương trình một của hệ, làm ta nghĩ đến hằng đẳng
2 2
thức y 2  2 y  1   y  1 hay nói cách khác, từ phương trình một ta có: x 2   y  1  5 .

Đây là phương trình có dạng là tổng các bình phương, dễ làm ta suy đoán đến hệ phương
trình đối xứng loại I, tức là đặt ẩn phụ theo định lý Vi-et ( đặt tổng và tích ) như sau:
u  x  y  1 và v  x  y  1 .

Nhưng đây cũng chỉ là suy đoán ban đầu, bây giờ ta sẽ đi xét phương trình hai để xuất
hiện u, v .
Thật vậy, ta có phương trình hai trong hệ tương đương với:
2 x  y  xy  4  x  y  1  x  y  1  5 .


u2  2 v  5 u  3  v  2
Do đó hệ phương trình đã cho trở thành  
 .
 u  5  v  10
u  v  5

Thế ngược lại tìm hệ của phương trình ban đầu.


Hoặc, ta có thể suy luận như sau: ta đi kết hợp cả hai phương trình trong hệ, vẫn với sự
xuất hiện x 2  y 2 ở phương trình một, đồng thởi có tích ở phương trình xy ta sẽ liên tưởng
2
đến hằng đẳng thức  x  y . Vì thế lấy phương trình hai nhân 2 rồi cộng với phương trình

một ta được:
2 x  y  2
 x  y  4  x  y  12  0  
 x  y  6
.

Thế ngược lại một trong hai phương trình trong hệ ban đầu để tìm nghiệm của hệ ban
đầu.
Bài toán kết thúc.
Bài tập tương tự:

Tài liệu sưu tầm và tổng hợp bản word đầy đủ liên hệ 0393732038 TÀI LIỆU TOÁN HỌC
103
Website:tailieumontoan.com

x 2  y 2  4 y  1
1. Giải hệ phương trình  .

3 x  xy  y  3

Đáp số:  x; y  1; 0 , 2;  1 .



x 2  2 x  y 2  4 y  0
2. Giải hệ phương trình  .

3 x  xy  2 y  0

Đáp số:  x; y  0; 0 , 1;  1 .

Câu 2.
1). Phương trình tương đương với
x  y  1xy  x  y  2 x  y  1  3
  x  y  1 xy  x  y  2  3 .

 x  y  1 là ước của 3.

x  y  1  1 x  y  0
+ Giải    (vô nghiệm).
xy  x  y  2  3 xy  5

x  y  1  1 x  y  2 x  1


+ Giải      .
xy  x  y  2  3 xy  1  y  1

x  y  1  3 x  y  2 x  1


+ Giải      .
xy  x  y  2  1 xy  1  y  1

x  y  1  3 x  y  4
+ Giải    (vô nghiệm).
xy  x  y  2  1 xy  5

Vậy  x; y  1;  1 , 1; 1 .

Nhận xét. Bài toán nghiệm nguyên giải bằng phương pháp đưa về phương trình ước số
Nhắc lại kiến thức và phương pháp.
• Phân tích đưa về phương trình ước số
x  y  1xy  x  y  5  2  x  y
  x  y  1 xy  x  y  3  2  2  x  y

  x  y  1 xy  x  y  3  2  x  y  1

  x  y  1 xy  x  y  2  x  y  1  3

  x  y  1 xy  x  y  2  3

• Phân tích số thành tích của 2 ước số


3  1.3  3.1 .
• Cho mỗi thừa số chứa biến ở vế này đồng nhất với thừa số ở vế kia

Tài liệu sưu tầm và tổng hợp bản word đầy đủ liên hệ 0393732038 TÀI LIỆU TOÁN HỌC
104
Website:tailieumontoan.com

2). Ta có  x 1  
y  1  4  xy  x  y  3.

Theo bất đẳng thức Cô si


x  y x 1 y 1
3  xy  x  y     xy2.
2 2 2
x2 

 y  2 x
y  x2 y 2
Theo bất đẳng thức Cô si 2  P   xy2 .
y 
 y x
 x  2 y
x 

Vậy giá trị nhỏ nhất của P là 2 khi x  y  1 .
Nhận xét: bài toán sử dụng bất đẳng thức Cosi dựa trên điểm rơi đã được suy đoán cũng
như kết hợp với điều kiện bài toán để tìm giá trị nhỏ nhất.
Nhắc lại kiến thức và phương pháp:
• Bất đẳng thức Cosi cho hai số thực dương x  y  2 xy .
2
a 2 b2 a  b
• Mở rộng ra một đánh giá   , a ; b; x ; y  0 .
x y xy
Ý tưởng: Đây là một bài toán có sự đối xứng rõ ràng giữa x , y nên ta mạnh dạn dự đoán
2
điểm rơi tại x  y  k . Thay ngược lại giả thiết bài toán, ta có  
k  1  4  k  1 . Với điểm
rơi x  y  1 , ta sẽ dễ dàng đánh giá hơn khi vận dụng bất đẳng thức Cosi, chính vì vậy,
khi đi khai khác giả thiết, ta suy ra:  x 1  
y  1  4  xy  x  y  1  4
x  y x 1 y 1
 xy  x  y  3  3     xy2.
2 2 2
x2 y 2
Vậy nên, bây giờ ta sẽ đánh giá biểu thức P theo   f  x  y  2 . Hiển nhiên có con
y x
số 2 vì điểm rơi x  y  1 . Vậy nên ta cần tìm f  x  y thỏa mãn f  x  y  2 . Mà như bên
x2 y 2
trên, ta đã tìm được x  y  2 do đó, ta cần chứng minh rằng   x  y . Đánh giá này
y x
ta có các cách như sau:
• Biến đổi tương đương, ta có:
x2 y2 x  x  y y  x  y 2
x y0    0   x  y  0 .
y x y x
• Sử dụng bất đẳng thức Cosi, ta có:
 x 2 x2

 y  y  2 .y  2 x
 y x2 y 2
  xy.
 2 y x
y y2
  x  2 .x  2 y
 x x
• Sử dụng đánh giá mở rộng như đã nêu, ta có:
2
x 2 y 2  x  y
  xy.
y x xy
Tài liệu sưu tầm và tổng hợp bản word đầy đủ liên hệ 0393732038 TÀI LIỆU TOÁN HỌC
105
Website:tailieumontoan.com
Bài toán kết thúc.
Bài tập tương tự:
1. Cho a; b là hai số thực dương thỏa mãn ab  1 . Tìm giá trị nhỏ nhất của biểu thức
a2 b2
P   2 a 2  b2  .
b a
2. Cho a; b; c là các số thực dương thỏa mãn a  b  c  3 . Tìm giá trị nhỏ nhất của biểu
a b c
thức P  2
 2
 .
1 b 1 c 1  a2
Câu 3.

1). Vì MP là đường kính suy ra PN  MN (1).


Vì MD là đường kính suy ra DN  MN (2).
Từ (1) và (2), suy ra N ; P; D thẳng hàng.
Nhận xét. Chứng minh ba điểm thẳng hàng ta quy về chứng minh chúng cùng thuộc một
đường thẳng.
Nhắc lại kiến thức và phương pháp.
• Góc nội tiếp chắn nửa đường tròn là góc vuông.
 là góc nội tiếp chắn nửa đường tròn được chia đôi bởi đường kính PM
+ Góc PNM
  90 hay PN  NM .
của đường tròn đường kính PM nên PNM
 là góc nội tiếp chắn nửa đường tròn được chia đôi bởi đường kính DM
+ Góc DNM
  90 hay DN  NM
của đường tròn (O) nên DNM
• Từ một điểm nằm ngoài một đường thẳng đã kẻ được một và chỉ có một đường thẳng
vuông góc với đường thẳng đó.
Ta có từ N ta kẻ được PN và DN vuông góc với MN , suy ra PN  DN hay ba điểm
P ; N ; D thẳng hàng.

Tài liệu sưu tầm và tổng hợp bản word đầy đủ liên hệ 0393732038 TÀI LIỆU TOÁN HỌC
106
Website:tailieumontoan.com
  MAD
2). Tứ giác APQD nội tiếp ( PQD   90 0 ),

  PDQ
suy ra PAQ   NDM
 (3).
  NAM
Xét (O) , ta có NDM  (4).
  NAP
Từ (3) và (4) PAQ  (*).
 , suy ra AP là phân giác của góc NAQ

  AMD
Xét (O) , ta có AND .

  QNP
Xét đường tròn đường kính MP có QMP   ANP
  QNP
 , nên NP là phân giác của

 (**).
góc ANQ
Từ (*) và (**), suy ra P là tâm đường tròn nội tiếp tam giác ANQ.
Nhận xét. Chứng minh một điểm là tâm đường tròn nội tiếp một tam giác ta chứng minh
điểm đó là giao điểm của hai đường phân giác trong của tam giác đó.
Nhắc lại kiến thức và phương pháp.
• Tứ giác có tổng hai góc đối diện bằng 180 là tứ giác nội tiếp.
  90
  90 (góc nội tiếp chắn nửa đường tròn (O) ) và DQP
Tứ giác APQD có DAP
(góc ngoài tại đỉnh đối diện bằng góc trong không kề với nó của tứ giác nội tiếp) suy
  DQP
ra DAP   90  90  180 do đó tứ giác APQD là tứ giác nội tiếp.

• Hai góc nội tiếp cùng chắn một cung của một đường tròn thì bằng nhau.
  QAP
Tứ giác APQD là tứ giác nội tiếp nên QDP  (hai góc nội tiếp cùng chắn cung QP

  NDM
của đường tròn (O) ) hay PAQ  .

  NAM
+ NDM  của đường tròn (O) ).
 (hai góc nội tiếp cùng chắn cung MN

  AMD
+ AND  (hai góc nội tiếp cùng chắn cung AD
 của đường tròn (O) ).

  QNP
+ QMP  (hai góc nội tiếp cùng chắn cung QP
 của đường tròn đường kính PM ) ,

  NAP
suy ra PAQ  hay AP là phân giác của NAP   QNP
 và AND  hay ND là phân

.
giác của QNA
• Giao điểm của hai đường phân giác của hai góc trong một tam giác là tâm đường tròn
nội tiếp tam giác đó.
 , ta có
 và ND là phân giác của QNA
Tam giác ANQ có AP là phân giác của NAP
AP và ND cắt nhau tại P nên suy ra P là tâm đường tròn nội tiếp của ANQ .

Câu 4. Ta có

Tài liệu sưu tầm và tổng hợp bản word đầy đủ liên hệ 0393732038 TÀI LIỆU TOÁN HỌC
107
Website:tailieumontoan.com

abc  ab  ac  a  abc  bc  ba  b abc  ca  cb  c


Q
a  1b  1c  1
a b  1c  1  bc  1a  1  c a  1b  1
Q
a  1b  1c  1
a b c
Q   .
a 1 b 1 c 1
Ta chứng minh rằng
a b c 1 2 3 5
      .
1  a 1  b 1  c 1  1 1  2 1  3 12
Bất đẳng thức trên tương đương với
 3 c   b 2   a 1 

 1  3  1  c    1  b  1  2    1  a  1  1  0
  

3c b2 a 1
   0
4 1  c 3b  1 2 1  a

   
1 1   1 1 
 3  c     
 3  c  
 b  2  
   
 4 c  1 3b  1   3b  1 2 1  a
   
1
 3  c  b  2  a  1 0
  2 1  a

3b  4c  1 2a  3b  1 1
 3  c   b  1  c   a  b  c  0.
12 b  1c  1 6 b  1a  1 2 a  1

3b  4c  1
Vì c  3; 0  b  c  3  c  0 (1).
12 b  1c  1

2a  3b  1
Vì b  1  c; 0  a  b  b  1  c  0 (2).
6 b  1a  1

1
Vì a  b  c; 0  a  a  b  c   0 (3).
2 a  1

Từ (1), (2) và (3), suy ra điều phải chứng minh.


5
Vậy giá trị nhỏ nhất của Q là khi a  1; b  2; c  3 .
12
Đề số 16

Câu 1.
1). Hệ phương trình đã cho tương đương với

xy  x  y  2

 3
x  y  6  27 x  y  9 xy 3 x  y
 3 3 3


Tài liệu sưu tầm và tổng hợp bản word đầy đủ liên hệ 0393732038 TÀI LIỆU TOÁN HỌC
108
Website:tailieumontoan.com
 xy  x  y  2
xy  x  y  2 
 3
 
x  y  3 xy  x  y  3 x  y
3 3  x  y3  3 x  y3
 
xy  x  y  2 x  y
     x y1.
x  y  3 x  y xy  x  y  2
 
Vậy nghiệm của hệ là x  y  1 .
Nhận xét: Bài toán sử dụng phép thế hằng số từ một phương trình vào phương trình còn
lại sau đó sử dụng hằng đẳng thức tìm nhân tử.
Nhắc lại kiến thức và phương pháp:
3
• Hằng đẳng thức bậc ba ax  by  a 3 x 3  3a 2 bx 2 y  3ab2 xy 2  b3 y 3 .
• Phương trình dạng f 3  x; y  g 3  x; y  f  x; y  g  x; y .
Ý tưởng: Ở cả hai phương trình của hệ, các biến x; y đều nằm trong các biểu thức bậc 3.
Và đặc biệt là cả hai phương trình cũng đều chứa hằng số. Vì vậy nếu thế hằng số này vào
hằng số của phương trình kia thì rõ ràng ta sẽ thu được một phương trình bậc ba đẳng cấp
của hai biến x; y .
Cụ thể như sau: 9 xy 3x  y  3xy  x  y  26 x 3  2 y 3 (*)
 x 3  3 x 2 y  3 xy 2  y 3  27 x 3  27 x 2 y  9 xy 2  y 3
3 3
  x  y   3 x  y  x  y  3 x  y  x  y .
Tuy nhiên, phương trình (*) là một phương trình đẳng cấp bậc ba nếu ta chia phương
trình cho y 3 , sẽ thu được một phương trình bậc ba. Ta có thể sử dụng máy tính cầm tay để
giải quyết phương trình đó. Với x  y , thế ngược lại phương trình một ta sẽ tìm được
nghiệm của hệ là x  y  1 .
Bài toán kết thúc.
Bài tập tương tự:

 y  2 x  y  3
1. Giải hệ phương trình:  2 .
x  4 xy  3  4 x 2  y 2

 2 15 15   2 15 15 
 
Đáp số:  x; y  0;  3 ,  
 5
;  , 
5   5
; .
5 
x 2  y 2  2

2. Giải hệ phương trình:  .
 x  y1  xy4  32

Đáp số: x  y  1 .

2). Điều kiện 4  x  4.


x
Phương trình đã cho tương đương với
x4 2
 
4  x  2  2x .

+Với x  0 là nghiệm.
+ Giải 4x 2  2  x4 2 
Đặt u  x  4; v  4  x ta thu được

Tài liệu sưu tầm và tổng hợp bản word đầy đủ liên hệ 0393732038 TÀI LIỆU TOÁN HỌC
109
Website:tailieumontoan.com

 v  2u  2

 2
2
 u2  2u  2  8  5u2  8u  4  0
 2
u  v  8

 2 14
u  ; v  2 96
 5 5  x4   x  (thỏa mãn).
 5 25
 u   2 ( l )
96
Vậy phương trình có hai nghiệm x  0; x   .
25
Nhận xét: Sử dụng phương pháp nhân liên hợp, sau đó đặt ẩn phụ tìm nghiệm của bài
toán.
Nhắc lại kiến thức và phương pháp:
ab
• Hằng đẳng thức a  b   a b  
a b  a b
a b
.

• Giải phương trình tổng quát dạng f  x  g  x  m



 f  x  g  x  m

 f  x ; g  x  0
2

  .
 f  x  g  x  2 m g  x  m2  f  x  g  x  m2  2  4 mg  x
  
Ý tưởng: Không khó để nhận thấy, phương trình có một nghiệm là x  0 . Đồng thời vế trái
của phương trình có xuất hiện biểu thức x  4  2 , dễ thấy rằng
2
x  
x  4  22   x4 2  
x  4  2 . Vì thế, phương trình đã cho tương đương với:
 x4 2  0  x  0
 x4 2  4x 2  2   x4 2  
x  4  2  
 4  x  2 x  4  2
Phương trình còn

lại có thể giải bằng cách tổng quát nêu ở trên, hoặc có thể giải quyết bằng cách đặt ẩn phự

v  4  x
như sau:  ( u; v  0 ). Ta có hệ phương trình

u  x  4



 v 2  u2  8 2 14 2 96

 u ; v  x4   x  .
 5 5 5 25
 v  2u  2

Bài toán kết thúc.
Bài tập tương tự:
1. Giải phương trình  x  1 1  
1  x  1  2x .
24
Đáp số: x  0; x  
25
2. Giải phương trình  x  9 1  
9  x  1  3x .
Đáp số: x  0 .
Câu 2.
2
1). +) 412  40  1  40 2  80  1  81 (mod 100).
414  812 (mod 100)  80 2  160  1 (mod 100)  61 (mod 100).
 415  61.41 (mod 100)  60.40  100  1 (mod 100)  1 (mod 100).
21
 415   41105  1 (mod 100)  41106  41 (mod 100).

Tài liệu sưu tầm và tổng hợp bản word đầy đủ liên hệ 0393732038 TÀI LIỆU TOÁN HỌC
110
Website:tailieumontoan.com
503
+) 57 4  1 (mod 100)  57 2012  57 4   1 (mod 100).
Suy ra A  41106  57 2012  41  1 (mod 100).
Vậy 2 chữ số cuối cùng của A là 42.
Nhận xét. Muốn tìm hai chữ số tận cùng của một số (thường gặp là một lũy thừa) ta xét số
dư của số đó khi chia cho 100
Nhắc lại kiến thức và phương pháp.
2
• Hằng đẳng thức bình phương của một tổng a  b  a 2  2 ab  b2 .
2
Ta có 412  40  1  40 2  80  1  4.100  81  0  81  81 (mod 100).
• Tính chất của đồng dư thức: a  b (mod n )  ac  bc (mod n ) với a ; b ; c và n đều là
các số nguyên dương.
Ta có 412  81mod100  414  812 mod100
2
812  80  1  8.100  160  1  0  60  1  61mod100
• Tính chất của đồng dư thức: a  b mod n  ac  bc mod n với a , b , c và n đều là các
số nguyên dương.
Ta có 414  61 mod100  415  61.41  2501  1 mod100
21
 415   121  1 mod100  41105  1 mod100  41106  41 mod100 Hoàn toàn áp dụng
503
các kiến thức như trên ta có 57 4  1(mod100)  57 2012  57 4   1(mod100) .

• Tính chất đồng dư thức: a  b mod n và c  d mod n khi đó a  c  b  d mod n .


Ta có 41106  41mod100 và 57 2012  1 mod100 .
Khi đó 41106  57 2012  41  1  42 mod100 .
Vậy hai chữ số cuối cùng của số A  41106  57 2012 là 42 .

1 5
2). Tập xác định x .
2 2
x 2  5  4x 2 5  3x 2
Ta có x 5  4x 2
 .
2 2
2x  1  1 3( x 2  1)
3 2x  1  3( )  3x  .
2 2
Cộng hai bất đẳng thức trên ta thu được
y  3 2x  1  x 5  4x 2  4.
Vậy ymax  4 khi x  1.
Nhận xét: bài toán sử dụng việc kết hợp đánh giá điểm rơi cũng như bất đẳng thức Cosi
để tìm giá trị lớn nhất của biểu thức ban đầu.
Nhắc lại kiến thức và phương pháp:
• Bất đẳng thức Cosi cho hai số thực dương a  b  2 ab .
• Giả sử m là điểm rơi của một bài toán, ta sử vận dụng các đánh giá quen thuộc xung
2
quanh điểm rơi là x 2  m2  2 mx   x  m  0 .

Tài liệu sưu tầm và tổng hợp bản word đầy đủ liên hệ 0393732038 TÀI LIỆU TOÁN HỌC
111
Website:tailieumontoan.com
Ý tưởng: Biểu thức bài cho chứa hai căn thức bận hai, đồng thời yêu cầu là tìm giá trị nhỏ
nhất vậy nên ta sẽ đánh giá qua hai bước, đó là: sử dụng đánh giá cosi để khử căn thức,
khéo léo biến đổi theo điểm rơi để khử dần biến số. Vậy nên việc quan trọng nhất đó là dự
đoán điểm rơi của bài toán, tuy nhiên như đã nói ta cần sử dụng Cosi cho từng căn thức
một, nên ta sẽ đánh giá cho mỗi căn như sau:
32 x  1  
2 x  1    2  2 x  1  3 2 x  1 
2 

2
m x  n 5  4 x
2 2 2 2
 2
m 2
 4n2  x 2  5n2
mx.n 5  4 x   x 5  4x 
2 2 mn
32 x  1   m 2
 4 n  x  5n
2 2 2

Do đó, suy ra P   . Bây giờ, ta sẽ có hướng tư duy là sử


2  2 mn
dụng đánh giá nào đó để khử hết biến. Biểu thức cuối có sự xuất hiện của x; x 2 do đó ta sẽ
nghĩ là nên đánh giá x về x 2 hay ngược lại. Và ta sẽ chọn giải pháp đầu tiên
x2  k 2 3 x 2  k 2  k  k m2  4n2  x 2  5n2
2 2
x  k  2 kx  2 x  . Khi đó, ta có: P   . Và nếu
2k 2k  2 mn
muốn khử được hết biến thì đầu tiên ta quan sát được là hai mẫu số phải bằng nhau và
tổng hệ số của x 2 bằng 0. Chính vì thế, ta được:
k   mn; 4n2  m2  3 m  n  k    1
 
 
2 x  1   ; m2 x 2  n2 5  4 x 2  x  1

Và từ đó, ta có đánh giá sau:
2x  1  1 x2  5  4x2
2
P  3 2 x  1  x 5  4 x  3. 
2 2
2 2 2
5  3x 3x  3 5  3x
 3x     4  Pmax  4 .
2 2 2
Bài toán kết thúc.
Bài tập tương tự:
1. Cho x; y; z là các số thực dương thỏa mãn xy  yz  zx  5 . Chứng minh rằng:
3 x 2  3 y 2  z 2  10 .
2. Chứng minh rằng với mọi 0  x  1 ta có bất đẳng thức sau:
9 x 1  x 2  13 x 1  x 2  16 .

Câu 3.

Tài liệu sưu tầm và tổng hợp bản word đầy đủ liên hệ 0393732038 TÀI LIỆU TOÁN HỌC
112
Website:tailieumontoan.com

A
R

Q S
P

B C

N M

  TQA
1). Do TPA   90 0 , nên tứ giác TAPQ nội tiếp.
  QTP
Do đó MTC   QAP
 (do tứ giác TAPQ nội tiếp)  BAN
  MAC  (do MN  BC ), suy ra

tứ giác MTAC nội tiếp, suy ra T  (O) .


Nhận xét. Có nhiều cách để chứng minh một điểm nằm trên một đường tròn, trong đó có
cách áp dụng tính chất “Có một và chỉ một đường tròn đi qua ba điểm không thẳng
hàng”.
Nhắc lại kiến thức và phương pháp.
• Tứ giác có hai đỉnh liên tiếp cùng nhìn cạnh đối dưới hai góc bằng nhau là tứ giác nội
tiếp.
Tứ giác TAPQ có đỉnh P và Q cùng nhìn cạnh AT dưới hai góc TPA   TQA
  90 0 suy

ra TAPQ là tứ giác nội tiếp.


• Hai góc nội tiếp cùng chắn một cung của một đường tròn thì bằng nhau.
  QAP
QTP  hay MTC
  BAN  của đường tròn
 (hai góc nội tiếp cùng chắn cung QP

ngoại tiếp tứ giác TAPQ ).


• Hai dây cung song song của một đường tròn tạo ra hai dây bị chắn hai bên bằng nhau
(Chứng minh dựa vào tính chất hình thang cân).
Trong đường tròn (O) có hai dây cung MN và BC song song nên ta có BN   MC.

• Hai góc nội tiếp chắn hai cung bằng nhau của một đường tròn thì bằng nhau.
Trong đường tròn (O) có góc nội tiếp BAN  và góc nội tiếp MAC
 chắn cung BN  chắn
 . Kết hợp với trên ta suy ra BAN
cung MC   MAC
.
Áp dụng kiến thức trên, tứ giác MTAC có hai đỉnh liên tiếp T và A cùng nhìn cạnh
  MAC
MC dưới hai góc bằng nhau MTC  (  BAN
 ), suy ra MTAC là tứ giác nội tiếp.
• Có một và chỉ một đường tròn đi qua ba điểm không thẳng hàng.
Tài liệu sưu tầm và tổng hợp bản word đầy đủ liên hệ 0393732038 TÀI LIỆU TOÁN HỌC
113
Website:tailieumontoan.com
Theo chứng minh trên ta có bốn điểm M ; T ; A ; C cùng thuộc đường tròn mà ba
điểm M ; A ; C cùng nằm trên đường tròn (O) , nên suy ra T cũng là điểm nằm trên
đường tròn (O) .

  PTA
2). Từ tứ giác TAPQ nội tiếp ta có PQA   CTA
  ABC
  PQ  BC  MN .

Từ đó Q  (1).
SA  NMA
  AMN
Mà tứ giác AMNR nội tiếp, suy ra ARN   180 0 (2).
 Q
Từ (1) và (2), suy ra QRA SA  180 0 , suy ra tứ giác ARQS nội tiếp, ta có điều phải chứng
minh.
Nhận xét. Có nhiều cách để chứng minh bốn cùng điểm nằm trên một đường tròn, trong
đó đưa về chứng minh bốn điểm tạo thành một tứ giác nội tiếp là thông dụng nhất.
Nhắc lại kiến thức và phương pháp.
Kiến thức nhắc lại trên
  PTA
PQA  (hai góc nội tiếp cùng chắn cung PA
 của đường tròn ngoại tiếp tứ giác TAPQ

).
  ABC
CTA  của đường tròn (O) )
 (hai góc nội tiếp cùng chắn cung AC
  PTA
Suy ra PQA   CTA  ABC.

• Một đường thẳng cắt hai đường thẳng đã cho tạo ra hai góc ở vị trí đồng vị bằng nhau
thì hai đường thẳng đã cho song song.
  ABC
Theo chứng minh trên ta có PQA  mà hai góc này ở vị trí đồng vị của hai đường

thẳng QS và BC suy ra QS  BC kết hợp với giải thiết MN  BC ta có QS  BC  MN .


• Một đường thẳng cắt hai đường thẳng song song tạo ra cặp góc đồng vị bằng nhau.
  AMN
Theo chứng minh trên ta có QS  MN nên hai góc ở vị trí đồng vị ASQ .

• Tứ giác nội tiếp có tổng hai góc đối diện bằng 180 .
  AMN
Tứ giác AMNR nội tiếp đường tròn (O) nên ta có ARN   180 0 . Kết hợp với
  ASQ
trên, ta có ARN   180 0 .

• Tứ giác có tổng hai góc đối diện bằng 180 là tứ giác nội tiếp.
Tứ giác ARQS có ARN  ASQ
  180 0 suy ra ARQS là tứ giác nội tiếp hay bốn điểm A ;

R ; Q ; S cùng thuộc một đường tròn.

Câu 4. Giả sử các số của tập hợp X được sắp theo thứ tự (đánh số lại)
x1  x2    xn .
Ta có x1  x2  x1  x3    x1  xn  x2  xn  x3  xn    xn1  xn , suy ra đối với một tập n

số thực phân biệt bất kỳ ta luôn có ít nhất n  1  n  2  2n  3 giá trị phân biệt của các

tổng xi  x j .

Vậy C X   2n  3 .

Xét tập X1  1; 2; ; n , khi đó với mọi 1  i  j  n


Tài liệu sưu tầm và tổng hợp bản word đầy đủ liên hệ 0393732038 TÀI LIỆU TOÁN HỌC
114
Website:tailieumontoan.com
xi  x j  i  j  3; 4; ; 2n  1  C X1   2n  3

Vậy C X  min  2n  3 .

nn  1 nn  1
Số các tổng xi  x j ( 1  i  j  n ) bằng , suy ra C X   .
2 2

Xét tập X2  2; 2 2 ;; 2n  , thì với mọi 1  i  j  n

xi  x j  2 i  2 j .

Giả sử tồn tại 1  r  s  n : xr  xs  xi  x j  2r  2 s  2i  2 j

2 r |2 i nn  1
 2 r 1  2 sr   2 i 1  2 ji    i r  r  i  s  j  C X2   .
2 |2 2

nn  1
Vậy C X   .
max 2

Đề số 17

Câu 1.
1). Hệ phương trình tương đương với

 x  1 y   x  1  2  y
 2 


 x  1 y 2  1  2  y (1)
  .
 y  2 x 2   y  2)  x  1 


 


 y  2  x 2
 1  x  1 (2)

+) Nếu x  1 , suy ra  x  1 y 2  1  0 nên từ (1)  2  y  0


 y  2   y  2 x 2  1  0 ;
Do đó từ (2)  x  1  0  x  1 (mâu thuẫn).
+) Nếu x  1 , tuơng tự suy ra x  1 (mâu thuẫn).
+) Nếu x  1  y  2 (thỏa mãn).
Đáp số: x  1; y  2
Nhận xét: Bài toán sử dụng kỹ thuật đánh giá theo miền nghiệm khi đoán trước được
nghiệm của hệ phương trình.
Nhắc lại kiến thức và phương pháp:
• Cho hai biểu thức f  x; y và g  x; y , trong đó g  x; y  0 .
Xét biểu thức: P  f  x; y.g  x; y .
Có hai trường hợp sau xảy ra đó là P  0  f  x; y  0 và P  0  f  x; y  0 .
• Kỹ thuật nhẩm nghiệm.
Ý tưởng: Bài toán này không phải là một hệ phương trình đồng bậc, nếu là đồng bậc hai
thì ta có thể giải quyết bằng cách đưa về hệ số bất định. Nhưng một điều đáng lưu ý ở bài
toán này đó chính là các biểu thức x  1; y  2 được gắn với hai đại lượng không âm. Nên

Tài liệu sưu tầm và tổng hợp bản word đầy đủ liên hệ 0393732038 TÀI LIỆU TOÁN HỌC
115
Website:tailieumontoan.com

nhiều khả năng sẽ xảy ra  x  1 y 2   y  2 x 2  0 . Xét các trường hợp thì thấy  x; y  1; 2
là nghiệm duy nhất của hệ phương trình. Hoặc ta có thể sử dụng kỹ thuật nhẩm nghiệm
như sau, đó là giả sử x  k , bây giờ ta sẽ thay thử các giá trị của k , tất nhiên sẽ lấy các giá
trị k nguyên và đẹp. Và cũng cho ta được nghiệm  x; y  1; 2 . Với cặp nghiệm này, thực
x  y  3 x  1  y  2  0
chất bài toán quy về giải hệ phương trình    .
 y  x  1  y  2  x  1
Vì thế ta tách hệ phương trình ban đầu, và nhóm nhân tử như sau:
• Hệ phương trình đã cho
 x  1 y 2  x  1  y  2  0  x  1 y 2  1  2  y 1

  
 y  2 x 2  y  2  x  1  y  2 x  1  x  1  2
2

• Đến đây, ta sẽ đánh giá miền nghiệm:
1   x  1 y 2  1  0  2  y  0  y  2

TH1. Nếu x  1   .
2   y  2 x 2  1  0  y  2

Hệ bất phương trình này vô nghiệm.
1   x  1 y 2  1  0  2  y  0  y  2

TH2. Nếu x  1   .
2   y  2 x 2  1  0  y  2

Hệ bất phương trình vô nghiệm
• Vậy x  1; y  2 là nghiệm duy nhất của hệ phương trình.
Bài toán kết thúc.
Bài tập tương tự:

2  x  1 y  x  y  4
2

1. Giải hệ phương trình  .



 y  3 x 2  y  x  2

Đáp số:  x; y  1; 3 .

2). Điều kiện x  0 .


3
Phương trình tương đương 2  x  1 x   x2  7 .
x
Chia hai vế cho x  0 , ta được
 1 3 7  3  1 3 4  3  3 2 
2 1   x   x    x    2 1   x    0   x   2 x     0 .
 x x x  x  x x x  x  x x 

3 3 x  1
+ Giải x   2  x   4  x 2  4 x  3  0   .
x x x  3
3 2 3 4
+ Giải x    x   2  x 3  3x  4  0
x x x x
  x  1 x 2  x  4  0  x  1 .
Đáp số x  1; x  3 .
Nhận xét: Bài toán sử dụng phương pháp ẩn phụ không hoàn toàn, sau đó nâng lũy thừa
tìm nghiệm của phương trình ban đầu.
Nhắc lại kiến thức và phương pháp:
Tài liệu sưu tầm và tổng hợp bản word đầy đủ liên hệ 0393732038 TÀI LIỆU TOÁN HỌC
116
Website:tailieumontoan.com
• Phương pháp đặt ẩn phụ không hoàn toàn: xét một phương trình bậc hai có dạng
mf  x.t 2  ng  x.t  k  0 (*), trong đó t là ẩn phụ được biểu diễn dưới dạng t  h  x .
Khi đó, ta có
2
t  ng  x  4 kmf  x , với t bắt buộc là một số chính phương. Nên ta tìm được
 
nghiệm của (*) đó là
ng  x  t ng  x  t
t  h  x ; t   h  x .
mf  x mf  x

 f  x ; g  x  0

• Cách giải phương trình f  x  g  x   .




f  x   g 2
 x 
Ý tưởng: Trước hết, ta cần quy đồng mẫu số bài toán, như vậy ta sẽ được phương trình có
dạng f  x. g  x  h  x và nếu nâng lũy thừa hai vế, ta sẽ thu được một phương trình bậc
5. Và phương trình bậc 5 nếu không có nghiệm nguyên thì sẽ rất khó để giải quyết. Vậy
3
nên ta cần nghĩ đến hướng tư duy khác, đó là bài toán xuất hiện căn thức x nên ta
x
2
 3 
mong muốn sẽ tạo được lượng k  x   để có thể đưa về phương trình bậc hai, sau đó
 x 
3
đặt t  x  để sử dụng phương pháp ẩn phụ không hoàn toàn. Tức là sẽ chia cả hai vế
x
của phương trình, ta được:
3 x2  7 3
• Ta có x    2  x  1 x   x 2  7
x 2( x  1) x
 1 3 7 3  1 3 4
 2 1   x   x   x   2 1   x    0 (*).
 x x x x  x x x
3  1 4
• Đặt t  x   0 , khi đó ta có (*)  t 2  2 1  t   0 .
x  x x
2 2
 1 4  1
Có 't  1     1   nên suy ra được
 x  x  x 

 1
t  1   1 
1 t  2  x 3 2 i 
 x x  x
 2   .
 1 1  t
t  1   1    x 3  2
 x x
x  ii
 x x
3 x  1
• Giải ( i ), ta có ( i )  x   4  x 2  4 x  3  0   .
x x  3
3 4
• Giải ( ii ), ta có ( ii )  x   2  x 3  3x  4  0  x  1 .
x x
Bài toán kết thúc.
Bài tập tương tự:
1. Giải phương trình:  x  3 4  x12  x  28  x .
Đáp số: x  4  
2  1 ; x  31  3 .

Tài liệu sưu tầm và tổng hợp bản word đầy đủ liên hệ 0393732038 TÀI LIỆU TOÁN HỌC
117
Website:tailieumontoan.com

2. Giải phương trình: x3  x  2x2  x  2 .


1 5 1  65
Đáp số: x  ; x .
2 8
Câu 2.
1). Giả sử tồn tại các số nguyên x , y , z thỏa mãn
x 4  y 4  7 z 4  5  x 4  y 4  z 4  8 z 4  5 (1) .
Ta có a 4  0,1 (mod 8) với mọi số nguyên a
x 4  y 4  z 4  0,1,2,3 (mod 8)
  4 .
8 z  5  5(mod 8)

Mâu thuẫn với (1).
Vậy không tồn tại  x; y; z thỏa mãn đẳng thức.
Nhận xét. Để giải bài toán trên cần sử dụng phương pháp phản chứng: “Giả sử xảy rồi
biến đổi thấy điều mâu thuẫn với giả sử”.
Nhắc lại kiến thức và phương pháp.
• Thêm cùng một lượng vào hai vế của đẳng thức ta được một đẳng thức mới tương
đương với đẳng thức ban đầu
x4  y 4  7 z4  5  x4  y 4  z4  8z4  5 .
• Lũy thừa bậc bốn của một số nguyên khi chia cho 8 dư 0 hoặc 1 tức là a 4  0, 1 mod 8
với mọi số nguyên a
 4
x  0, 1 mod 8  4
x  y  z  0, 1, 2, 3 mod 8
4 4
 4
 y  0, 1 mod 8   4 .
 8 z  5  5 mod 8
z  0, 1 mod 8
4 


• Hai vế của một đẳng thức có số dư khi chia cho cùng một số nhận được số dư khác
nhau thì đẳng thức này không thể tồn tại.
Ta thấy VT  x 4  y 4  z 4 chia 8 dư 0 hoặc 1 hoặc 2 hoặc 3 .
Mà VP  8 z 4  5 chia 8 dư 5 do đó không thể tồn tại đẳng thức x 4  y 4  z 4  8 z 4  5
hay x 4  y 4  z 4  8 z 4  5 .
Vậy không tồn tại các bộ ba số nguyên  x; y; z thỏa mãn đẳng thức
x4  y 4  7 z4  5 .

2). Phương trình tương đương với


 2  2
 x  1   x  1   x  1   x  1   y  2 x  24 x  y  8 x  8 x  y .
2 2 3 2 3 3 3

  
3 3 3
+) Nếu x  1  8 x 3  8 x 3  8 x  2 x  1  2 x  y 3  2 x  1 (mâu thuẫn vì y nguyên).
+) Nếu x  1 và  x; y là nghiệm, ta suy ra x;  y cũng là nghiệm, mà x  1  mâu
thuẫn.
+) Nếu x  0  y  0 (thỏa mãn).
Vậy x  y  0 là nghiệm duy nhất.
Nhận xét. Để giải bài toán trên cần sử dụng phương pháp biến đổi tương đương đưa về
xét khoảng giá trị của nghiệm.
Tài liệu sưu tầm và tổng hợp bản word đầy đủ liên hệ 0393732038 TÀI LIỆU TOÁN HỌC
118
Website:tailieumontoan.com
Nhắc lại kiến thức và phương pháp.
• Hằng đẳng thức A 2  B2   A  B A  B
4 4  2 2  2 2
VT   x  1   x  1   x  1   x  1   x  1   x  1 
  
2
• Hằng đẳng thức  A  B  A 2  2 AB  B2
VT   x 2  2 x  1  x 2  2 x  1 x 2  2 x  1  x 2  2 x  1

 2 x 2  24 x  8 x 3  8 x .

Khi đó phương trình đã cho tương đương với phương trình 8 x 3  8 x  y 3


Giải phương trình nghiệm nguyên bằng cách xét khoảng giá trị của nghiệm
• Không tồn tại lũy thừa bậc ba nào giữa hai lập phương (lũy thừa bậc ba) liên tiếp.
3 3 3
+) Nếu x  1  8 x 3  8 x 3  8 x  2 x  1  2 x  y 3  2 x  1
3 3
(mâu thuẫn vì y nguyên và 2x và 2 x  1 là hai lập phương liên tiếp).
3 3 3
+) Nếu x  1  8 x 3  8 x 3  8 x  2 x  1  2 x  y 3  2 x  1
3 3
(mâu thuẫn vì y nguyên và 2x và 2 x  1 là hai lập phương liên tiếp).
+) Nếu x  0  y  0 (thỏa mãn).
Vậy x  y  0 là nghiệm duy nhất.

Câu 3.

 , suy ra OBD
1). Tứ giác OBCD nội tiếp và CO là phân giác góc BCD   OCD
  OCB
  ODB
,

nên tam giác OBD cân tại O , do đó OB  OD (1).


  OBE
Tứ giác OBCD nội tiếp ODC  (cùng bù với góc OBC
 ) (2).

Trong tam giác CEF có CO vừa là đường cao vừa là đường phân giác nên tam giác
CEF cân tại C .
  AFC
Do AB  CF  AEB   EAB
 , suy ra tam giác ABE cân tại B , nên BE  BA  CD (3).

Từ (1), (2) và (3), suy ra OBE  ODC (c – g – c).

Tài liệu sưu tầm và tổng hợp bản word đầy đủ liên hệ 0393732038 TÀI LIỆU TOÁN HỌC
119
Website:tailieumontoan.com
Nhận xét. Có ba trường hợp bằng nhau cơ bản của hai tam giác. Ở bài này, chúng ta sử
dụng trường hợp bằng nhau “cạnh-góc-cạnh” từ đó ta sẽ đi tìm ra các cạnh và góc bằng
nhau.
Nhắc lại kiến thức và phương pháp.
• Hai góc nội tiếp cùng chắn một cung của một đường tròn thì bằng nhau.
  ODB
+ BCO  của đường tròn ngoại tiếp tam giác
 (hai góc nội tiếp cùng chắn cung OB
BCD ).
  CBD
+ OCD  của đường tròn ngoại tiếp tam giác
 (hai góc nội tiếp cùng chắn cung DO
BCD ).
  OCD
Mà BCO  (vì CO là tia phân giác của BCD ), suy ra OBD
  ODB
.
• Tam giác có hai góc bằng nhau là tam giác cân.
Tam giác OBD có OBD   ODB
 (chứng minh trên) nên OBD cân tại O .
• Tam giác cân có hai cạnh bên bằng nhau.
Tam giác OBD cân tại O suy ra OB  OD .
• Tứ giác nội tiếp có góc ngoài tại một đỉnh bằng góc trong tại đỉnh không kề với nó.
 là góc ngoài tại đỉnh B
Tứ giác BCDO nội tiếp đường tròn ngoại tiếp BCD có EBD
 là góc trong tại đỉnh D không kề B suy ra EBD
và CDO   CDO
.
• Tam giác có đường cao cũng là đường phân giác thì tam giác đó cân.
Tam giác CEF có CO vừa là đường cao vừa là đường phân giác nên tam giác CEF
cân tại C .
• Tam giác cân có hai góc kề cạnh đáy bằng nhau.
  CFE
Tam giác CFE cân tại C , suy ra CEF  hay AEB
  AFC.
• Một đường thẳng cắt hai đường thẳng song song tạo ra hai góc ở vị trí đồng vị bằng
nhau.
  EAB
AFC
  EAB
 (hai góc ở vị trí đồng vị của AB  FC ), suy ra AEB  , nên EBA cân tại

B (tam giác có hai góc bằng nhau), do đó BE  BA mà ABCD là hình bình hành nên
AB  CD suy ra BE  CD .
Xét OBE và ODC có: OB  OD ; EBD   CDO
 ; BE  CD suy ra OBE  ODC (c – g
– c).

2). Từ OBE  ODC  OE  OC .


Mà CO là đường cao tam giác cân CEF , suy ra OE  OF .
Từ đó OE  OC  OF , vậy O là tâm đường tròn ngoại tiếp tam giác CEF .
Nhận xét. Đường tròn ngoại tiếp tam giác cách đều ba đỉnh của tam giác do đó ta chứng
minh điểm O cách đều các điểm E ; C ; F hay OE  OC  OF .
Nhắc lại kiến thức và phương pháp.
• Hai tam giác bằng nhau có các cặp cạnh và cặp góc tương ứng bằng nhau.
OBE  ODC  OE  OC .
• Tam giác cân có đường cao cũng là đường trung trực của cạnh tương ứng
CO là đường cao của tam giác cân CFE nên CO là đường trung trực của FE .
• Một điểm thuộc trung trực của một đoạn thẳng thì cách đều hai mút của đoạn thẳng.

Tài liệu sưu tầm và tổng hợp bản word đầy đủ liên hệ 0393732038 TÀI LIỆU TOÁN HỌC
120
Website:tailieumontoan.com
Điểm O thuộc đường trung trực CO của đoạn thẳng FE nên OE  OF , suy ra
OE  OC  OF .
• Điểm cách đều ba đỉnh của một tam giác là tâm đường tròn ngoại tiếp tam giác đó.
Ta có OE  OC  OF suy ra O là tâm đường tròn ngoại tiếp CEF .

3). Theo trên, ta có BE  CD mà CE  CF  BC  DF .


 , nên
Ta có CI là đường phân giác góc BCD
IB CB DF
   IB.BE  ID.DF .
ID CD BE
Mà CO là trung trực EF và I  CO , suy ra IE  IF .
Từ hai đẳng thức trên, suy ra IB.BE.EI  ID.DF.FI .
Nhận xét. Chứng minh một đẳng thức ta kết hợp các đẳng thức đã cho, đã chứng minh
được để ghép vào đẳng thức cần chứng minh.
Nhắc lại kiến thức và phương pháp.
Ta có BE  CD (chứng minh trên) và CE  CF ( ECF cân tại C ) suy ra
CB DF
CE  BE  CF  CD  BC  DF suy ra  .
CD BE
• Đường phân giác trong của một tam giác chia cạnh dối diện thành hai đoạn thẳng có
tỷ số bằng với tỷ số hai cạnh tương ứng của tam giác.
 của tam giác CBD nên IB  CB , suy ra ta
Ta có CI là phân giác của góc trong BCD
ID CD
IB DE
được   BE.BI  DE.DI .
ID BE
• Nhắc lại kiến thức.
Ta có I nằm trên trung trực CO của đoạn thẳng FE nên suy ra IE  IF nhân vế theo
vế với đẳng thức BE.BI  DE.DI ta được BE.BI .IE  DE.DI .IF

x3 x2
Câu 4. Ta chứng minh  (1) .
x3  8 y 3 x2  2 y 2

x3 x4 2
  x 2  2 y 2   x  x 3  8 y 3   4 x y  4 y  8 xy
2 2 4 3
 3 3
 2
x  8y x  2 y 
2 2

 x 2  y 2  2 xy (đúng).

y3 y2
Ta chứng minh 3
 (2).
y 3   x  y x2  2 y 2

y3 y4  3 2
  x 2  2 y 2   y  y 3   x  y    x 2  2 y 2   y 4  y  x  y
3
 3
 2
 2y2   
y 3   x  y x 2

  x 2  y 2  x 2  3 y 2   y  x  y
3

1 2
Ta có x 2  y 2   x  y
2

Tài liệu sưu tầm và tổng hợp bản word đầy đủ liên hệ 0393732038 TÀI LIỆU TOÁN HỌC
121
Website:tailieumontoan.com
x 2  3 y 2  x 2  y 2  2 y 2  2 xy  2 y 2  2 y  x  y
1
  x 2  y 2  x 2  3 y 2  
2 3
 x  y .2 y x  y  y  x  y  (2) (đúng).
2
Từ (1) và (2), suy ra P  1 .
Dấu “=” xảy ra khi và chỉ khi x  y .
Vậy Pmin  1 .
Nhận xét: Bài toán sử dụng phương pháp dự đoán điểm rơi, từ đó phát hiện tư duy bất
đẳng thức phụ cần thiết để tìm giá trị nhỏ nhất của bài toán.
Nhắc lại kiến thức và phương pháp:
• Bất đẳng thức Cosi cho hai số thực dương x  y  2 xy .
1 2
• Hệ quả của bất đẳng thức Cosi, đó là: x 2  y 2   x  y .
2
Ý tưởng: Đây là một bài toán chứa các biểu thức đồng bậc, nên điểm rơi của bài toán sẽ có
dạng x  ky . Từ đó thay ngược lại biểu thức P , ta có:
k3 4 k 4
P 3
 k  .
k 8 3
1   k  1
3
k 8 1   k  1
3

3
Các biểu thức k 3  8; 1   k  1 đều xuất hiện trong căn, nên ta mong muốn nó là một số
3 
m  3
phương trình và bằng nhau, do đó k 3  8  1   k  1 và k 3  8  m2 nên dễ thấy  .


 k  1
Tức là điểm rơi của bài toán tại x  y và giá trị nhỏ nhất của P là 1 . Việc dự đoán điểm
rơi này là cần thiết, nó giúp ta có nhiều sự lựa chọn hơn trong các việc đánh giá như
1 2
x  y  2 xy hay x 2  y 2   x  y . Với điểm rơi đó, hai mẫu số trong đã bằng nhau P
2
nên nếu có đánh giá P , ta cần đánh giá hai căn về hai biểu thức cùng mẫu, ví dụ như cùng
mẫu số x  2 y; 2 x  y; x 2  2 y 2 ; 2 x 2  y 2 . Bây giờ, quan sát từng căn thức một:
x3 x
• Với f  x; y  3 3
x 3 , trong căn thức mẫu số có bậc ba, trên tử là bậc
x  8y x  8y3
nhất, vậy để đồng hóa bậc thì ta cần đánh giá căn thức về một biểu thức dạng bậc
không trên bậc nhất . Hơn nữa lại xuất hiện 8y 3 nên ta sẽ chọn đánh giá x  2 y để tối
thiểu hóa ẩn y , tức là ta sẽ cần chứng minh:
x 1 2
  x  x  2 y  x 3  8 y 3  y  y  x x  2 y  0 .
3
x  8y 3
x  2y
Nhưng điều này chưa hoàn toàn đúng, vì cần phải có điều kiện y  x , vậy nên hướng
tư duy như trên chưa đúng. Tức là ta sẽ lựa chọn biểu thức x 2  2 y 2 thay vì x  2 y như
ta đã chọn, vì thế:
x x 2
3 3
 2 2
 xx2  2 y 2   x2 x3  8 y 3 
x  8y x  2y
2
 xy 2  x  y  0 .

Tài liệu sưu tầm và tổng hợp bản word đầy đủ liên hệ 0393732038 TÀI LIỆU TOÁN HỌC
122
Website:tailieumontoan.com

4y3 4y
• Với g  x; y  3
y 3
, với hướng tư duy tương tự, chúng ta sẽ
y   x  y
3
y   x  y
3

có:
4y3 2y2
  x 2  y 2  x 2  3 y 2   y  x  y .
3
3
 2 2
y 3   x  y x  2y

Điều này luôn đúng do theo bất đẳng thức Cosi, ta có:

 1 2
x  y   x  y
 2 2
  x 2  y 2  x 2  3 y 2   y  x  y .
3
 2

 2 2 2 2 2 2
x  3 y  x  y  2 y  2 xy  2 y


Bài toán kết thúc.
Bài tập tương tự:
1. Cho a; b là hai số thực dương thỏa mãn a  b  2 . Tìm giá trị nhỏ nhất của biểu thức
2 a 2  3b 2 2 b 2  3 a 2
P  .
2 a 3  3b3 2b3  3a 3
2. Cho a; b là hai số thực dương. Chứng minh rằng:
 2     
a  b  3 b2  a  3   2 a  1 2b  1  .
 4  4   
2  2 

Đề số 18

Câu 1.
1). Điều kiện 0  x  1 .
Phương trình tương đương với
3
x3  x
 
1 x  1  1  3  
1 x  1  x  x  3 .

 
Nếu 0  x  1  3 1  x  1  3 , đồng thời x  x  3  1  4  3,
suy ra VT  VP (loại).
Thử lại ta thấy x  1 là nghiệm.
Nhận xét: bài toán kết hợp giữa phương pháp nhân liên hợp và phương pháp đáng giá để
tìm nghiệm của phương trình.
Nhắc lại kiến thức và phương pháp:
• Biểu thức liên hợp x  m  x   xm x  xm  x 
m
 xm x  với x  0; x  m  0 .
xm  x

m  f  x  0
• Đánh giá: m  f  x  n  g  x  h  x với  .

n  2 m

Tài liệu sưu tầm và tổng hợp bản word đầy đủ liên hệ 0393732038 TÀI LIỆU TOÁN HỌC
123
Website:tailieumontoan.com
 m  f x  n  n
  
Ta có: 0  f  x  m   , suy ra phương trình vô nghiệm. Vậy x  m
 g  x  h  x  2 m

là nghiệm của phương trình đã cho.
Ý tưởng: Bài toán xuất hiện ba căn thức nằm trong một tích, sẽ rất khó để định hình ra
hướng giải, ẩn phụ sẽ rất phức tạp. Nhưng nếu xét hai căn thức đầu tiên ta thấy
2 2
 x3   x  3 . Vì thế ta sẽ nghĩ ngay đến chuyện dùng hằng đẳng thức dạng

a 2  b2  a  ba  b . Khi đó phương trình đã cho tương đương với:


3
x3  x
 
1  x  1  1  3 1  x  3  x  3  x ( i ).

Với phương trình ( i ), ta sẽ đi nhẩm một vài giá trị nghiệm đẹp thỏa mãn các yêu cầu là
1  x  0 và các biểu thức trong căn thức là số chính phương vì thế ta khẳng định nó có
nghiệm duy nhất x  1 , đồng thời 1 lại là miền chặn của biến do đó ta sẽ đi đánh giá
phương trình ( i ). Tức là với 0  x  1 ta sẽ đi chứng minh ( i ) vô nghiệm như sau:


3 1  x  3  3
0  x 1  VT  VP  ( i ) vô nghiệm.

 x  3  x  3


Vậy ta kết luận x  1 là nghiệm của phương trình đã cho.
Bài toán kết thúc.
Bài tập tương tự:
3. Giải phương trình  x  2  x 1  
2  x  1  1.
Đáp số: x  2 .
4. Giải phương trình  x 1 x2  
3 x 1  1.
Đáp số: x  3 .

2).
+ Xét x  y  0 là nghiệm.
+ Xét x  0; y  0 hệ phương trình tương đương với
1
 1 1
 1

  2 2 
  2 2 (1)

 x 2
y 
x
2
y

   .
 1 1 
   1   1 1 
   2 
1    4 
 x  y 
 2    8 (2)
 x  y 


 xy  

 xy 
 1 1
3
   2
 1 1  x y
Thay (1) vào (2) ta thu được     8  
  x y 1.
 x y   1
  1
 xy
Nhận xét: bài toán sử dụng phép chia các biến, sau đó kết hợp hai phương trình tìm mối
liên hệ giữa hai biến để tìm nghiệm của hệ phương trình.
Ý tưởng: quan sát thấy hệ phương trình có dáng dấp của hệ phương trình đối xứng loại
hai, tức là sự xuất hiện của tổng x  y và tích xy sẽ làm ta nghĩ đến phép đặt Viet là
S  x  y ; P  xy để từ đó giải hệ hai ẩn S , P . Nhưng nếu làm như thế ta thu được hệ

Tài liệu sưu tầm và tổng hợp bản word đầy đủ liên hệ 0393732038 TÀI LIỆU TOÁN HỌC
124
Website:tailieumontoan.com
 2 2
S  2 P  2 P


S 1  P  4 P 2 , hệ này giải bằng phương pháp thế sẽ thu được phương trình lũy thừa bậc
 


4 phức tạp, vì thế ta sẽ nghĩ đến phương án khác đó chính phương pháp “ chia để trị “.
Trước hết là xét phương trình hai, vế trái của nó xuất hiện tích đồng thời vế phải cũng
xuất hiện tích số dạng xy.xy . Do đó ta sẽ nghĩ đến việc chia một biểu thức bên vế trái cho
 1 1  1 1 1
xy và ta được như sau:   1    4 . Phương trình này chủ đạo là hai biến ;

 x y  xy  x y vì
1 1
vậy ta cũng chia phương trình một để xuất hiện hai biến này, đó là x 2  y 2  2 . Và nếu đặt
ax  1 a 2  b2  2 a 2  b2  2
  
by  1 hệ phương trình đã cho trở thành a  b1  ab  4 a  b2  2 ab  4
 
a  b  2
2 2
a  b  2
2 2

  
a  ba 2  2 ab  b2   4 a  b3  8
 
 2 2
 a  b  2  a  b  1  1  1  1  x  y  1


a  b  2 x y

Bài toán kết thúc.
Bài tập tương tự:

 2 2

 x  1  y  1  27 xy
1. Giải hệ phương trình  2 .

 x  1 y 2  1  10 xy


1   1 
2 
 
Đáp số:  x; y   ; 2  3  , 2; 2  3 , 2  3;

.
2 
 
3
 3 1 
 x  y 1    16
3 

  xy 
2. Giải hệ phương trình   2 .
 2  1 
 x  y 1    8
2 


 xy 
Câu 2.
 1 1
1). Ký hiệu K   3 n    , do n  1  K  1 .
 27 3 
3 3
1 1  1 1  2
Ta có K  n    K  1  K    n   K  
3
27 3  3  27  3 
K 1 1 4 8
 K3  K2    n   K 3  2K 2  K 
3 27 27 3 27
K 4 1 3
 K 3   n  K 2  K 3  3K 2  K   K 3  n  K 2  K  1
3 3 3
2
 1 1
suy ra n  K 2  n   3 n    không biểu diễn được dưới dạng lập phương của một số
 27 3 
nguyên dương.
Nhận xét. Áp dụng kiến thức về phần nguyên, quan hệ giữa phần nguyên của một số với
số đó.

Tài liệu sưu tầm và tổng hợp bản word đầy đủ liên hệ 0393732038 TÀI LIỆU TOÁN HỌC
125
Website:tailieumontoan.com
Nhắc lại kiến thức và phương pháp.
• Áp dụng giả thiết của bài toán đề tìm ra điều kiện.
1 1
Ta có n là số nguyên dương nên n  1 khi đó 3 n  1.
27 3
• Giá trị phần nguyên của một số không vượt qua số đó.
1 1  1 1
3 n   1 nên  3 n     1 hay K  1 .
27 3  27 3 
• Một số thực bất kỳ có giá trị lớn hơn hoặc bằng giá trị của phần nguyên của nó và nhỏ
hơn số nguyên liền sau số nguyên biểu diễn phần nguyên của số thực đó.
Ta có
   
 3 n 1  1  3 n 1  1   3 n 1  1  1
 27 3  27 3  27 3 

1 1
 K  3 n   K  1.
27 3
3
Tiếp tục biến đổi như trên bài giải ta được K 3  n  K 2  K  1 .
• Không tồn tại một lập phương nào giữa hai lập phương liên tiếp.
3 3
Ta có K 3 và K  1 là hai lập phương liên tiếp. Mà K 3  n  K 2  K  1 nên n  K 2
không thể viết được dưới dạng lập phương của một số nguyên dương.
2
 1 1
Vậy với mọi số nguyên dương n , biểu thức n   3 n    không biểu diễn được
 27 3 
dưới dạng lập phương của một số nguyên dương.

2). Ta có 6  x 2  5  6  y 2  5  z 2  5

 6  x  y x  z  6  y  z y  x  z  xz  y


3  x  y   2  x  z  3  x  y   2  y  z   z  x   z  y 
  
2 2 2
9x  9 y  6z 3
   3 x  3 y  2 z ,
2 2
3x  3 y  2 z 2
suy ra P  
6  x 2  5  6  y 2  5  6  z 2  5 3
Đẳng thức xảy ra khi x  y  1; z  2
2
Vậy Pmin  .
3
Nhận xét: Bài toán sử dụng phép thế từ giả thiết và bất đẳng thức Cosi cho hai số thực
dương để tìm giá trị nhỏ nhất của biểu thức.
Ý tưởng: Quan sát thấy, bài toán có sự đối xứng giữa hai kiến x; y nên điểm rơi sẽ xảy ra
tại x  y  kz . Thế lại giả thiết ta sẽ tìm được x  y  1; z  2 . Giả thiết cho xy  yz  zx  5
đồng thời số 5 này cũng xuất hiện ở biểu thức P nên ta sẽ nghĩ đến chuyện thế giả thiết
vào P . Khi đó ta có:

Tài liệu sưu tầm và tổng hợp bản word đầy đủ liên hệ 0393732038 TÀI LIỆU TOÁN HỌC
126
Website:tailieumontoan.com
3x  3 y  2 z
P .
6  x  y x  z  6  y  z y  z  z  xz  y
Và điều ta cần là sử dụng đánh giá nào đó để triệt tiêu tử số và mẫu số, tức là tìm số m
thỏa mãn:
6  x  y x  z  6  y  z y  z  z  xz  y  m3x  3 y  2 z .
1
Thì lúc đó P  . Câu chuyện tiếp theo là tìm m , quan sát thấy các biểu thức trong căn
m
đều là tích của hai thừa số dương, ta sẽ sử dụng bất đẳng thức Cosi cho hai số thực dương
nhưng phải thỏa mãn điều kiện điểm rơi. Với căn thức cuối, với điểm rơi
x  y  z  x  z  y thì ta có ngay rằng
x  y  2z
 z  x z  y  .
2
Cũng với tư duy đó, ta sẽ thấy:
3  x  y   2  x  z
6  x  y x  z 
2
3  y  z   2  y  x
6  y  z y  x 
2
3
Nên 6  x  y x  z  6  y  z y  z   z  x z  y  3x  3 y  2 z .
2
2 2
Tức là P   Pmin  tại x  y  1; z  2 .
3 3
Bài toán kết thúc.
Bài tập tương tự:

1. Cho a; b; c  0 và thỏa mãn a  b  c  1 . Chứng minh rằng:


bc ca ab 1
P    .
a  bc b  ca c  ab 2
2. Cho a; b; c  0 và thỏa mãn a  b  c  1 . Chứng minh rằng:
bc ca ab 3
P    .
a  bc b  ca c  ab 2

Câu 3.

Tài liệu sưu tầm và tổng hợp bản word đầy đủ liên hệ 0393732038 TÀI LIỆU TOÁN HỌC
127
Website:tailieumontoan.com
B P C

M
I N

A D

Q
  BPM
1). Tứ giác BPIM nội tiếp và AD  BC , suy ra MAD   BIM
 , nên tứ giác AMID nội

tiếp.
Tương tự tứ giác DNIA nội tiếp.
Vậy các điểm A; M ; I ; N ; D thuộc một đường tròn ( K ) .
Nhận xét. Chứng minh năm điểm cùng thuộc một đường tròn ta chứng minh cho bốn
điểm trong đó tạo thành một tứ giác nội tiếp.
Nhắc lại kiến thức và phương pháp.
• Hai góc nội tiếp cùng chắn một cung của một đường tròn thì bằng nhau.
  BIM
BPM  của đường tròn ngoại tiếp tứ giác
 (hai góc nội tiếp cùng chắn cung BM
BPIM ).
• Một đường thẳng cắt hai đường thẳng song song tạo ra các cặp góc so le trong bằng
nhau.
  MAD
BPM  (hai góc so le trong của BP  AD ), suy ra BIM
  MAD
.

• Tứ giác có góc ngoài tại một đỉnh đối diện bằng góc trong không kề với nó thì tứ giác
đó là tứ giác nội tiếp.
 là góc ngoài tại đỉnh I và góc MAD
Tứ giác MIDA có góc BIM  là góc trong tại đỉnh

A không kề với I thỏa mãn BIM   MAD


 nên suy ra MIDA là tứ giác nội tiếp.
Hoàn toàn tương tự ta chứng minh được DNIA là tứ giác nội tiếp.
Do đó 5 điểm A; M ; I ; N ; D cùng thuộc một đường tròn.

2). Do các tứ giác BPIM và CPIN nội tiếp nên ta có QMI   BPI
  CNI  , suy ra tứ giác MINQ

nội tiếp.
Mà M ; I ; N  ( K ) , suy ra tứ giác MINQ nội tiếp đường tròn ( K ) .
Vậy Q thuộc đường tròn ( K ) .
Nhận xét. Chứng minh một điểm nằm trên một đường tròn ta chứng minh điểm đó cùng
ba điểm nằm trên đường tròn tạo thành một tứ giác nội tiếp.
Nhắc lại kiến thức và phương pháp.
• Tứ giác nội tiếp có góc ngoài tại một đỉnh bằng góc trong tại đỉnh không kề với nó.

Tài liệu sưu tầm và tổng hợp bản word đầy đủ liên hệ 0393732038 TÀI LIỆU TOÁN HỌC
128
Website:tailieumontoan.com
+ Tứ giác BPIM là tứ giác nội tiếp có góc ngoài tại đỉnh M và góc trong tại đỉnh I
  BPI
không kề với nó nên QMI .

+ Tứ giác CPIN là tứ giác nội tiếp có góc ngoài tại đỉnh P và góc trong tại đỉnh N
  CNI
không kề với nó nên BPI   CNI
 , suy ra QMI  .

• Áp dụng nhắc lại kiến thức.


 là góc trong tại đỉnh
 là góc ngoài tại đỉnh N và góc QMI
Tứ giác MINQ có góc INC
  QMI
M không kề với N thỏa mãn INC  nên suy ra MINQ là tứ giác nội tiếp.

• Qua ba điểm không thẳng hàng xác định được một và chỉ một đường tròn.
Bốn điểm M ; I ; N ; Q cùng thuộc một đường tròn mà ba điểm M ; I ; N cùng thuộc
đường tròn ( K ) nên suy ra Q cũng thuốc đường tròn ( K ) .

  PIC
3). Khi P; I ; Q thẳng hàng, kết hợp với Q thuộc đường tròn ( K ) ta có AIQ  (đối đỉnh);
  PNC
PIC  (do tứ giác NIPC nội tiếp).
  QND
PNC  (đối đỉnh); QND
  QID
 (do tứ giác INDQ nội tiếp).
   .
 AIQ  QID , suy ra IQ là phân giác DIA nên IP là phân giác góc BIC
PB IB ID IB  ID BD PB BD
Do đó       .
PC IC IA IC  IA AC PC CA
Nhận xét. Chứng minh đẳng thức là tỷ số của các đoạn thẳng ta thường sử dụng mối liên
hệ giữa các đoạn thẳng thông qua định lý Ta-lét, tính chất đường phân giác trong/ngoài
tam giác, tính chất của tỷ lệ thức.
Nhắc lại kiến thức và phương pháp.
• Hai góc đối đỉnh thì bằng nhau.
 và góc PIC
+ Góc AIQ   PIC
 là hai góc đối đỉnh nên AIQ .
 và góc QND
+ Góc PNC  là hai góc đối đỉnh nên PNC
  QND
.
• Áp dụng nhắc lại kiến thức.
  PNC
+ PIC  của đường tròn ngoại tiếp tứ giác
 (hai góc nội tiếp cùng chắn cung PC
PINC ).
  QID
+ QND  (hai góc nội tiếp cùng chắn cung QD
 của đường tròn ngoại tiếp tứ giác
 
QNID ), suy ra AIQ  QID .
• Một tia nằm giữa hai tia và chia góc tạo bởi hai tia đó thành hai góc bằng nhau thì tia
này là tia phân giác.
Ta có IQ nằm giữa IA và ID mà AIQ   QID
 suy ra IQ là phân giác của góc AID.
• Hai góc đối đỉnh có chung đường phân giác hay tia phân giác của góc này là tia đối
của tia phân giác góc kia.
+ IQ là phân giác của góc AID.
+ IQ là tia đối của tia IP .
 và góc BIC
Góc AID  là hai góc đối đỉnh, suy ra IP là phân giác của góc BIC

• Đường phân giác trong của một tam giác chia cạnh dối diện thành hai đoạn thẳng có
tỷ số bằng với tỷ số hai cạnh tương ứng của tam giác.
 của tam giác CBI nên PB  IB
Ta có IP là phân giác của góc trong BIC
PC IC

Tài liệu sưu tầm và tổng hợp bản word đầy đủ liên hệ 0393732038 TÀI LIỆU TOÁN HỌC
129
Website:tailieumontoan.com
• Định lý Ta-lét.
IB IC IB ID PB IB ID
Ta có BC  AD nên    , suy ra   .
ID IA IC IA PC IC IA
PB IB ID IB  ID BD PB BD
• Tỷ lệ thức       (điều phải chứng minh).
PC IC IA IC  IA AC PC AC

Câu 4. Giả sử A có n số, chúng ta xếp chúng theo thứ tự 1  x1  x2  x2    xn  100


(1).
Suy ra với mỗi k  1,2,3, , n  1 ta có xk1  xi  x j  xk  xk  2 xk (2), với 1  i , j  k.
Áp dụng kết quả 2 ta thu được x2  1  1  2 ; x3  2  2  4 ; x4  8 ; x5  16 ; x6  32 ;
x7  64 , suy ra tập A phải có ít nhất 8 phần tử.
+) Giả sứ n  8  x8  100 .
Vì x6  x7  32  64  96  x8  2 x7  x7  50.
Vì x5  x6  16  32  48  x7  2 x6  x6  25.
25
Vì x4  x5  8  16  24  25  x6  2 x5  x5  (mâu thuẫn).
2
+) n  9 ta có tập 1; 2; 3; 5; 10; 20; 25; 50; 100 thỏa mãn yêu cầu bài toán .
Đáp số: n  9 .
Nhận xét. Các bài toán dạng này chủ yếu đánh giá tư duy và không có cách giải tổng quát.
Nhắc lại kiến thức và phương pháp.
• Từ 1 đến 100 có 100 số tự nhiên. Tập hợp A là tập hợp con của tập  có phần tử nhỏ
nhất bằng 1 và lớn nhất bằng 100 nên tập hợp A không vượt quá 100 phần tử.
Tổng quát, tập hợp A có 2  n  100 phần tử, sắp xếp các phần tử này theo thứ tự
1  x1  x2  x2    xn  100

x  a
• Theo đề bài có x  a  b với x, a, b đều là thuộc tập hợp A nên ta có  do đó mỗi

x  b

k  1,2,3, , n  1 ta có xk1  xi  x j  xk  xk  2 xk với 1  i , j  k.
• Áp dụng kết quả xk1  xi  x j  xk  xk  2 xk ta được
x2  1  1  2 ; x3  2  2  4 ; x4  8 ; x5  16 ; x6  32 ; x7  64 ,
suy ra tập A phải có ít nhất 8 phần tử.
• Giả sử n  8 theo thứ tự giả sử ta suy ra được x8  100 .
• Áp dụng các kiến thức trên, cùng cách tính toán giải phương trình bậc nhất.
+ Vì x6  x7  32  64  96  x8  2 x7  x7  50.
+ Vì x5  x6  16  32  48  x7  2 x6  x6  25.
25
+ Vì x4  x5  8  16  24  25  x6  2 x5  x5  (mâu thuẫn).
2
• Vì A là tập hợp có ít nhất 8 phần tử mà xét trường hợp có 8 phần tử cho kết quả mâu
thuẫn nên tập hợp A có ít nhất 9 phần tử.
Với n  9 theo thứ tự giả sử ta suy ra được x9  100 từ đó ta tìm được một tập hợp
1,2,3,5,10,20,25,50,100 thỏa mãn yêu cầu bài toán.
Đáp số: n  9 .

Tài liệu sưu tầm và tổng hợp bản word đầy đủ liên hệ 0393732038 TÀI LIỆU TOÁN HỌC
130
Website:tailieumontoan.com

Đề số 19

Câu 1.

1) Cộng hai vế của hệ ta được (2x + 3y)2 = 25


Ta có hệ:

2 x + 3 y = 5 2 x + 3 y = −5
 2 và  2
x + y = 2 x + y = 2
2 2

−1
2) Điều kiện x ≥
2
Đặt 2 x + 1 = a (a ≥ 0); 4 x 2 − 2 x + 1 = b(b > 0)

Ta có (1- b)(a - 3) = 0

1
B = 1 thì x1 = 0; x 2 = ; a = 3 thì x3 = 4
2

Câu 2.

1) Phá ngoặc
(1 + x )(1 + y ) + 4 xy + 2(x + y )(1 + xy ) = 25. ⇔ ( xy + 1)
2 2 2
+ 2( x + y )(1 + xy ) + ( x + y ) 2 = 25
⇔ ( xy + 1 + x + y ) 2 = 25 ⇔ ( x + 1)( y + 1) 2 = 25
Vì x, y không âm nên (x + 1)(y + 1) = 5 ta có (x; y) = (0; 4) ; (4; 0)
k 2 + k +1 k2 k +1 k 1 1 1
2) Xét = + = + = 1− + (k ∈ N )
k (k + 1) k (k + 1) k (k + 1) (k + 1) k k +1 k
Thay k lần lượt từ 1 ta có
 3 7 n 2 + n + 1  1   n 
 + + ...  = n + 1 −  =  n +  = n (đpcm)
1 . 2 2 . 3 n (n + 1)   n + 1   n + 1 

Câu 3.

Tài liệu sưu tầm và tổng hợp bản word đầy đủ liên hệ 0393732038 TÀI LIỆU TOÁN HỌC
131
Website:tailieumontoan.com

1) Ta có
AC
cot ∠ACB = ⇔ AC = AB. cot 30o = 2 3R
AB
AB AB
sin ∠ACB = ⇒ BC = = 4R
BC sin 30o
1 1 1 1 1 1
= + = + = ⇒ AH = R 3
AH2 AB2 AC2 12R 2 4R 2 3R 2
2) Ta có ∠ACB = ∠HAB (cùng phụ với ∠CAH )
1
Mà ∠HAB = ∠HNB (cùng bằng số đo cung HB)
2
∠HBN = ∠ACB
Từ đó tứ giác CMNH nội tiếp. Tâm đường tròn nội tiếp CMNH thuộc trung trực của HC
cố định.
Câu 4.

Áp dụng bất đẳng thức Bu-nhi-a-cốp –xki ta có


a2 + 4 1
( a ;1 ) và (1; 4) ta có 17( a + 1) ≥ ( a + 4) ⇔ a + 1 ≥ (1); Dau :" =" ⇔ a =
2 4 2 2 4

17 2
b2 + 4 1
( b ;1 ) và (1; 4) ta có 17(b + 1) ≥ (b + 4) ⇔ b + 1 ≥ (1); Dau :" =" ⇔ b =
2 4 2 2 4

17 2
a2 + b2 + 8 5
Từ (1) và (2) ta có P ≥ (*) Mặt khác Từ GT ta có a + b + ab =
17 4
Lại áp dụng bất đẳng thức Cô – si ta có:

Tài liệu sưu tầm và tổng hợp bản word đầy đủ liên hệ 0393732038 TÀI LIỆU TOÁN HỌC
132
Website:tailieumontoan.com

 2 1
a + 4 ≥ a

 2 1 3 2 1 5 1 1
b + ≥ b ⇔ (a + b ) + ≥ (a + b + ab) = ⇔ a + b ≥ ; Dau :" =" ⇔ a = b =
2 2 2

 4 2 2 4 2 2
a + b
2 2

 ≥ ab
 2
1
+8
2 17
Thay Vào (*) ta có P ≥ =
17 2
17 1
Vậy Min( P ) = ⇔a=b=
2 2
Đề số 20

Câu I.

1
1) Điều kiện: x ≥ −
3
Với x = 1 là nghiệm của phương trình
Với x > 1 vế trái lớn hơn 4. Phương trình vô nghiệm
Với x < 1, vế trái nhỏ hơn 4. Phương trình vô nghiệm
Vậy nghiệm của phương trình là x = 1.
2) Giải hệ phương trình

 5x + 2y + 2xy =  5x2 + 2y2 + 2xy =


2 2
26 26
 ⇔ 

3x(+ 2x +)(
y x − y =)11. 
3x + 2x2 − 2xy + xy − y2 =11

 5x2 + 2y2 + 2xy =26


 2 (
⇒ 5x2 + 2y2 + 2xy + 2 2x2 + 3x − y2 − xy = 26 + 2.11 = 48)
2x + 3x − y − xy =
2
11

 x=2
⇔ 9x + 6x − 48 =0 ⇔ 
2
x = − 8
 3

Với x = 2 ta có:

 y =1
2.22 + 3.2 − y2 − 2y = 11 ⇔ y2 + 2y − 3 = 0 ⇔ 
 y = −3

8
Với x = − ta có
3
Tài liệu sưu tầm và tổng hợp bản word đầy đủ liên hệ 0393732038 TÀI LIỆU TOÁN HỌC
133
Website:tailieumontoan.com
2
 8  8 8 8 43
2  −  + 3  −  − y2 + y = 11 ⇔ y2 − y + = 0 Phương trình vô nghiệm
 3  3 3 3 9

Vậy nghiệm của hệ phương trình là (2; 1); (2; -3)

Câu II.

1) Giả sử n2 + 391 =
a 2 với a nguyên dương. Ta có
  n + a =1  n =−195
  (L)
 n − a =−391   a =196
( )(
n−a n+a = )
−391 ⇔ 
= n + a 391=
⇔
n 195
   TM ( )
  n − a =−1  a =196
 
Vậy số nguyên dương n thỏa mãn đề bài là 195.
2) Ta có

xy + z + 2x2 + 2y2 (
xy + z x + y + z + x + y)

1 + xy 1 + xy

( x + z )( y + z ) + x + y xy + z + x + y
≥ ≥ =
1
1 + xy 1 + xy
1
Dấu “=” xảy ra khi x= y= z=
3

Câu III.

1) Chứng minh rằng M là trực tâm của tam giác ABC.


Ta có các tứ giác BEPH và PHQM là tứ giác nội tiếp. Từ đó

Tài liệu sưu tầm và tổng hợp bản word đầy đủ liên hệ 0393732038 TÀI LIỆU TOÁN HỌC
134
Website:tailieumontoan.com

∠H1 =∠P1 =∠P2 =∠H2 mà ∠H2 =∠C1 (cùng phụ với ∠QHC )
∠H1 =∠C1 nên CM // EH ⇒ CM ⊥ AB tương tự BM ⊥ AC . Vậy M là trực tâm
của tam giác ABC.
2) Chứng minh rằng BEFC là tứ giác nội tiếp.
∠EBH = ∠HPF (cùng bù với góc ∠HPE )
∠HPF = ∠PFA ⇒ ∠EBH = ∠PFA
Vậy tứ giác BEFC nội tiếp.

Câu IV.

Số các số được đánh dấu ≥ 1

Nếu tất cả các số được đánh dấu là số dương ta có đpcm.

Nếu các số đánh dấu có số âm giả sử là a n thì số a n +1 là số dương cũng được đánh dấu và
a n + a n +1 > 0 , mọi số âm đều có số có tổng dương, các cặp số này không trùng nhau. Vậy
tổng các số được đánh dấu là dương.

Đề số 21

Câu 1.

1)ĐKXĐ: ∀x ∈ R
x 2 − x + 2 = 2 x 2 − x + 1 ⇔ x 2 − x + 1 − 2 x 2 − x + 1 + 1 = 0 ⇔ ( x 2 − x + 1 − 1) 2 = 0
⇔ x 2 − x + 1 = 1 ⇔ x( x − 1) = 0 ⇔ x = 0; hoac : x = 1
Phương trình có 2 nghiệm x1= 0;x2 = 1
2)
 x − y + xy = 1  y = 1 − x − xy  y 2 = 1 − x 2 − xy
2 2 2 2

 ⇔  2 ⇔ 
3 x + y = y 2 + 3  x + xy − 3 x − y + 2 = 0 ( x − 1)( x + y − 2) = 0
 y = 0
 x = 2 − y 
 y = 1 − x − xy
2 2
 2  x = 1
  y + 2 y − 3 = 0  x = 1
⇔  x = 1 ⇔ ⇔ 
 x = 2 − y 
x =1  y = 1
  y 2 − y = 0 
 x = 5
 y = −3
Vậy hệ có 3 nghiệm (x;y) = (1; 0); (1; 1); (5;-3)
Câu 2.

1) Ta có
13
( )
13 = 134 .13 ≡ 3(mod10);6 6 ≡ 6(mod10);2009 2009 = (2009 )
3 2008
.2009 ≡ 9(mod10)
1313 + 6 6 + 2009 2009 ≡ 3 + 6 + 9 ≡ 8(mod10)
Tài liệu sưu tầm và tổng hợp bản word đầy đủ liên hệ 0393732038 TÀI LIỆU TOÁN HỌC
135
Website:tailieumontoan.com
nên 1313 + 6 6 + 2009 2009 có tận cùng là 8
2)áp dụng BĐT Cô si cho 2 số dương
9a + 4a + 5b 13a + 5b
9a và 4a+5b ta có 9a (4a + 5b) ≤ =
2 2
9b + 4b + 5a 13b + 5a
9b và 4b+55 ta có 9b(4b + 5a ) ≤ =
2 2
3(a + b) 1 1
Nên P ≥ = nên Min(P) = khi a = b
18a + 18b 3 3
2
Cách khác áp dụng bất đẳng thức Bunhicôpsky 2 dãy
a; b và dãy 4a + 5b ; 4b + 5a ta có
( a (4a + 5b) + b(4b + 5a ) )
2
≤ (a + b)(9a + 9b) ⇔ a (4a + 5b) + b(4b + 5a ) ≤ 3(a + b)
a+b 1
⇔ ≥
a (4a + 5b) + b(4b + 5a ) 3
1
Min (P)=
3
4a + 5b 4b + 5a 4a + 5b 4b + 5a
khi = ⇔ = ⇔ 4ab + 5b 2 = 4ab + 5a 2 ⇔ a = b
a b a b
Câu 3.

1 O1
B H D

O2

a-Tam giác BKO1 dd tam giác O2KA nên


a O1 B O1 K O1 K
= = = = TgB1
b O2 A AK BK
AH a AH
Mà = tgB1 nên =
BH b BH
b
b-Theo a ta có HB = .HA
a
trong ∆ vuông BO2H ta có BH2+O2H2=O2B2
nên
b2 b2 b2
2
. AH 2
+ (b − AH ) 2
= b 2
⇔ 2
. AH 2
+ b 2
− 2bAH + AH 2
= b 2
⇔ 2
. AH 2 − 2bAH + AH 2 = 0
a a a
2 2
b 2a b 2ab 2
⇔ 2 . AH − 2b + AH = 0 ⇔ (a 2 + b 2 ) AH = 2a 2 b ⇔ AH = 2 suyra : BH =
a (a + b 2 ) (a 2 + b 2 )

Tài liệu sưu tầm và tổng hợp bản word đầy đủ liên hệ 0393732038 TÀI LIỆU TOÁN HỌC
136
Website:tailieumontoan.com

8a 3b 3
S ABCD = 2. AH .BH =
(a 2 + b 2 ) 2
Câu 4.

4a + 6b
Ta có 3a 2 + 8b 2 + 14ab = (3a + 2b)(a + 4b) ≤ = 2a + 3b
2
4b + 6c
3b 2 + 8c 2 + 14cb = (3b + 2c)(b + 4c) ≤ = 2b + 3c
2
4c + 6a
3c 2 + 8a 2 + 14ac = (3c + 2a )(c + 4a ) ≤ = 2c + 3a
2
Ta có
a2 b2 c2
P= + +
3a 2 + 8b 2 + 14ab 3b 2 + 8c 2 + 14bc 3c 2 + 8a 2 + 14ca
a2 b2 c2
P≥ + + =Q
2a + 3b 2b + 3c 2c + 3a

Áp dụng BĐT Bunhiacopsky cho 2 dãy


a b c
; ; và 2a + 3b ; 2b + 3c ; 2c + 3a
2a + 3b 2b + 3c 2c + 3a
1 1
Ta có Q(5a + 5b + 5c) ≥ (a + b + c) 2 ⇔ Q ≥ (a + b + c) vậy P ≥ Q ≥ (a + b + c)
5 5
Dấu “=” xảy ra khi khi a=b=c

1
Cách khác: Chứng minh Q ≥ (a + b + c) áp dụng BĐT Cô-Si
5
a 2
2a + 3b 2a a 2
2a 2a + 3b 8a − 3b
+ ≥ ⇔ ≥ − = tương tự
2a + 3b 25 5 2a + 3b 5 25 25
b2 8b − 3c c2 8c − 3a
≥ ; ≥
2b + 3c 25 2c + 3a 25
Vậy
8a − 3b 8b − 3c 8c − 3a a + b + c
Q≥ + + =
25 25 25 5
Đề số 22

Câu 1.
1)ĐKXĐ: x ≥ −1
Đặt x + 35 = a.; x + 1 = b; (a > 0; b ≥ 0)
14 x + 35 + 6 x + 1 = 84 + x 2 + 36 x + 35 ⇔ 14a + 6b − 84 − ab = 0
a = 6  x = 1; (T / m)
⇔ (a − 6)(14 − b) = 0 ⇔  ⇔
b = 14  x = 195; (T / m)
2).Dùng phương pháp quy nạp toán học
* Với n = 1 đúng giả sử đúng với n = k ta có

Tài liệu sưu tầm và tổng hợp bản word đầy đủ liên hệ 0393732038 TÀI LIỆU TOÁN HỌC
137
Website:tailieumontoan.com
k2 (k + 1) 2
Sk = ; ta phải chứng minh đúng với n=k+1 nghĩa là S k +1 =
4k 2 + 1 4(k + 1) 2 + 1
2(k + 1) − 1 (k + 1) 2 k2 2(k + 1) − 1
S k +1 = S K + ⇔ = +
4 + (2k + 1) 4
4(k + 1) + 1 4k + 1 4 + (2k + 1) 4
2 2

(k + 1) 2 k2 2(k + 1) − 1
⇔ − =
4(k + 1) + 1 4k + 1 4 + (2k + 1) 4
2 2

(k + 1) 2

k2
=
[
(k + 1) 2 (4k 2 + 1) − k 2 4(k + 1) 2 + 1) ]
Ta có :
4(k + 1) 2 + 1 4k 2 + 1 [ ]
4(k + 1) 2 + 1) (4k 2 + 1)
4k + 8k + 4k + k + 2k + 1 − 4k − 8k − 4k 2 − k 2
4 3 2 2 4 3
2k + 1
= = ; ( DPCM )
16k + 32k + 24k + 8k + 5
4 3 2
4 + (2k + 1) 4
Cách khác:
a 1 1 1 
đặt a = 2n - 1( n ∈ N * ) xét tổng quát =  2 − 2  thay n lần lượt từ
4+a 4
4  a − 2a + 2 a + 2a + 2 
1 ;2;3;4;…. Ta có a lần lượt 1;3;5;7;…..

1 1 1 1 1 1 4n 2
Ta có 4 S = − + − + ............... − = 1 − =
1 5 5 13 (2n − 1 + 1) 2 + 1 (2n − 1 + 1) 2 + 1 4n 2 + 1
Câu 2.

1) Với n < 6 không có số nào thoả mãn


Với n = 6 thoả mãn Với n>6 n có dạng n = 7k;7k+1;7k+2;7k+3;7k+4;7k+5;7k+6 ( (k ∈ N * )
n = 7k thì n+7  7; n=7k+1 thì n+13  7; n=7k+2 thì n+5  7
n=7k+3 thì n+25  7; n=7k+4 thì n+17  7; n=7k+5 thì n+37  7; n=7k+6 thì n+1  7
Vậy chỉ có duy nhất n=6 thoả mãn đề bài
2) Ta thấy a-b  7;(a+c)-(b+d)=((a-b)+(c-d))  7 mà 2014-844=1170=167x7+1
Không chia hết cho 7 vậy sau một số hữu hạn lần thay thế ta không thể nhận được tập
hợp các cặp số M 1 = {(2018,702), (844,2104), (1056,2176), (2240,912)}
Câu 3.

P
A
C

K O O'

B
Q

1) ta có ∠ DCB= ∠ DAB (Chắn cungBD)


∠ DAB = ∠ QPB (chắn cungBQ)
Tài liệu sưu tầm và tổng hợp bản word đầy đủ liên hệ 0393732038 TÀI LIỆU TOÁN HỌC
138
Website:tailieumontoan.com
Nên ∠ DCB= ∠ QPB(1)
∠ DBC= ∠ PAQ( cùng bù ∠ DAC)
∠ PAQ = ∠ PBQ ( chăn cung PQ)
Nên ∠ DBC= ∠ PBQ(2)
Từ (1) ;(2) ta có
∆ BCD đ d với ∆ BPQ (g.g)
2)theo a) thì ∠ DCB= ∠ QPB nên tứ giác KPCB nội tiếp nên đường tròn ngoại tiếp tam giác
KCP luôn đi qua điểm B cố định khi M thay đổi
Câu 4.

Giả sử x,y,z là những số thực thoả mãn điều kiện


0 ≤ x, y, z ≤ 2 và x + y + z = 3
Tìm giá trị nhỏ nhất và lớn nhất của biểu thức :
M = x 4 + y 4 + z 4 + 12(1 − x )(1 − y )(1 − z )
Giải: cách 1 đặt 1-x = a;1 - y = b;1 – z = c thì a + b + c = 0 nên a3 + b3 + c3 = 3abc
và − 1 ≤ a; b; c ≤ 1
M=(1-a)4+(1-b)4+(1-c)4+12abc=a4+b4+c4-4(a3+b3+c3-3abc)+6(a2+b2+c2)-4(a+b+c)+3
M= a4+b4+c4+6(a2+b2+c2)+3
áp dụng BĐT Bunhiacopsky cho 2 dãy a;b;c và 1;1;1 ta có 3(a2+b2+c2) ≥ (a+b+c)2=0
Nên a2+b2+c2 ≥ 0 Dấu “=” xảy ra khi a=b=c=0
áp dụng BĐT Bunhiacopsky cho 2 dãy a2;b2;c2 và 1;1;1 ta có 3(a4+b4+c4) ≥ (a2+b2+c2)2=0 ⇔
a4+b4+c4 ≥ 0 Dấu “=” xảy ra khi a = b = c = 0
Vậy M ≥ 3 suy ra min (M)= 3 khi a=b=c=0 khi đó x=y=z=1
Mặt khác a2+b2+c2=(1-x)2+(1-y)2+(1-z)2=3-2(x+y+z)+x2+y2+z2= x2+y2+z2-3
Giả sử 0 ≤ x ≤ y ≤ z ≤ 2 ; ta có x2+y2+z2-3=(x+y)2-2xy+z2 -3 ≤ (3-z)2 +z2-3=2z2-6z+6
2
 3 3
x +y +z -3=2z -6z+6= 2 z −  + ≤ 2 vì 0<z ≤ 2 suy ra a2+b2+c2 ≤ 2
2 2 2 2

 2 2
ta có a ;b ;c ∈ [0;1] nên (a +b +c ) ≤ ( a2+b2+c2) suy ra M ≤ 7(a2+b2+c2)+3 ≤ 17
2 2 2 4 4 4

Max(M)=17 khi (a;b;c)=(-1;0;1) và các hoán vị hay (x;y;z) =(0;1;2)và các hoán vị

Cách 2: thay x4+y4+z4=(x2+y2+z2)2-2(x2y2+y2z2+z2x2)


Mà x2+y2+z2=(x+y+z)2-2(xy+yz+zx)=9-2a (đặt xy+yz+zx=a)
x y +y z +z x =(xy+yz+xz) -2xyz(x+y+z)=a -6xyz
2 2 2 2 2 2 2 2

12(1-x)(1-y)(1-z)=12(1-x-y-z+xy+yz+zx-xyz)=12a-12xyz-24
Nên M=(9-2a)2-2(a2-6xyz)+ 12a-12xyz-24=81-36a+4a2-2a2+12xyz+12a-12xyz-24
M=2a2-24a+57=2(a-6)2-15 (*)
Ta có x2+y2+z2 ≥ xy+yz+zx ⇔ (x+y+z)2 ≥ 3(xy+yz+zx) nên a ≤ 3
Ta cũng có (2-x)(2-y)(2-z) ≥ 0 ⇔ 8-4(x+y+z)+2(xy+yz+zx)-xyz ≥ 0
⇔ xy+yz+zx ≥ 2+xyz ≥ 2 hay a ≥ 2 vậy 2 ≤ a ≤ 3 thay suy ra -4 ≤ (a-6) ≤ -3
Hay 16 ≥ (a-6)2 ≥ 9 vào (*) 3 ≤ M ≤ 17
Vậy Min(M)= 3 khi a=3 khi đó x=y=z=1;
Max(M) =17 khi a=2 khi đó (x;y;z)=(0;1;2) và các hoán vị

Tài liệu sưu tầm và tổng hợp bản word đầy đủ liên hệ 0393732038 TÀI LIỆU TOÁN HỌC

You might also like